Download as pdf or txt
Download as pdf or txt
You are on page 1of 92

COURSE INTRODUCTION

Algebra, is a word that conjures up a whole world of processes of abstract and


generalised thinking; it is the backbone of mathematical thinking and communication.
By now you will have studied enough algebra to see how it develops these processes
and abilities. You will have also studied groups, rings, fields, vector spaces, matrices
and their basic properties. Based on the assumption that you would be familiar with the
content of the IGNOU undergraduate courses MTE-02 (Linear Algebra) and MTE-06
(Abstract Algebra), we now move ahead. We will refer to the above courses for the
required background in this study guide and you can refer to them in your programme
centre; we have made them available there. You can also view or download the material
online from e-gyankosh at the following URLs:
http://www.egyankosh.ac.in/handle/123456789/14374 for MTE-02.
http://www.egyankosh.ac.in/handle/123456789/14401 for MTE-06.
This course has been specially designed to help you develop a better understanding of
the courses you will study in the second year of the Master’s programme. Thus, it
unfolds very differently from conventional algebra course. Throughout the course, we
place example of concepts and processes before you, and then link them to applications
in other areas of mathematics, as well as in other disciplines.
This course is built around Chapters 5, 6, 8, 9, 13 and 14 of ‘Algebra’ by Michael Artin,
Prentice-Hall, Indian Edition (henceforth referred to as the textbook).We have also
created a Study Guide, comprising two blocks of wrap-around, support material, to help
you understand the content in the textbook. In this study guide, we include units related
to content that is not given in the textbook, like Unit 5 (Applications of Semigroups),
for example.
In the Study Guide we have recommended a path of study which takes you in and out
of the textbook and the Guide. The Guide is basically providing you with clarifications,
further examples, further exercises, solutions to certain exercises in the textbook, and
necessary content that is not available in the text book.
Let us take a brief overview of the content. We start with a discussion on group actions
and congruency. Then we focus on some groups like permutation groups, ‘special’
groups, free groups and free abelian groups. Block 1 of the Study Guide ends with two
specially created self-contained units (i.e., not linked to matter given in the textbook) on
semi-groups and congruences.
The next part of the course introduces you to finite group representations, in which
most of the concepts would be completely new to you. We then go on to discuss field
theory with the focus on finite fields, since it is applications of these fields and their
theory that you would be using later.
A word about the icons and notational conventions we use in the Study Guide. In each
block we list the symbols used in the block after the block introduction. Artin has listed
the symbols used in the textbook towards the end of the book. Next, when we ask you
to read a portion of the book, in the margin we use an icon, as below.

BOOK

When we give you exercises to do, whether from the textbook or any other ones, we
place the icon EXERCISES in the margin next to them. When we give a note or
clarification of comment, we place NOTE in the wide margin next to it. At the
completion of the note we have placed the symbol  to indicate the completion of the
note. Finally, the end of a proof of a theorem is shown by placing a  at the end of the
last line, and the end of an example is shown by placing *** after its solution.
You may find the Preface and Note for the Teachers on Pages xiii-xvii of the textbook
useful as a guide to read the book. 3
Study Guide-I We hope you will enjoy reading this course. If you have any queries, corrections or
comments do not hesistate to contact us through e-mail at the addresses
svenkat@ignou.ac.in and pksinclair@ignou.ac.in
You can also write to us at the following address:
The Course Coordinator
MMT-003, Algebra
School of Sciences
IGNOU
Maidan Garhi
New Delhi-110068
All the best!

4
BLOCK INTRODUCTION
In this block, you will continue the study of Algebra from where you stopped in your
undergraduate course in group theory (See Block 1 and 2, MTE-06 of the course
material.). This study would be done largely from Chapters and of ‘Algebra’, by
Michael Artin, supported by the material given in this block.
You would first study group actions, which is very important not only in abstract areas
of mathematics like the study of Riemann Surfaces, but also in many topics in Discrete
Mathematics which have applications. In the first unit of this block, we have illustrated
this through examples. We use this concept to study the conjugacy classes of a group.
We also use the technique built around group actions in proving Sylow Theorems.
The next concept we discuss is the ’symmetric group’. Apart from proving the well
known result that the alternating group An is simple, we also discuss how to count the
number of conjugacy classes in a symmetric group and how to find them.
In the third unit, we discuss what are called classical groups in mathematics literature.
You already know that invertible matrices over any field form a group with respect to
matrix multiplication. This group is known in mathematics literature as The General
Linear Group. From your study of Linear Algebra in undergraduation and in
MMT-002, you are also familiar with unitary and orthogonal matrices. The set of
unitary matrices form a subgroup of the general linear group over the field of complex
numbers. In unit 3 we focus on this group and its subgroup, the group of special unitary
matrices, which are unitary matrices with determinant one. Apart from these groups we
discuss the special linear group, the group of invertible matrices with determinant one.
The aim of the study is to illustrate some of the concepts like orbits, stabilisers etc. that
we have studied earlier.
The fourth unit has two main topics for discussions, namely, free groups and free
abelian groups. In this unit, we define a free group and prove some elementary results
about free groups. In the sections on free abelian groups we begin by discussing some
basic facts about finitely generated free abelian groups and end the unit with the
structure theorem on finitely generated free abelian groups.
Unit 5 focusses on semigroups, a concept that is usually marginalised in the
undergraduate studies. However, as you will see in the unit, semigroups have a variety
of important applications in the study of automata, linguistics and various social
sciences. This is why we have offered you this opportunity to study them. This is the
first unit which is not a ’wrap’ around some material in the textbook.
The last unit in this block is on congruences. This a part of number theory and we use
this topic to illustrate some of the results that we have studied earlier. Apart from some
of the standard results in congruences like the Chinese Remainder Theorem, we also
prove the quadratic reciprocity law. In the recent times some nice applications of
congruences have been found. We discuss some of them in this unit.

5
Study Guide-I
NOTATION & SYMBOLS
See page 601 of Artin for Notations and Symbols not in this list.

A∗ the conjugate transpose of A, where A is a complex matrix


G+ a group G whose law of composition is addition
G× a group G whose law of composition is multiplication
G1 ⊕ G2 the direct sum of the groups G, and G2
n! the product of the integers 1, 2, . . . , n.
C(n, k) a binomial coefficient
`(x) length of the string x
Map(S, T) the set of all functions from S into T
Mm×n (F) the set of all m × n matrices with entries in the field F
P(X) the power set of the set X
T≤S T is a subgroup (or subsemigroup) of S
<S> the smallest semigroup (or group) generated by the set S.
If S and T are sets, we use the following notations:
S ( T S is a proper subset of T, meaning that it is a subset, and the T
contains an element which is not a member of S.
T ) S this is the same as S ( T.
T\S the complement of S in T
S ∩ T the intersection of the sets S and T
S ∪ T the union of the sets S and T
S × T {(s, t)|s ∈ S, t ∈ T}
ϕ :S→T a map ϕ from S to T, or a function whose domain is S and whose
codomain is T.
s!t The wiggly arrow indicates that the map under consideration
sends the element s to the element t, i.e., that ϕ(s) = t.
(S, A, δ ) the semiautomaton with S the set of states, A the input alphabet
and δ the next-state function
(S, A1 , A2 , δ , λ ) the automaton with (S, A1 , δ ) being a semiautomaton,
A2 the output alphabet and λ the output function
G = (A, G, →, g0 ) the phase-structure grammar with alphabet A, G the set of
grammar symbols
→ the set of rewriting rules and g0 the initial symbol
L(G ) language generated by the phase structure grammar G
∧ empty word

6
UNIT 1 GROUP OPERATIONS
Structure Page No.
1.1 Introduction 7
Objectives
1.2 Group Actions 7
1.3 Conjugacy Classes 13
1.4 Summary 13
1.5 Solutions/Answers 14
Sections in Artin’s book related to this unit:

Chapter Section
5 5, 6 and 7
6 1 and 3

1.1 INTRODUCTION
Permutation groups and matrix groups are among the most basic classes of groups.
These groups are equipped with a natural “action” on a set. For example, a group of
permutations of {1, . . . , n}, i.e., a subgroup G of the symmetric group Sn , acts naturally
on the set {1, . . . , n} in the sense that given any σ ∈ G and i ∈ {1, . . . , n}, we can
associate another element of {1, . . . , n}, namely σ (i). Likewise a subgroup G of the
multiplicative group GLn (F) of n × n nonsingular matrices with entries in a field F acts
naturally on the set Fn of n-tuples of elements of F simply by matrix multiplication.
The notion of group action (or as Artin calls them, group operation) is an abstraction of
the above basic examples. In general, a group G acts (or operates) on a set S if there is a
map
G×S → S
satisfying certain natural properties [See axioms (5.1) on page 176 of Artin.]. It turns
out that a group can have several different actions on different sets and understanding
these can give a better insight into the structure of the group
In this unit we will discuss, in Section 1.2 below, several examples of group actions and
some basic concepts and properties of group actions, in general. Next, in Section 1.3,
we will discuss and illustrate two important tools in studying finite groups, namely, the
so called Counting Formula and the class equation. Further applications of group
actions will be discussed in the next unit.
Objectives
After studying this unit along with Sections 5 and 6 of Chapter 5 and Sections 1 and 3
of Chapter 6 from the Artin’s book, you will be able to
• define group action (or group operation, as per the terminology used by Artin);
• check whether a given map G × S −→ S, where G is a group and S is a set, is a group
action;
• to find the stabilisers and orbits for given actions;
• understand and apply class equation for a finite group.

1.2 GROUP ACTIONS


In this section, we start our formal discussion on group action. We will define a group
action and discuss some examples of group action. 7
Study Guide-I
BOOK Read Section 5 of Chapter 5 from the book.
Let us now look at some examples that illustrate the notion of group action(or as Artin
calls them, group operations).
Example 1: Let F be any field and let GLn (F) denote the group of n × n invertible
matrices as usual.
a) The group GLn (F) acts on Fn by left multiplication. Further, since GLn (F) maps
non-zero vectors to non-zero vectors, there is also an action of GLn (F) on Fn \ 0.
b) Let Mn (F) be the group of n × n matrices. Then, GLn (F) acts on Mn (F) by left
multiplication.
c) The group GLn (F) acts on Mn (F) by conjugation. If g ∈ GLn (F) and x ∈ Mn (F),
we can define an actin by g, x gxg−1 .
In all these actions, it is clear that the n × n identity matrix, which is the identity
element of the group GLn (F), acts as the identity map. So, the first condition of group
action is satisfied. The second condition of the group action, the associativity of the
action, follows from the fact that matrix multiplication is associative.
∗∗∗
Example 2: Let C be the field of complex numbers. Then, GL2 (C) acts on C via
Möbius transformations defined as follows:
a b az + b
 
z= (1)
c d cz + d
Related to this action is the action of GL+
2 (R), the group of 2 × 2 real matrices with
Im(z) denotes the positive determinant, on the upper half plane H = { z ∈ C| Im(z) > 0}. The action is
imaginary part of the the same as in Eqn. (1). From the relation
complex number z. 
az + b

(ad − bc)
Im = Im(z) (2)
cz + d |cz + d|2
it follows that GL+2 (R) maps elements in H to H . You should check that the relation
in Eqn. (2) is correct.
∗∗∗
Example 3: Let G be any group and suppose there is a group action φ : G × S −→ S.
a) If H ⊂ G is any subgroup of G, the restriction of φ to H × S is again an action of H
on S. Although this remark looks trivial, we can derive many interesting actions
from this remark. For example, GL+ 2 (R) acts on the upper half plane. The groups
SL2 (R) and SL2 (Z)(the group of 2 × 2 matrices with integer entries and
determinant 1) act on the upper half plane and they are of interest in geometry.
Also, from the fact that GLn (R) acts on Rn , we get interesting actions of
subgroups of GLn (R).
b) Suppose that G maps a subset of S to itself. In other words, suppose that S1 ⊂ S is
such that φ (g, s) ∈ S1 for all s ∈ S1 , then there is an action of G on S1 .
∗∗∗
Example 4: For any group G, we know that the set of all automorphisms of G forms a
group with respect to composition of maps; it is usually denoted by Aut (G). There is
an action of Aut G on G. In particular, we have the action by the group of inner
automorphisms that we discussed in the introduction. The group of inner
automorphisms is a subgroup of the group of all permutations of G.
So we see that we have obtained a homomorphism G → Aut (G) ⊂ Perm (G)
∗∗∗
Example 5: Take G = Z and the set X to be the set of points in Cartesian plane with
integer co-ordinates (x, y). So this will look like just dots placed in regular intervals
8 both horizontally and vertically in the plane. We can define action of G on X as follows:
Now fix integers m and t. We define a horizontal motion φm on the plane as follows: for Group Operation
the line Lt parallel to the X-axis at height t, (defined by the equation y = t) push every
point of X ∩ Lt right horizontally by a distance tm. When t < 0 we interpret the negative
sign as movement in the opposite direction (left or westward) and use |t| as the
magnifying factor whereas when t = 0, the points are not moved. Think of the lines Lt
as a collection of sticks placed one above other. These points of X are marked on these
sticks at regular distance. Imagine frogs sitting on all these points of the frogs. This
motion may be thought of as all the frogs jumping eastward. All the frogs in a stick
jump by the same amount. But two frogs at different sticks jump differently: frogs at
higher level jump a longer distance. The frog at (x, y) is the one at the stick at height y,
so it will jump to the position φm (x, y) given by (x + ym, y). This is easily expressed in
terms of matrices. Writing co-ordinates as column vectors we can see that
x 1 m x x + ym
      
φm · = =
y 0 1 y y

Notice that (x + (m + n)y, y) = (x + my + ny, y). And so φm+n = φm ◦ φn .


It is also checked that
1 m+n 1 m 1 n
    
= .
0 1 0 1 0 1

So, this gives another example of action of Z, this time on the set of all points on the
plane with integer co-ordinates.
∗∗∗
Example 6: Consider the collection of all partitions of {1, 2, 3, 4, 5} into two subsets of
cardinality 3 and 2. The number of such partitions is simply the number of ways of
choosing the 3 numbers for the first set,
 and so it is given by the binomial coefficient
C(5, 3) = 15. A typical partition, say, {3, 2, 5}, {4, 1} , may be represented in a
diagram of two rows of boxes filled with numbers: (in each row we arrange the
numbers in increasing order).

2 3 5
1 4
Fig. 1: A partition of {1, 2, 3, 4, 5} into two subsets of size 3 and 2.

The group S5 acts on this collection of 15 such diagrams by acting on each box of the
diagram. For example, since 1 and 5 are interchanged under the transposition (1 5), the
positions of 1 and 5 in Fig. 1 are changed to the ones in Fig. 2.

2 3 1
5 4
Fig. 2: Changed partition under the action of (1 5).

More generally, let

{{i11 , i12 , · · · , i1n1 } , {i21 , i22 · · · , i2n2 } , · · · , {ik1 , ik2 , · · · , iknk }} (3)

be a partition of {1, 2, . . . , n} into k subsets, the i-th subset having ni elements and
n1 + n2 + · · · + nk = n. Then, we have an action of Sn on the set of all partitions of this
type. It is easy to check that there are

C (n, n1 ) C (n − n1 , n2 ) C (n − n1 − n2 , n3 ) · · · C (n − n1 − n2 − · · · − nk−1 , nk )
n!
= (4)
n1 !n2 ! · · · nk ! 9
Study Guide-I partitions of the type in Eqn. (3). Note that the order in which we choose the subsets
doesn’t matter, i.e. it doesn’t matter whether we select the subset with n1 elements first
or the subset with n2 elements first etc., because the final formula in the RHS of
Eqn. (4) is independent of the order in which we choose the subsets.
∗∗∗
Try the following exercises to check your understanding of group actions.

EXERCISES E1) Checkthat Eqn. (1) defines an action.

NOTE Please refer to page 176, lines 20 and 21.


The group of rigid motions of a plane is defined as follows:

Definition 1: A map m : R2 −→ R2 is called a rigid motion if

|x − y| = |m(x) − m(y)| ∀ x, y ∈ R2 .

In other words, m preserves distances. Note that a rigid motion is necessarily a 1-1
map.(Why?)
A characterisation of group of rigid motions given in Chapter 5, Section 2, pages
157–162. You may browse through it if you are interested. 
NOTE Please correct equation (5.3) on page 177.
The equation should be

Os = s0 ∈ S s0 = gs for some g ∈ G



NOTE Please refer to page 177, lines 9—11.
Note that, since a rigid motion preserves length, it preserves the lengths of the sides of a
triangle. So, it maps a triangle to a congruent triangle. 
The concepts of orbit, stabiliser and transitive action are discussed in page 177 of
Artin’s book. Here are some examples to help you understand these concepts.
Example 7: Consider the action of G = GLn (F) on Fn given by left multiplication.
The element 0 is left n
 fixed by all the elements. Let v ∈ F be any t
non-zero vector.
For a row vector v, vt Suppose A = aij ∈ GLn (F) is in Ge1 , where e1 = (1, 0, . . . , 0) . Then, the first column
denotes its transpose. of A is the vector e1 , and so A is of the form
n−1 entries
 
 1
z }| {
∗ ∗ ···∗ 
 0
 

B
 0 (5)
 

 
 .. 
 . 
0

where B is a (n − 1) × (n − 1) invertible matrix with det(A) = det(B). ( You can check


this by finding the determinant of A by expanding along the 1st column.) Conversely,
any matrix of this form fixes e1 .
Further, the action of GLn (F) on Fn \ {0} is transitive. We can show this as follows: Let
v and w be any two non-zero vectors. Then, we can extend the singleton sets {v} and
{w} to bases B1 = {v1 = v, v2 , . . . , vn } and B2 = {w1 = w, w2 , . . . , wn }. The matrix
that maps B1 to B2 is an invertible matrix since it maps a basis to another basis.
Further, it maps v to w. So, v and w are in the same orbit. Thus all the non-zero
10 elements of Fn are in the same orbit.
∗∗∗ Group Operation

Example 8: Let us go back to Example 6. It is easy to check that S5 acts transitively


on the set of all partitions of {1, 2, 3, 4, 5} into two subsets of cardinality
 3 and 2 and
there is just oneorbit. To find a permutation that moves the partition {1, 2, 3}, {4, 5}
to the partition {3, 2, 5}, {4, 1} , just paste the two rows of the diagram into one long

1 2 3 4 5
 

row: ↓ ↓ ↓ ↓ ↓.
3 2 5 4 1
The stabiliser subgroup of any such partition above is also easy to describe: it consists
of those permutations that do not mix the contents of one row with another. They are all
isomorphic to S3 × S2 .
In general, the stabiliser of a partition of type given in Eqn. (3) is isomorphic to
Sn1 × Sn2 × · · · × Snk where n1 , n2 , . . . , nk is a partition of n.
∗∗∗
Here are some exercises to test your understanding of the concepts of transitivity and
stabiliser.

E2) Find thestabilisers for the following elements under the action given: EXERCISES
a) Stabiliser of the vector e1 = (1, 1, . . . , 1)t ∈ Fn under left multiplication by
elements in GLn (F).
1 1
 
b) Stabiliser of the element ∈ M2 (R) under left multiplication by
0 0
elements in GL2 (R).

E3) Which of the actions amongst b) and c) in Example 1 are transitive? Give reasons
for your answer. Describe the orbits under the action given in b), Example 1.

After finishing Sec. 5, try exercises 2, 4, 5 and 9 in page 192 of Artin’s book under
the heading ‘5. Abstract Symmetry: Group Operations.’ EXERCISES
Read Section 6 of chapter 6. BOOK

Please refer to the example that starts from 7th line from the bottom in page 178
and end at 6th line from the top in page 179. NOTE
Cayley’s theorem tells us that we can embed D3 in S6 . However, from the discussion
mentioned above, we see that we can do better; we can actually embed D3 in S3 , a
smaller permutation group. We did this by considering the action of D3 on the cosets of
one of its subgroups. So, the question arises: When can we find such an embedding?
Let G be a finite group and let H be its subgroup. As we saw, there is an action of G on
the (left) cosets of H in G.

g · xH = gxH (6)

This gives a homomorphism τ : G −→ S(G/H). Let K be the kernel of this


homomorphism. If g ∈ K, then g is mapped to identity under this map, and so
gxH = xH for all x ∈ G. In particular, gH = H, so g ∈ H. This also means gx = xh for
some h ∈ H. So, x−1 gx ∈ H for all x ∈ G. Put differently, g ∈ xHx−1 for all x ∈ G.
Let N be a normal subgroup of G contained in H. We will show that N is contained in
the kernel of the homomorphism G −→ S(G/H). Let n ∈ N and x ∈ G. Since
N = xNx−1 , n = xn0 x−1 or nx = xn0 for some n0 ∈ N. So, nxH = xn0 H = xH since
n ∈ N ⊂ H. So, N ⊂ K. From our discussion so far, we get the following result:
11
Study Guide-I Proposition 1: Let G be a finite group and H be its subgroup. There is a
homomorphism

τ : G −→ S(G/H) (7)

The kernel of τ is the largest normal subgroup of G contained in H.

If G is a group and H is a Coming back to our original question, we see that if G has a subgroup H which is not
subgroup of G, [G : H] normal and doesn’t contain any non-trivial subgroup of G, then we can embed G in Sn
denotes the index of H in G. with n = [G : H]. In particular, if H is a subgroup of G which is simple and non-normal,
we can embed G in Sn with n = [G : H]. 
For notational convenience, let us write [G : H] for the index of H in G. If the map τ
defined in Eqn. (7) is injective, then |G| divides [G : H]!. Put differently, if |G| doesn’t
divide [G : H]!, then τ is not injective and so its kernel is non-trivial. Since the kernel of
a group homomorphism is a normal subgroup, G contains a non-trivial normal
subgroup which is contained in H. Let us summarise this as a proposition.

Proposition 2: let G be a finite group and H be its subgroup such that |G| doesn’t
divide [G : H]!. Then H contains a non-trivial normal subgroup of G.
Try the next exercise which is an application of Proposition 2.

EXERCISES E4) If H is a (proper) subgroup of G of index p where p is the smallest prime dividing
|G|, then H is a normal subgroup of G.

EXERCISES Do exercises 1, 2 and 6 under the section ‘6. Operation on Cosets.’

BOOK Read Section 7 of Chapter 6. Omit example 7.4

NOTE Please refer to Proposition (7.2) on page 180, Counting formula.


Note that the formula |G| = |Gs ||Os | makes sense only when G and S are finite. The
other form |Os | = [G : Gs ] makes sense even when G and S are infinite but the
quantities involved in both sides of the equation are finite. 
Let us now apply counting formula to one of the examples we discussed earlier.
Example 9: Let us look at part a), Example 7. We saw that GLn (F) acts transitively on
non-zero vectors and any matrix that stabilises A is of the form given in Eqn. (5).
Suppose F is a finite field with q elements. Let us use the formula

(order of the group) = (order of the stabiliser) × (order of the orbit) (8)

There are qn − 1 vectors in Fn \ {0} and the orbit of e1 contains all the qn − 1 elements.
If we know the order of the stabiliser of e1 we can find the order of the group
G = GLn (F). Note that, there are qn−1 ways of choosing the last n − 1 entries in the
first row of the matrix in Eqn. (5) whereas B can be any element of GLn−1 (F). So, the
order of the stabiliser of e1 is qn−1 |GLn−1 (F)|. Now, using Eqn. (8), we get

|GLn (F)| = qn−1 |GLn−1 (F)| (qn − 1) (9)

You can find the order of GLn (F) by successively using Eqn. (9). We leave the details
as an exercise to you.
∗∗∗
Here are some exercises for you.

12 EXERCISES E5) a) Use induction to prove that


n(n−1)
|GLn (F)| = q (qn − 1) qn−1 − 1 · · · (q − 1)

2 Group Operation

= qn − qn−1 qn − qn−2 · · · (qn − 1)


 

where F is a finite field with q elements.


b) Find the order of SLn (F) where F is the finite field with q elements (Hint:
Use the determinant map det : GLn (F) −→ F∗ ). For a field F, we will write
F∗ for F \ {0}.
Do exercises 5 and 6 in page 194 under the heading ‘7. Counting formula’. EXERCISES
We will see more applications of the counting formula in the material that follows. We
close this section here.

1.3 CONJUGACY CLASSES

Read Section 1 of Chapter 6, starting from line 3 of page 198 and section 6. BOOK

Do problems 1, 2, 3, 4, 6 and 7 under the heading ‘1. The operations of a group on


itself.’ EXERCISES
In Artin’s book, the class equation of icosahedral group is determined in Section 3 of
Chapter 6 and the discussion is from a geometric viewpoint. Since the prerequisites for
this discussion is not generally part of the mathematics syllabus of the degree courses in
India, we will omit the section. However, we will discuss some of the useful material
from that section that we will need later.

Definition 2: A group G is simple if it has no subgroups other than {1} and G itself.

Lemma 1:
a) Let G be a group and N a normal subgroup of G. If N contains an element x ∈ G,
then it contains the conjugacy class Cx of x in G. In other words, a normal
subgroup is a union of conjugacy classes in G.
b) The order of N is the sum of the orders of the distinct conjugacy classes of G that
are contained in it.

Proof: a) By the definition of a normal subgroup, if x ∈ N, gxg−1 ∈ N, ∀g ∈ G. In


other words the conjugacy class Cx of x in G is contained in N. Since conjugacy
classes form a partition of G, every element of N must be in some conjugacy class
of G and N will contain the whole conjugacy class in G of that element.
b) This follows immediately from a) since conjugacy classes are mutually disjoint.


Read Section 3 of chapter 6. BOOK

Do exercises 1, 5, 9, 13 under ‘3. Operations on Subsets in pages 230 and 231. EXERCISES
We conclude this unit here. We will summarise the contents of the unit in the next
section.

1.4 SUMMARY

In this unit we discussed the following: 13


Study Guide-I 1) The definition of a group G acting on a set S.
2) The stabiliser of an element s ∈ S under a group action is Gs = {g ∈ G | gs = s}.
3) The orbit of an element s ∈ S is Os = {gs | g ∈ G}.
4) The counting formula [G : Gs ] = |Os | and |G| = |Os | |Gs |.
5) The conjugacy class of a group, the class equation |G| = |Z| + ∑g∈G |G| and some
applications of the class equation.

1.5 SOLUTIONS/ANSWERS

E1) You can check easily that the identity matrix acts as the identity map. To check
the associativity, we compute:
  0 0   0
a b a b aa + bc0 ab0 + bd0
 
z= z
c d c0 d0 ca0 + dc0 cb0 + dd0
(aa0 + bc0 )z + ab0 + bd0
=
(ca0 + dc0 )z + cb0 + dd0
a0 z + b0
 0 0
a b
0 0 z= 0
c d c z + d0
0 0
a ac0 z+b
z+d0 + b a(a0 z + b0 ) + b(c0 z + d0 )
  0 0  
a b a b

∴ z = =
c d c0 d0 c ca0 z+d
0 z+b0
0 +d c(a0 z + b0 ) + d(c0 z + d0 )
Regrouping the terms, RHS in the equation above is
(aa0 + bc0 )z + ab0 + bd0
=
(ca0 + dc0 )z + cb0 + dd0
 0
aa + bc0 ab0 + bd0

= z
ca0 + dc0 cb0 + dd0
E2) a) Writing G = GLn (F), the stabiliser of e1 is
( )
n
Ge1 = aij ∈ GLn (F) ∑ aij = 1, for i = 1, 2, · · · , n.

j=1

a b 1 1
   
b) If is in the stabiliser of , then check that a = 1, c = 0, and
c d 0 0
b, d are arbitrary.
E3) Neither action is transitive because a singular matrix can never be carried to a
non-singular matrix under either action. Regarding the action in b), since any
invertible matrix can be reduced to the identity matrix through row reduction, the
invertible elements are in a single orbit that contains the identity matrix. If a
matrix is not invertible, its orbit contains a unique matrix in the reduced row
echelon form. (Refer to Section 2, Chapter 2 of Artin’s book, particularly
Proposition (2.18) in page 16.)
Regarding the action in c), the orbits are simply the similarity classes of matrices.
We can describe the orbits under the conjugacy action using the rational canonical
form if the field F is not algebraically closed. If F is algebraically closed, we can
describe the orbits using the Jordan form also.
E4) Let K be the kernel of the homomorphism in Eqn. (7). We will show that H = K.
The subgroup K is non-trivial because |G| - p!. To see this note that (|G|, p!) = p
and |G| 6= p. So, there are prime factors of |G| that do not divide p!. So, |G| - p!.
Next, we need to show that [H : K] = 1. We have [G : K] | [G : H]! = p!. But,
[G : K] = [G : H][H : K] = p [H : K]. Thus [H : K] | (p − 1)! and therefore all its
prime divisors are < p. Since p is the smallest prime divisor of |G|, it follows that
14 [H : K] = 1 or H = K.
E5) a) When n = 1, |GL1 (F)| = |F∗ | = q − 1.Therefore, the result is true for n = 1. Group Operation
Suppose the result is true for n:
n(n−1)
|GLn (F)| = q (qn − 1) qn−1 − 1 · · · (q − 1) (10)

2

Then,

|GLn+1 (F)| = qn |GLn (F)| qn+1 − 1 by Eqn. (9).




So, by induction hypothesis,


n(n−1)
|GLn+1 (F)| = qn q qn − 1 qn−1 − 1 · · · q − 1 qn+1 − 1
   
2

n(n+1)
=q qn+1 − 1 qn − 1 qn−1 − 1 · · · q − 1
   
2

The result now follows by induction.


b) The map det : GLn (F) −→ F∗ is a surjective group homomorphism. (Check
the
surjectivity.)
The kernel is precisely SLn (F). Therefore,
GLn (F) ∗
SLn (F) = |F | = q − 1. It is fairly straightforward to find the order of
SLn (F) from this relation.
Solutions to selected exercises from Section 5, ‘Abstract symmetry: group
operations’ on pages 192 and 193.

Q2) Reflexivity: Identity element 1 of G acts as identity permutation, 1 · s = s, hence


s ∼ s.
Symmetry: Given s ∼ s0 . That is there is a g ∈ G with g · s = s0 . But we know that
s = 1 · s = (g−1 ◦ g) · s = g−1 · (g · s) = g−1 s0 .
Transitivity: Given s ∼ s0 and s0 ∼ s00 . This means there are elements g1 , g2 such
that s0 = g1 · s and s00 = g2 · s0 . Now the axiom of group actions says that
(g2 ◦ g1 ) · s = g2 · (g1 · s) = g2 · s0 = s00 . This shows s00 ∼ s with s00 = g · s with
g = g2 ◦ g1 .

Q4) Let the action G0 on S be denoted by the symbol ∗. To define an action of G on


the same set (denoting by ·), we use ϕ.
def
g · s = ϕ(g) ∗ s

This ∗ satisfies the conditions of group action because ϕ is a group


homomorphism:

(g1 ◦ g2 ) · s = ϕ(g1 g2 ) ∗ s = ϕ(g1 ) ∗ (ϕ(g2 ) ∗ s) = g1 · (g2 · s)


1 · s = ϕ(1) ∗ s = 1 ∗ s = s
(though we use the same symbol 1 to denote the identity elements of both the
groups the meaning should be clear).

Q5) Let G = D4 be the dihedral group of symmetries of the square.

a) Stabiliser of a vertex: No rotation fixes a vertex. So it has to be a


reflection, and one whose axis goes through that vertex. So it would be a
group of order 2, consisting of that reflection about the diagonal through
that vertex and identity.
Stabilizer of an edge: Rotations disturb all lines. And a reflection fixes all
lines except its axis. But these axes don’t contain the edges. So the
stabilizer of an edge has to be trivial group. 15
Study Guide-I b) Stabiliser of a diagonal is the group of order 2 generated by the reflection
that has that diagonal as an axis.

Q9) a) Using row echelon form, the orbits are of the form GL2
(C)x where x is
1 α
either the identity matrix or a matrix of the form .
0 0
b) Using Jordan canonical form, the orbits are of the form

PAP−1 P ∈ GL2 (C)




where A has one of the following forms:

α 1 α 0 α 0
     
, ,
0 α 0 α 0 β

where α and β are distinct complex numbers.

Q10) a) We apply Proposition (2.9), b) in page 114. We leave it to you to work


out the details.
b) We are going to use block multiplication of matrices in the solution. If you
are not familiar with this, see page 8 of Artin. Let A be m × n matrix with
m ≤ n. We can write A in the form [A1 | A2 ] where A1 is m × m matrix
and A2 is a m × (n − m) matrix. Let B be a n × n matrix. Let Q be a
non-singular n × n matrix and letB = Q−1 . We  can write view B as made
B1 B2
up of block matrices in the form , where B1 is a m × m
B3 B4
matrix, B2 is a m × (n − m) matrix and B3 is a (n − m) × m and matrix and
B4 is a (n − m) × (n − m) matrix. Now, if P is a m × m matrix, then

B1 B2
 
−1
PAQ = P[A1 | A2 ]
B3 B4
B1 B2
 
= [PA1 | PA2 ]
B3 B4
= [PA1 B1 + PA2 B3 | PA1 B2 + PA2 B4 ] (11)

Here, we have multiplied the matrices as if we are multiplying a scalar, a


1 × 2 matrix and a 2 × 2 matrix. Go through the calculation above
carefully and check that we have multiplied compatible matrices only in
all the steps.
Now, if A = [I|0], then A1 = I and A2 = 0. From Eqn. (11), we get
P[I | 0]Q−1 = [PB1 | PB2 ]. If (P, Q) is in the stabiliser of [I | 0], then
PB1 = I and PB2 = 0. So, P = B−1 1 and B2 = 0.

Solutions to selected exercises from ‘6. The Operation on Cosets.’ in


pages 193 and 194.

Q1) It is those elements g ∈ G such that g.aH = aH in G/H. This happens if and only
if a−1 ga ∈ H, that is g ∈ aHa−1 . So the stabilizer is the subgroup aHa−1 of G.

Q2) We are given that left multiplication by x fixes every coset of the cyclic
subgroup H generated by x in G. That is, xgH = gH for all g ∈ G. So,
x2 gH = x(xgH) = xgh. Repeatedly applying this we see that xn gH = gH for all
g ∈ G, n ∈ Z. That is, g−1 xn g ∈ H for all g ∈ G. As H is the group generated by
x, this is precisely the statement of normality of H.

16 Q6) Proof of Prop.(6.5) in page 179, Artin’s book:


a) Let g ∈ G be such that gs = s0 . Then a−1 g s = a−1 (gs) = a−1 s0 = s since

Group Operation
as = s0 . It follows that a−1 g ∈ Gs or g = ah for some h ∈ Gs .
If g = ah, gs = ahs = as = s0 since hs = s.
b) If g ∈ Gs0 , a−1 ga s = a−1 g (as) = a−1 g (s0 ) = a−1 (gs0 ) = a−1 s0 = s.
  

∴ a−1 ga ∈ Gs . We leave it to you to check that aga−1 ∈ Gs0 when g ∈ Gs .

Solutions to selected exercises from ‘7. The Counting Formula.’ in pages


194.
Q5) Let n ≥ 1, m ≥ 1 be positive integers such that there are distinct coset
representatives g1 , g2 , . . . , gn of H in G and h1 , h2 , . . . , hm of K in H. Then, we
claim that, if (i, j) 6= (p, q) then gi hj K 6= gp hq K. If i = p, then, from
gi hj K = gp hq K we get hj K = hq K and so hi and hq are in the same left coset of K
in H. But, this contradicts our choice of h0i s. You can check that, i 6= j, from
gi hj K = gp hq K, it follows that gi and gp are in the same left coset of H in G,
again contradicting our choice of gi s.
Suppose that [G : H] and [H : K] are finite. Then, we can find n distinct coset
representatives of H in G and m distinct coset representatives of K in H. If we
show that the nm cosets hi gj K exhaust all the left cosets of K in G, it follows that
[G : K] is finite and [G : K] = [G : H][H : K]. Let gK be a left coset of K in G.
Then g = gi h for some i and some h ∈ H. Again, h = hj k for some j and some
k ∈ K. So, gK = gi hj kK = gi hj K and the left cosets gi hj K exhaust all the left
cosets of K in G.
Note that another way of interpreting the problem is as follows: If [G : K] is
finite, [G : H] and [H : K] are finite and [G : K] = [G : H][H : K]. This can also be
proved using a similar argument. You have to prove that [H : K] and [G : H] are
finite using the fact that gi hj K are distinct, We leave this as an exercise to you.
Q6) a) Consider the action of H on the left cosets of K in G. The orbit of K,
considered as a left coset of K in G, is finite since the number of cosets of
K in G is finite. The stabiliser of K under the action of H is precisely
H ∩ K. From the relation |Os | = [G : Gs ] in the 8th line from the bottom on
page 180 of Artin’s book, it follows that [G : H ∩ K] is finite.
b) Let G be the group S4 , H = h(1, 2, 3)i, H = h(2, 3, 4)i. Then, check that
H ∩ K = {1}. Then, [H : H ∩ K] = 3 and [G : K] = 24 3 = 8.

Solutions to selected exercises from ‘1. The operations of a group on itself.’


on page 229.
Q1) 1 · x = x, the first condition is satisfied.
(g1 g2 ) · x = x (g1 g2 )−1 = xg−1 −1
1 g2
= g1 · xg−1 = g1 · g2 (g2 x)

2

So, the second condition is satisfied.


Q2) Let g ∈ G.
Og = {hg|h ∈ H}
is the right coset of H in G.
Q3) Since Cg = 1 if and only if g ∈ Z, it follows that

|G| = ∑ |C| + ∑ |C|


|C|=1 |c|>1

∑ 1 + ∑ Cg

=
g∈Z g6∈Z

= |Z| + ∑ |C| 17
Study Guide-I where the second sum is over the conjugacy classes with more than one element.
Q4) Suppose |G| = pn , n ≥ 1. For each s ∈ S, |Os | = [G : Gs ]. Since [G : Gs ] | |G|,
[G : Gs ] = pk , 0 ≤ k ≤ n. We have
s fixed by G ⇔ G = Gs
⇔ [G : Gs ] = 1
So, we have to show that [G : Gs ] = 1 for at least one s. If this is not true, each
orbit will have pk elements for some 0 < k ≤ n, i.e. order of each orbit will be
divisible by p. So, the number of elements in S, which is the sum of the order of
the orbits, will also be divisible by p, which is a contradiction to the assumption
p - |S|.
Q6) 1 + 1 + 1 + 2 + 5 is not possible because |Z| = 3 and |Z| must divide 10.
1 + 2 + 2 + 5 is possible. Check that this is the class equation of D5 .
1 + 2 + 3 + 4 is not possible because there is a conjugacy class of order 3 and 3
does not divide 10, the order of the group.
1 + 1 + 2 + 2 + 2 + 2 is not possible. Here |Z| = 2. So, 2 | |Z(g)| for any g ∈ G,
|G|
therefore |Cg | = |Z(g)| | 5. So, there cannot be any conjugacy class with 2
elements.
1 0 a b
   
Q7) Let A = and X = be any matrix in GL2 (F5 ). Then
0 2 c d
0 −b
 
AX − XA = . So, AX = XA iff b = 0, c = 0. Since ad − bc 6= 0, this
c 0
implies that
a 0
  

Z(A) = a, d ∈ F5
0 d

So, |Z(A)| = |F∗5 × F∗5 | = 16. Since 2


GL2 (F5 ) = 5(5 − 1)(5 − 1) = 480, the order
GL2 (F5 )
of the conjugacy class is Z(A) = 30.

Solutions to selected exercises from ‘1. The operations on subsets.’ on page


230.

Q1) Let
S = { {a, b}| a, b ∈ D3 }
There are C(6, 2) = 15 subsets of D3 with two elements, i.e. |S| = 15.
What are the possible sizes of orbits? We know that the size of an orbit of an
element is the index of its stabiliser in the group by counting formula. Since D3
has order 6, the possible sizes for orbits are 1, 2, 3 and 6.
The elements of S which have orbit of length 1 are the subsets of order 2 of D3
which are stabilised by conjugation. But, according to Proposition 3.2, any
subset of D3 stabilised under conjugation should be a union of conjugacy classes.
Using the notation in Proposition 3.6 in page 165, we know that the conjugacy
classes in D3 are {1}, {x, x2 }, {y, xy, x2 y}.(See page 198.) So, using Proposition
3.2 in page 203, the only set of two elements fixed by D3 is the conjugacy class
{x, x2 }.(Why?)
Consider sets of the form {1, g} where g ∈ D3 . There are 5 such sets. They are
divided into 2 orbits:
{{1, x}, {1, x2 }} and {{1, y}, {1, xy}, {1, x2 y}}

Now, consider the subsets that do not contain identity and contain x or x2 , but not
18 both. There are six such subsets, three containing x and three containing x2 .
1) Show that the stabiliser of {x, y} in D3 is Z(x) ∩ Z(y). Group Operation

2) Show that Z(x) = {1, x, x2 } and Z(y) = {1, y} and deduce that the
stabiliser of {x, y} is trivial.

So, all these six subsets are in a single orbit.

There are three more sets, {y, yx}, {y, yx2 } and {yx, yx2 } and they form a single
orbit because there are no orbits of length one other than {x, x2 }.

Q5) No. Take G = D3 , and U = x, x2 , y, yx, yx2 . Then |G| and |U| are coprime, but


U is stabilised by the whole group G since it is the union of conjugacy classes.


1
 

 1  

. .. 
Q9) Let A =   where the vacant spaces contain zeros. Note
 

 1 

 1 
1
that A interchanges e1 and en , e2 and en−1 etc. Since A has order 2, note that
A−1 = A. Check that if B is upper triangular, ABA−1 is lower triangular and
viceversa.

Q13) a) Let H = {1, g}. Then, H is the union of conjugacy classes of G. The
element 1 will be in a conjugacy class with just one element. So, the other
element g is also in a conjugacy class with just one element. Thus, g is in
the centre of H.
b) The argument is similar. Again the identity element will be in a conjugacy
class with one element. The order of the every conjugacy class of G that is
contained in H is at most p − 1 and hence strictly less than p. Since p is the
smallest prime divisor of the order of G, the order of the conjugacy class
has to be 1.

19
Study Guide-I

20
UNIT 2 SYLOW THEOREMS AND
THE SYMMETRIC GROUP
Structure Page No.
2.1 Introduction 21
Objectives
2.2 Sylows Theorems 21
Applications of Sylow Theorems
2.3 Symmetric Group 23
Conjugacy Classes in Sn
An is Simple
2.4 Summary 28
2.5 Solutions/Answers 28
Sections in Artin’s book related to this unit:

Chapter Section
6 4,5 and 6

2.1 INTRODUCTION
In your degree course in algebra, you have learnt Lagrange’s theorem which says that
the order of any subgroup of a finite group divides the order of the group. It is natural to
ask if the converse is true, i.e., if G is a finite group and d a positive integer such that d
divides the order of G, then does G have a subgroup of order d.
It turns out that the converse is not true, in general. For example, the alternating group
A4 of order 12 has no subgroup of order 6.(See, for example, page 38 of Unit 2 of
MTE-06.) In Sec. 2.3 of this unit, we will discuss a theorem due to the Norwegian
mathematician Sylow which states that for any prime p, if pn is the highest power of p
dividing the order of G, then G has a subgroup of order pn .
The study of the symmetric group is important because every finite group is a subgroup
of a symmetric group. Symmetric groups are also a rich source of examples and
counter examples. In your degree classes, you would have already learned some P. L. M. Sylow
elementary properties of symmetric groups. In Sec. 2.3 of this unit, we will discuss (1832–1918)
some more results about symmetric groups. In particular, we will see how to find the
order of conjugacy classes in the symmetric group.
Objectives
After reading this unit together with the prescribed material from the Artin’s book, you
will be able to
• apply Sylow theorems to prove the existence of groups of specified order;
• explain the relationship between conjugacy classes in Sn and partitions of n and use
the relationship to describe the conjugacy classes in Sn ;
• explain when does a conjugacy class in Sn of an element in An splits into two
conjugacy classes in An ;
• calculate conjugacy classes and class equation of Sn and An ;

2.2 SYLOW’S THEOREMS


As we will see in Unit 5, where we prove the structure theorem for finite abelian
groups, the finite abelian groups are easier to classify. Non-abelian groups are lot more
difficult. The Sylow theorems provides very basic tools for understanding the structure
of finite non-abelian groups. 21
Study Guide-I Read section 4 of Chapter 6 in Artin’s book. Attempt all the problems except problem
BOOK 5 under ‘4. The Sylow Theorems’ in pages 231 and 232 of Artin’s book. In problem
5, try only part (a).

2.2.1 Applications of Sylow Theorems

Sylow theorem is often useful in checking if a finite group is simple.


As the conjugates of Sylow subgroups are again Sylow subgroups, if we can show that
there is only one Sylow subgroup for some prime p, then that subgroup should be
normal. So the group is non-simple.
Example 1: Show that a group of order 28 is not simple.
Solution: By the third Sylow theorem, the number of 7-Sylow subgroups should be
divisible by 4 and also be 1 modulo 7. Such a number has to be 1. Thus, the 7-Sylow
subgroup is unique, hence a normal subgroup.
∗∗∗
Let us do another, slightly more complicated, example.
Example 2: Show that a group |G| of order 300 is not simple.
Solution: We will show that it will have a normal subgroup. Factorising |G| we get
300 = 3 × 4 × 25. Let us see how many 5-Sylow subgroups are there. That number has
to divide 12 and also be 1 modulo 5. There are two possibilities, 1 and 6. If it is 1, there
is a unique 5-Sylow subgroup, giving a normal subgroup.
The other possibility is there being six 5-Sylow subgroups. Let us call these six
5-Sylow subgroups H1 , H2 , H3 , H4 , H5 and H6 . Let S = {H1 , H2 , H3 , . . . , H6 }. Now G
acts on S by conjugation.(Check this!) Hence we get a group homomorphism

ψ : G −→ S6

given by

ψ(g)(i) = j, if gHi g−1 = Hj

It is clear that ψ does not map every g ∈ H to the identity permutation.(Why?)


If this map were injective, it would mean the image, a subgroup of order 300, is inside
the group of order 720 = 6! But as 300 does not divide 720 the homomorphism cannot
be injective, and it has a non-zero kernel. That kernel will provide a non-trivial normal
subgroup of G we are looking for, proving that G is not simple in this case too.
∗∗∗
A more elaborate example is discussed in Chapter 6, Section 5 of Artin’s book. Go
through the section now.
Read Section 5 of Chapter 6. After that, come back and read the explanation given
BOOK
for some of the more difficult parts of this section.
NOTE Lines 16 and 17 from the top in page 210 of Artin’s book. ‘The counting formula
shows that |N (Ki )| = 3, and hence that N (Ki ) = Ki .’
Explanation: Since any two 3-Sylow subgroups are conjugate, G acts transitively on
{K1 , K2 , K3 , K4 }. So, by counting formula,

12 = |G| = |GKi | |OKi | = 4 |GKi |

and hence GKi = N(Ki ) has cardinality 3. But, since |Ki | = 3 and Ki ⊂ N (Ki ), we
obtain N(Ki ) = Ki . 
22 NOTE Lines 17 and 18 from the top in page 210 of Artin’s book. ‘Since the only element
common to the subgroups Ki is the identity element, only the identity element stabilises Sylow Theorems and The
Symmetric Group
all of these subgroups.’
Explanation: If g ∈ G stabilises all the Ki ,
4 4
g∈ N (Ki ) = Ki = {1},
\ \

i=1 i=1

where the first equality follows since N (Ki ) = Ki for 1 ≤ i ≤ 4. So, the identity element
is the only element that stabilises all the Ki . 
Lines 20 and 21 on page 210 of Artin’s book.‘Since G has four subgroups of order 3, NOTE
it contains eight elements of order 3 and they certainly generate the group.’
Explanation: The group generated by elements of order 3 has order at least 8 and its
order divides 12, the order of the group, by Lagrange’s Theorem. So, the subgroup
generated by elements of order 8 in G has order 12. 
Attempt all the exercises under ‘5. The Groups of Order 12.’ in page 232 of
Artin’s book. EXERCISES
In the next section, we will discuss the symmetric group.

2.3 THE SYMMETRIC GROUP

You would have already studied about symmetric groups in your degree course. It is an
important class of groups. In this section, we will discuss the conjugacy classes of
symmetric groups. In particular, we will determine the conjugacy classes in A4 , S4 and
A5 . We will also show that An is simple for n ≥ 5.
But, before we do that, we ask you to read the material on symmetric groups in Artin’s
book. You may be already familiar with many of the results from your degree course.
However, reading the material will help you to quickly recapitulate what you have
studied earlier and also help you in familiarising yourself with the notation used in the
Artin’s book.

Read section 6 of chapter 6 from the book BOOK

The definition of sign of a permutation in Artin’s book is different from the one given
in MTE-06 and in many other books. So, browse through pages 24, 25 and 26 of
Artin’s book to familiarise yourself with this definition. NOTE
Attempt exercises 2, 3, 4, 5, 6, 7, 8, 10, 15 and 16 for this section in pages 232
and 233. EXERCISES
Important: Note that, as in Artin’s book, we read the permutations left to right. In
other words, the product of two permutations σ τ means that we apply σ first and τ
next.

2.3.1 Conjugacy Classes in Sn

In view of the fact that every permutation in Sn can be uniquely written as a product of
disjoint cycles[Prop. (6.6) on page 213 of Artin], we make the following definition.

Definition 3: The cycle-type of p ∈ Sn is defined to be the tuple (e1 , e2 , . . . , eh ) where


e1 , . . . , eh are integers satisfying e1 ≥ e2 ≥ · · · ≥ eh ≥ 1 and e1 + e2 + · · · + eh = n such
that

p = σ1 σ2 · · · σh

where σ1 , σ2 , . . .,σh are disjoint cycles in Sn and the length of σi is ei for i = 1, 2, . . . , h. 23


Study Guide-I Note that, the cycle-type of a permutation in Sn is a partition of n. Suppose we have a
partition

e1 + e2 + · · · + eh = n of n with e1 ≥ e2 ≥ · · · ≥ eh ≥ 1 (1)

Then, we can always find a cycle p ∈ Sn with cycle-type (e1 , e2 , . . . , eh ). (We will
always write the parts of a partition in non-increasing order.) In this terminology,
Prop. (6.10) on page 214 of Artin can be summarised as follows.

Proposition 1: Two permutations in Sn are conjugate to each other if and only if they
have the same cycle type.
We have the following corollary:
The number p(n) grows
rapidly with n. Finding an Corollary 1: The number of distinct conjugacy classes in Sn = p(n) where p(n)
exact formula for p(n) is not
easy. A remarkable formula
denotes the number of partitions of n.
has been obtained with Having found the number of conjugacy classes in Sn , let us now find the number of
essential contributions by elements in a conjugacy class. As you would expect, this also depends on the cycle
the Indian mathematician structure of the partition corresponding to the conjugacy class. Let us denote the
S. Ramanujan; this is known conjugacy class of an element σ ∈ Sn by [σ ]. To determine the number of elements in a
as the Hardy-Ramanujan- conjugacy class in Sn , we need to know the number of permutations that correspond to
Rademachar formula, which a given partition. Again, we will make use of group actions for counting.
can be found, for example,
in the book ‘Theory of Suppose the cycle-type of a permutation σ is (e1 , e2 , . . . , eh ). For i = 1, 2, . . . n, let αi be
Partitions’ by G. Andrews. the number of times i occurs in (e1 , e2 , . . . , eh ). Note that, some of the αi s could be zero.
We can then write Eqn. (1) as

nαn + (n − 1)αn−1 + · · · + 2α2 + α1 = n

and we may write the partition (e1 , e2 , . . . , eh ) as 1α1 2α2 . . . nαn . We will drop the i’s for
which αi = 0 from the product. For example, we can write the partition
(4, 3, 2, 2, 1, 1, 1) of n = 14 as 13 · 22 · 33 · 4.
We know that |[σ ]| = [Sn : N(σ )]. We also know that |Sn | = n!, so we have to calculate
|N(σ )|, the order of the normaliser of σ , i.e.

|{τ ∈ Sn |τσ = σ τ }|

Now, observe that τ ∈ Sn will satisfy τσ = σ τ or equivalently τ −1 σ τ = σ if and only if


τ −1 σ τ and σ have the same cycle type, say 1α1 2α2 3α3 · · · kαk , and moreover the cycle
decomposition of τ −1 σ τ differs from that of σ only by a cyclic shift of the symbols
within a single cycle or an ordering of the αi cycles of length i for i = 1, . . . , k. There
are i cyclic shifts possible in a cycle of length i and the αi cycles of length i can be
permuted in αi ! ways. It follows that the number of τ ∈ Sn for which τ −1 σ τ = σ is
It is a standard convention given by ∏ki=1 iαi αi !.
that 0! = 1.
In other words,

|Z(σ )| = 1α1 × 2α2 × · · · × kαk × α1 !α2 ! · · · αk ! (2)

So, by the orbit-stabiliser relation, the number of elements in the conjugacy classes Cσ
corresponding to the partition

|k + k +
{z· · · + k} + (k
|
− 1) + (k − 1) + · · · + (k − 1) + · · · + 2| + 2 +
{z } {z· · · + 2} + 1| + 1 +
{z· · · + 1}
αk times αk−1 α2 times α1 times
(3)

is
n!
|Cσ | = k k
(4)

24 ∏ i ∏ αi !
αi
i=1 i=1
Let us now find the conjugacy classes in the group S4 . The partitions of 4 and the Sylow Theorems and The
Symmetric Group
number of elements in the conjugacy classes are given below:

Partition Number of elements in the


corresponding conjugacy class
1+1+1+1 1
4!
1+1+2 (12 ×21 )×(2!×1!)
=6
4!
1+3 (11 ×31 )×(1!×1!)
=8
4!
2+2 22 ×2!
=3
4!
4 41 ×1!
=6

So, the number of conjugacy class in S4 is five and the class equation is

24 = 1 + 3 + 6 + 6 + 8

The conjugacy classes of S4 are

[(1)], [(1 2)], [(1 2 3)], [(1 2)(3 4)], [(1 2 3 4)]

Let us now write down the class equation for A4 . Since A4 is a normal subgroup it is
the union of S4 -conjugacy classes of elements that contain an even permutation. Note
that a cycle of length k can be written as a product of k − 1 transpositions for k ≥ 3:

(a1 a2 . . . , ak ) = (a1 a2 ) (a1 a3 ) · · · (a1 ak )

You can use this to verify that the S4 -conjugacy classes in A4 that contain an even
permutation are

[(1)], [(1 2)(3 4)], [(1 2 3)] (5)

But, in general, the elements that are conjugates in Sn need not be conjugates in An . So,
some of the Sn -conjugacy classes may split into smaller conjugacy classes in An . We
leave it to you to check that the Sn -conjugacy class of an element σ ∈ An will split into
at most two An -conjugacy classes and this happens precisely when stabiliser of σ in Sn
contains only even permutations.(See Exercise 1.) So, the question arises: When will
the stabiliser of σ ∈ An in Sn will contain only even permutations?

Proposition 2: Let σ ∈ An . Then, Z(σ ) has only even permutations if and only if the
parts in the partition of n corresponding to σ are odd and distinct.

Proof: Necessity: Suppose the partition corresponding to σ has an even part. This
means that the decomposition of σ into disjoint cycles contains a cycle of even length,
say

σ = (1 2 . . . 2k)( )( ) · · · (6)

Then, g = (1 2 . . . 2k) is an odd permutation and it commutes with σ . This is because g


and all the cycles other than the first cycle g = (1 2 . . . 2k) that occur in the
decomposition in Eqn. (6) are disjoint and g commutes with itself. So, g ∈ Z(σ ).
Suppose the partition corresponding to n contains 2 identical parts of size k, say

σ = (1 2 . . . k)(k + 1 . . . 2k)( )( ) · · ·

Then, τ = (1 k + 1 2 k + 2 . . . k 2k) is an odd permutation and we claim that it


commutes with σ . To see this, note that τ merely interchanges the cycles (1 2 . . . k)
and (k k + 1 . . . 2k) and it leaves all the other cycles unchanged. Thus, τ ∈ Z(σ ).
Sufficiency: Suppose the partition of n corresponding to σ has odd and distinct parts.
This means that, in Eqn. (2), αi = 0 for even i and αi = 1 for odd i. So, 25
Study Guide-I |Z(σ )| = i1 i2 · · · im , where ij is odd for j = 1, 2, . . . , m. This means that all the elements
of Z(σ ) will have odd order and so the cycle decomposition of any element of Z(σ )
will have only cycles of odd length. But, a cycle of odd length is an even permutation.
So, all the cycles that occur in the cycle decomposition of any element of Z(σ ) are even
cycles and so the element itself is even. 

Let us now use Proposition 2 to identify the conjugacy classes in the list in Eqn. (6)
which will split into two when we consider the An -conjugacy classes. As you can see,
[(1 2 3)], which corresponds to the partition 4 = 3 + 1 will split into two conjugacy
classes in A4 . Since this Sn -conjugacy class has eight elements, it will split into two
conjugacy classes [(1 2 3)] and [(1 2 4)] of four elements each. So, the class equation of
A4 is

1+3+4+4

We can find the conjugacy classes of S5 by using the method we used for S4 .

Partitions Number of elements in the


corresponding conjugacy classes
1+1+1+1+1 1
1+1+1+2 10
1+2+2 15
1+1+3 20
2+3 20
1+4 30
5 24

So, the class equation of S5 is

120 = 1 + 10 + 15 + 20 + 20 + 30 + 24

The conjugacy classes are

[(1)], [(1 2)], [(1 2)(3 4)], [(1 2 3)], [(1 2 3)(4 5)], [(1 2 3 4)], [(1 2 3 4 5)]

We leave it to you to find the class equation and the conjugacy classes of A5 .
Here are some exercises for you to check your understanding of our discussion on
conjugacy classes of Sn and An .

EXERCISES E1) Let g ∈ An be an even permutation. Let C1 and C2 , respectively, denote the
conjugacy classes of g in Sn and An and let H1 denote the centraliser of g in Sn .
Using the orbit-stabiliser relationship show that |C1 | = |C2 | if H1 contains an odd
permutation and |C1 | = 2 |C2 | if H1 doesn’t contain an odd permutation.

E2) Find the elements in the two conjugacy classes of size four each in A4 .

E3) Write down the class equation and the conjugacy classes of A5 .

2.3.2 An is Simple

In this subsection, we will prove that An is simple. We first prove the following result.

26 Proposition 3: The alternating group An is generated by cycles of length 3.


Proof: We have seen that An consists of permutations that can be written as a product Sylow Theorems and The
Symmetric Group
of an even number of transpositions. We will pair them and show that product of any
two transpositions is a 3-cycle. Consider the case of two transposition having a
common element i. From the solution to Exercise 10 in page 233, it follows that the
product (i j)(i k) is simply the 3-cycle (i j k). In the case of disjoint transpositions (i j)
and (r s) we can insert (j r) and its inverse (which is itself) to get

(i j)(r s) = (i j)(j r)(j r)(r s) = (i j)(j r)(r j)(r s).

The last term in the above equation is (i j r)(r j s). This completes the proof. 

Theorem 1: The alternating group An is simple for all n ≥ 2, except in the case n = 4.

Proof: Recall that a group with no proper subgroups is called a simple group.
We know that A2 is the trivial group and A3 has order 3. So, both the groups are simple
by Lagrange’s theorem.
We have already seen that A4 has a normal subgroup of order four.
Now let us consider An for n ≥ 5.
Step I: If N is a normal subgroup of An containing a 3-cycle, then N = An .
Proof of Step I: Let (a1 a2 a3 ) be any three cycle in N. In view of Proposition 3, it is
enough to show that N contains every 3-cycle. Suppose (b1 b2 b3 ) is any 3-cycle.
Choose a4 , a5 (resp. b4 , b5 ) in {1, 2, 3, . . . n} distinct from a1 , a2 and a3 (resp. b1 , b2 , b3 )
and let g be a permutation of the form

a1 a2 a3 a4 a5 · · ·
 

b1 b2 b3 b4 b5 · · ·

Then, g−1 (a1 a2 a3 ) g = (a1 g a2 g a3 g) = (b1 b2 b3 ). If g is an even permutation, we are


done since N is a normal subgroup of An . Otherwise, consider the permutation
h = (b4 , b5 ). Since {b1 , b2 , b3 } ∩ {b4 , b5 } = 0,
/ h leaves b1 , b2 and b3 fixed. So,

(gh)−1 (a1 a2 a3 ) gh = (a1 gh a2 gh a3 gh) = (b1 h b2 h b3 h) = (b1 b2 b3 ) .

Also, gh is an even permutation.


Step II: We need to show that any nontrivial normal subgroup N contains a 3-cycle.
Proof of Step II: Let σ ∈ N be such that σ 6= 1 and σ has the maximum number of
fixed points. By a fixed point of σ we mean i ∈ {1, 2, . . . , n} such that iσ = i. We claim
that σ is a 3-cycle. Otherwise, suppose we write σ = c1 c2 · · · ck , where the ci are
disjoint cycles and the cycles are written in the descending order of their length. Then,
either c1 has length greater than 2 or all the ci are transpositions.
If the length of c1 is greater than 2, it has length at least 3. If it has length 3, there is at
least one more cycle in the decomposition since we have assumed that σ is not a
3-cycle. If c1 has length 4, since c1 is an odd permutation, and there is another cycle of
length at least 2. The other possibility is the c1 is a cycle of length at least five. In all
these cases, σ will move at least five elements, a1 , a2 , a3 , a4 and a5 . In this case, we can
write

σ = (a1 , a2 , a3 , . . .) · · · (7)

where the first cycle has length 3 or more.


Yet another possibility is c1 has length two and σ is a product of even number of
disjoint transpositions, say, σ = (a1 a2 ) (a3 a4 ) · · · . In this case we can write

σ = (a1 a2 ) (a3 a4 ) · · · (8) 27


Study Guide-I Consider τ = (a3 a4 a5 ) and let α = τ −1 σ τ. Then, α will be of the form
(a1 a2 a4 , . . .) · · · if σ is of the form given in Eqn. (7) on the previous page and it will be
of the form (a1 a2 ) (a4 a5 ) if σ is of the form given in Eqn. (8) on the preceding page. In
either case α 6= σ and ασ −1 6= 1. Let β = ασ −1 . Now, any i other than a3 , a4 or a5 will
be fixed by τ and if i is also fixed by σ , then it will be fixed by β = τ −1 σ τσ −1 .
If σ is as in Eqn. (7) on the previous page, a1 β = a1 and since σ moves a1 , a2 , a3 , a4
and a5 , β has more fixed points than σ .
If σ is as in Eqn. (8) on the preceding page, we a1 β = a1 and a2 β = a2 and so β has
more fixed points than σ in this case also.
This completes the proof of simplicity of An


We conclude this unit here. In the next section, we will summarise what we have learnt.

2.4 SUMMARY

In this Unit, we discussed the following:


1) How to apply Sylow theorems to prove the existence of groups of specified order.
2) The relationship between conjugacy classes in Sn and partitions of n and used it to
describe the conjugacy classes in Sn .
3) The conditions under which a conjugacy class in Sn of an even permutation splits
into two conjugacy classes in An .
5) How to calculate the conjugacy classes in S4 , A4 , S5 and A5 .

2.5 SOLUTIONS/ANSWERS

E1) Let H1 and H2 , respectively, denote the stabiliser of g under the conjugation action
of Sn and An , respectively. Then H2 = H1 ∩ An by definition. We have
|Sn | |An |
|C1 | = and |C2 | =
|H1 | |An ∩ H1 |
This can be re-written as
n!
|C1 | |H1 | = n! and |C2 | |H2 | = (9)
2
If |C1 | = |C2 |, then |H1 | = 2 |H2 |. So, H1 * An , i.e., H1 contains an odd
permutation.
Conversely, suppose H1 contains an odd permutation. In this case H1 An = Sn
(Why?). From the isomorphism
H1 H1 An
'
H1 ∩ An An
it follows that |H1 | = 2 |H2 | (How ?). Hence, from Eqn. (9), it follows that
|C1 | = |C2 | (How?).
E2) The elements in the conjugacy class [(1 2 3)] are (2 4, 3), (1 2 3), (1 3 4), (1 4 2).
The elements in the conjugacy class [(1 2 4)] are (2 3 4), (1 2 4), (1 3 2) and
28 (1 4 3).
E3) The conjugacy classes in S5 that contain only the even permutations are Sylow Theorems and The
Symmetric Group
[(1)], [(1 2)(3 4)], [(1 2 3)], [(1 2 3 4 5)]

The only conjugacy class that will split into two is [(1 2 3 4 5)] and it will split
into two conjugacy classes of 12 elements each. The class equation is

60 = 1 + 15 + 20 + 12 + 12
Solutions to selected exercises in ‘4. The Sylow Theorems.’ in page 231 and
232 of Artin’s book.
Q1) The number of subgroups of order 5 is of the form 1 + 5k and it divides 4. So, it
has to be 1. Therefore, there is a unique 5-Sylow subgroup. Since any element of
order 5 generates a group of order 5, all the elements of order 5 must be in this
unique subgroup of order 5. Therefore, the group has 4 elements of order 5.(Why?)
Q2) Let G be a group of order pq. Suppose that p < q without loss of generality. The
number of q- Sylow subgroups is of the form 1 + kq and divides p. Since q > p,
k = 0 and so the Sylow group of order q is normal.
Q3) Let G be a group of order p2 q. If q < p, the argument used in the previous problem
can be used to prove that the group order p2 is normal. So, let us assume that q > p.
Let nq be the number of q-Sylow subgroups of the G. Then nq is of the form
1 + kq, k ≥ 0 and nq | p2 . So, nq is 1, p or p2 . If nq = 1, we are done. But, nq
cannot be p because q > p, so nq = 1 + kq > p. So, nq = 1 + kq = p2 . So,
q | p1 − 1 = (p − 1)(p + 1). Since q - p − 1(Why?), q | p + 1. Then, p + 1 = tq > tp
or (t − 1)p < 1. So, t = 1 and q = p + 1. This is not possible if p > 2 and we are
done in this case. So, let us assume that p = 2 so that |G| = 12. The number of
2-Sylow subgroups of G is of the form 1 + 2k and divides 3. So, the only
possibilities are 1 and 3. If it is 3, G acts on the 2-Sylow subgroups by
conjugation. So, there is a homomorphism G −→ S3 . Since 12 - 6, the map is not
injective and so it has a non-trivial kernel which is a normal subgroup of G.
Q4) Check that the order of the group is 21 by considering the number of possibilities
for a and c and applying the multiplication principle from elementary
combinatorics.(See  the first
 unit in Block 2 of MTE-13.) Then, check that
1, 1 1 0
 
x= ,y= satisfy the conditions in (4.9)b, where we use i to denote
0, 1 0 4
the residue class of i (mod 7).
Q5) Let D10 = xi yj x2 = y10 = 1, xyx = y−1 and H = hxi × hy5 i where the product


is the internal direct product. (Why is hxi × hy5 i a group?) Then H is a 2-Sylow
subgroup of D10 . We leave it to you to find the conjugates of H.
1 a
  
Q6) Check that a ∈ Fp is a p-Sylow subgroup.
0 1
Q9) It is enough to prove that any group G of order pe has a subgroup of order pr for all
r ≤ e.(Why?) It is easy to prove this when G is cyclic. We leave it to you to fill in
the details. Let us therefore assume that G is not a cyclic group and apply induction
on e. If e = 1, the result is trivially true. Suppose it is true for all groups of order
pn with n < e. We have to prove that the result is true for groups of order pe .
Let us first show that G has a proper normal subgroup of order, say pk , for some
k < e. If G is abelian, then choose any element g 6= 1 in G and consider the group
N = hgi. Then, |N| = pk for k < e since we have assumed that G is not cyclic. If G
is not abelian, then Z(G) 6= G and Z(G) 6= {1} since G is a group of prime power
order. We have that Z(G) is a normal subgroup of G and it has order pk for some k.
So, in any case, G has normal subgroup N of order pk with k < e. By our induction
hypothesis, for any r ≤ k, N has a subgroup of order pr and therefore G also has a
subgroup of that order. Let e > r > k, and let r = k + v, say, where v > 1. Now,
G
consider the group N which is a group of order pm with m = e − k < e and so it has 29
Study Guide-I a subgroup of order pv . The inverse image of this subgroup under the canonical
G
map G → N will give a subgroup of order pk+v in G. The proof is now complete
by induction.
Solutions to selected exercises in ‘5. The Groups of Order 12.’ in page 232
of Artin’s book.

Q1) For all the abelian groups, the class equation is |1 + 1 + 1{z+ · · · + 1}. This leaves
12 times
us with three non abelian groups given in Theorem (5.1) in page 209.

Let us first consider the alternating group A4 . We have already seen in


Theorem(5.1) in page 209 of the book that if 3-Sylow subgroup is normal and
2-Sylow subgroup is not normal, the group will be one of the groups described in
iv) or v) of the Theorem, not the alternating group A4 . So, we can take for
granted the fact that the 4-Sylow subgroup H is normal but the 3-Sylow subgroup
K is not normal.
Since the 4-Sylow subgroup is normal, it is the union of certain conjugacy
classes of G. Further, since
H is abelian H ⊂ Z(g) for all g ∈ H. Therefore,
G
4||Z(g)| and |Cg | = Z(g) 3. The number of elements in a conjugacy class that is
contained in H is 1 or 3. The identity element is in a conjugacy class of one
element and there can be either three more conjugacy classes with one element
each or one conjugacy class with 3 elements. If there are three more conjugacy
classes with 1 element each, H will be in the centre of the group. Since G = HK
and H and K are abelian, G will be abelian. So, therefore H is the union of a
conjugacy class with one element and a conjugacy class with 3 elements.

Let K1 , K2 , K3 and K4 have the same meaning they have in the discussion of
case 2 in page 210 of Artin’s
 4 book.
If g 6= 1 is any element not in H, then it is in
one
 3 of the K
i s because ∪i=1 i \ {1} = 8 and the intersection of H with
K
∪i=1 Ki \ {1} is the empty set because H ∩ Ki = {1}. Let x ∈ K1 , x 6= 1. If g,
g1 in H are such that gxg−1 = g1 xg−1 −1
1 , since K1 = hxi, gg1 ∈ N(K1 ) = K1 , But,
H ∩ K1 = {1}, so g = g1 . So, the remaining 8 elements are in two conjugacy
classes of 4 elements each. Thus, the class equation of A4 is 1 + 3 + 4 + 4.
Let us now consider the dihedral group D6 of order 12. This is exercise 8 a) in
page 229. Here are some more details that may enable to solve the exercise in
case you haven’t solved it earlier. Let

D6 = xi yj x2 = y6 = 1, xyx = y−1


According to the discussion in last paragraph in page 210 of Artin’s book, this
corresponds to the case where H is the Klein four group. In this case the
subgroup of order 6, hyi, is normal and it is the union of conjugacy classes.
Further, since hyi ⊂ Z(g) for each g ∈ hyi, each conjugacy class of G contained
in hyi will have either one element or two elements. Since
xyx = xyx −1 = y−1 = y5 and xy4 x−1 = y−4 = y2 , there are two conjugacy

classes y, y , y2 , y4 . The remaining element y3 is in a conjugacy class of


 5 

single element and it is in the centre of the group.

You can check that the remaining six elements are in two conjugacy classes of
order three. Determine also the conjugacy classes.

Q2) First dispose off the case p = 2. Then, modify the proof of Corollary(4.4) in
page 205 of Artin’s book to prove the result.

Q4) a) The 11-Sylow subgroup is normal. Let us write H for the 11-Sylow
subgroup. Let K be a 5-Sylow subgroup of G. Let H = hxi and K = hyi.
30 Since H is normal, yxy−1 ∈ H. So, yxy−1 = xr , 1 ≤ r ≤ 10.
b) Let φy (x) = yxy−1 . Then φy is an automorphism of H and o φy = o(r)

Sylow Theorems and The
Symmetric Group
where r denotes the residue class of r (mod 11). Since o φy = o(y) = 5,
o(r) = 5. Since Z∗11 is a cyclic group of order 10, the set of elements in Z∗11
that satisfy g5 = 1 are precisely the squares in Z∗11 . So, the allowed values of
r are the set of squares in Z∗11 which is {1, 3, 4, 5, 9}.
1 1
 
c) Let H be the subgroup of GL2 (F11 ) generated by where 1 is the
0 1
residue class of 1 (mod 5). Let

1 0
  
K= c ∈ {1, 3, 4, 5, 9}
0 c

Check that
−1  c
1 0 1 1 1 0 1 c 1 1
    
= =
0 c−1 0 1 0 c−1 0 1 0 1

where c−1 is the inverse of c (mod 11).


 When
 c 6= 1, we get 4 of the
c 0
possible values for r. If we take y = , and K = hyi, HK will be an
0 c
abelian group and yxy−1 = x in this case. So, there are two different
isomorphism classes.

Solutions to selected exercises in ‘6. Computation in the Symmetric


Group.’ in page 232 and 233 of Artin’s book.

Q2) Suppose conjugating (1 2 3)(4 5) by σ will yield (2 4 1)(3 5). Then, comparing
the two permutations, 1σ = 2, 2σ = 4, 3σ = 1, 4σ = 3, 5σ = 5. So, σ is the
permutation

1 2 3 4 5
 
i.e., σ = (1 2 4 3).
2 4 1 3 5

Q4) a) The five cycle (1 2 3 4 5) has order five in S7 . The element (1 2 3 4 5)(6 7)
has order ten because (1 2 3 4 5) has order five and (6 7) has order two and
they commute. Suppose there is an element α which has order 15. Consider
the action of hαi on the set {1, 2, 3, 4, 5, 6, 7}. Every element in the set must
have stabiliser different from {1} because the set has only seven elements
while hαi has order 15. So, the stabiliser of any element has order 3, 5 or
15. Accordingly, the possible orders for the orbits are 1, 3 and 5. Since the
sum of the elements in the orbit add up to seven, the only possibilities are
1 + 3 + 3 and 1 + 1 + 5. In the first case, the stabilisers have order 15, 5 and
5. So, α 3 will belong to the stabilisers of all the elements and so it acts
trivially on the set {1, 2, 3, 4, 5, 6, 7}. So, α 3 = 1. This means that α has
order 5, a contradiction. We leave the other case to you.
b) Let σ be any element of Sn and let σ = c1 c2 . . . ck be its decomposition into
disjoint cycles of length greater than 1. If ai is length of the cycle ci , then
a1 + a2 + · · · + ak < n since each ci is of the form (i1 , i2 , i3 , . . . , ik ) with each
ij ∈ {1, 2, 3, . . . , n} and the cycles are disjoint. Since any two cycles ci and cj
commute, (c1 c2 . . . ck )m = cm m m
1 c2 . . . ck for any integer m. It follows that, if
m
m = lcm (a1 , a2 , · · · , ak ), (c1 c2 . . . ck ) = 1. So, the order of c1 c2 . . . ck
divides lcm (a1 , a2 , · · · , ak ).
Therefore, the maximum possible order of an element in Sn is

max { lcm (a1 , a2 , . . . , ak )| ai ≥ 2, a1 + a2 + · · · + ak ≤ n} 31


Study Guide-I Using the above considerations, let us list all a1 , a2 , . . ., ak , with ai ≥ 2 and
a1 + a2 + · · · + ak ≤ 7. Clearly k ≤ 3. The possibilities with k ≥ 2 are,
2, 2 2, 3 2, 4 2, 5 3, 3 3, 4 2, 2, 3
Apart from the elements arising from the above, there are cycles of length 2,
3, 4, . . .,7 which have order 2, 3, . . ., 7, respectively. So, the possible orders
of the elements in S5 are 1, 2, 3, 4, 5, 6, 7, 8, 10 and 12.

Q5) First, prove by induction that, according to Artin’s definition of sign,


sign ((a1 , a2 , . . . , an )) = (−1)n−1 , n≥2

To prove the result for n = 2, (a1 , a2 ) = q−1 (1, 2)q for some permutation q. So,

sign ((a1 , a2 )) = sign q−1 (1, 2)q = sign ((1, 2))




according to Artin’s definition. The permutation matrix for the permutation


(1, 2), considered as an element of Sn , is

0 1
 
1 0 .
In−2

where In−2 is the identity matrix of size n − 2. This matrix has determinant −1.
To complete the inductive proof use the relation

(a1 , a2 , . . . , an ) = (a1 , a2 , . . . , an−1 ) (a1 , an ) .

Finally, if p = c1 c2 · · · ck is the decomposition of p into disjoint cycles, it follows


that
sign(p) = sign (c1 ) sign (c2 ) . . . sign (ck ) = (−1)r

where the cycle ci has length ni and r = (n1 − 1) + (n2 − 1) + · · · + (nk − 1).

Q6) We have already proved in problem 4 that, if ai is the order of the cycle ci and
the the cycles are disjoint, the order of the permutation σ = c1 c2 · · · ck divides
lcm (a1 , a2 , . . . , ak ). It remains to show that lcm (a1 , a2 , . . . , ak ) divides the order of
the permutation σ . For this, it is enought to show that a1 , a2 , . . ., ak divide the
order of σ . Let us show that a1 , say, divides the order of σ . Suppose
c1 = (i1 , i2 , . . . , ia1 ). If we show that cm 1 acts as the identity permutation i1 , i2 , . . .,
ia1 , it will follow that a1 divides m. To show that cm 1 fixes i1 , i2 , ia1 , note that
σ m = cm 1 when restricted to the set {i ,
1 2 i , . . . , i a1 } because all the other cycles c2 ,
c3 ,. . .,ck act as identity permutation on the set {i1 , i2 , . . . , ia1 } since the cycles are
pairwise disjoint. Since σ m is identity, it follows that cm 1 is identiy on the set
{i1 , i2 , . . . , ia1 }. Also, c1 leaves fixed every j ∈ {1, 2, 3, . . . , n} \ {i1 , i2 , . . . , ia1 }
since the cycles are disjoint. Thus, cm 1 is the identity permutation and we are
done.

Q7) No, consider the 5-cycle (2 1 3 4 5). This 5-cycle is a conjugate of the 5-cycle
(1 2 3 4 5), but it is not a power of the 5-cycle (1 2 3 4 5).(Check this! Compare
how the powers of (1 2 3 4 5) and (2 1 3 4 5) act on 1 and 2.)

Q8) This is a standard result in combinatorics. A permutation that does not fix any
index fixed is called a derangement. One way of finding the number of
derangements is using the principle of inclusion and exclusion. See page 45 of
Block 2 of the IGNOU course MTE-13, Discrete Mathematics, for the
derivation. It is available online at http://www.egyankosh.ac.in/
32 bitstream/123456789/11595/1/Unit-6.pdf.
Q10) a) We first show that a cycle of length k can be written as a product of k − 1 Sylow Theorems and The
Symmetric Group
transpositions for k ≥ 3:

(a1 a2 , . . . , ak ) = (a1 , a2 ) (a1 , a3 ) · · · (a1 , ak )

The permutation on the LHS acts as follows: It sends a1 to a2 , a2 to a3 , . . .,


ak to a1 . We need to check that the permutation on the RHS also acts in the
same way. The first transposition (a1 , a2 ) sends a1 to a2 and a2 does not
occur in any of the remaining transpositions on the right. So, a1 goes to a2
under the permutation on the RHS. Similarly, ai , 1 ≤ i ≤ k − 1 occurs in the
transposition (a1 ai ) and ai is mapped to a1 under this permutation. The next
permutation (a1 , ai+1 ) maps ai to ai+1 and ai+1 is fixed by all the remaining
transpositions. The last element ak occurs only in the last transposition
(a1 , ak ) and this transposition maps ak to a1 .
Note that, from this, we can deduce that Sn is generated by the
transpositions (i, j) for i, j ∈ {1, 2, . . . , n}. We have seen that we can write
any element of Sn as a product of disjoint cycles. (See Proposition(6.6) on
page 213 of Artin’s book.) So, it follows that any permutation can be written
as a product of transpositions.
b) You can easily deduce this from what we have done in exercise 5. We leave
the details to you.

Q15) Let H be the subgroup of Sn generated by the transpositions (i, i + 1),


i = 1, 2, . . . , n − 1. If σ = (a1 , a2 , . . . , ak ) and τ are any two permutations, then
τ −1 σ τ = (a1 τ a2 τ · · · ak τ), following the notation in the book. Let (i j) be any
transposition with j ≥ i + 1. Let us show that (i j) is in H. Let j = i + k. Let us
apply induction on k. If k = 1, j = i + 1, there is nothing to prove since (i i + 1) is
in H. Suppose, it is true for k − 1, i.e. (i, i + k − 1) is in H. We have
(i + k − 1 i + k)−1 (i i + k − 1)(i + k − 1 i + k) = (i i + k). So, by induction,
(i i + k) is also in H, i.e., (i i + k) = (i j) is also in H.

Q16) Let us denote (1 2 . . . n) by τ. Note that


(
k+i if k + i ≤ n
kτ i =
k + i − n if k + i > n

The result now follows from the relation 


(1 2 . . . n)−i (1 2)(1 2 . . . n)i = 1τ i , 2τ i = (1 + i 2 + i), i = 1, 2, · · · , n − 2 and
the previous exercise.

33
Study Guide-I

34
UNIT 3 SPECIAL GROUPS
Structure Page No.
3.1 Introduction 35
Objectives
3.2 Definitions 35
3.3 The Special Unitary Group 36
3.4 The Special Linear Group 36
3.5 Summary 37
3.6 Solutions/Answers 37
Sections in Artin’s book related to this unit:

Chapter Section
8 1,2 and 4

3.1 INTRODUCTION

In this unit, we will discuss an important class of groups called classical groups. You
are already familiar with three of the classical groups, GLn (R) the general group of
n × n invertible matrices, the orthogonal group and the unitary groups. Recall that the
real(resp. complex) orthogonal group O(n)(R)(resp. O(C)) is the group of n × n
real(resp. complex) matrices P such that Pt P = I and the Unitary group Un is the group
of complex matrices P such that P∗ P = 1 where P∗ is the conjugate transpose of P. In
Sec. 3.2, we will discuss the definitions of various groups that we will study in this unit.
In Sec. 3.3, we will discuss the special unitary group, the subgroup formed by the
unitary matrices of determinant one. In Sec. 3.4, we discuss the special linear group,
the group of real matrices with determinant one.
Objectives
After studying this unit, you should be able to
• define the Symplectic, Orthogonal and Unitary groups;
• explain how to identify SU2 with the sphere S3 ;
• describe the conjugacy classes in SU2 in terms of latitudes of SU2 ;
• explain how the longitudes of SU2 are subgroups which are conjugate to each other;
• explain how to identify SL2 (R) with S1 × R.

3.2 DEFINITIONS

In this section, you will begin the study of the so called classical groups. You will learn
the definitions of the various groups, like the Orthogonal group, the Special Orthogonal
group and the Special Unitary group, that you will study in the later sections of the unit.

Read section 1 of Chapter 8 in Artin’s book. Attempt problems 1, 2, 3 and 11 under


BOOK
‘1. The Classical Linear Groups’ in page 300 and 301 of Artin’s book.

Here is an exercises for you to try.

E1) Show that, if P ∈ SP2n (R) or P ∈ Op,q (R) where p + q = n, det(P) = ±1. Deduce EXERCISES
that SP2n (R) is a subgroup of GL2n (R) and Op,q is a subgroup of GLn (R).

We conclude this section here. In the next section, we will discuss the special unitary
35
group.
Study Guide-I
3.3 THE SPECIAL UNITARY GROUP

Recall the standard hermtian form given by


n
(X, Y) = ∑ xi yi where X = (x1 , x2 , . . . , xn ) , Y = (y1 , . . . , yn ) ∈ Cn
i=1

The n × n matrices that preserve this form, namely n × n matrices U satisfying

(UX, UY) = (X, Y) for all X, Y ∈ C

are called unitary matrices.


In this section, we will discuss the special unitary group SU2 , the group of unitary
matrices with determinant 1. We will show that SU2 is homeomorphic to the 3-sphere
S3 .(See equation (2.7) in page 273 of the book.) We will also describe the conjugacy
classes in SU2 in terms of latitudes of the three sphere S3 .(See Proposition 2.14 on page
275.)

Read section 2 of Chapter 8 in Artin’s book. Attempt problems 1, 2, and 3 under ‘2.
BOOK
The Special Unitary Group’ in page 301 of Artin’s book.

3.4 THE SPECIAL LINEAR GROUP

The group of n × n real matrices with determinant one is called the Special Linear
Group. In this section, we will study the group of 2 × 2 real matrices with determinant
1. Note that, unlike the Special Unitary Group which is defined over the field of
complex numbers, the definition of the special linear group, matrices of determinant is
1, makes sense over any field. In particular, it is defined over the finite fields also.
However, we restrict ourselves to the real and complex fields in our course.
Read section 4 of chapter 8 in Artin’s book. Do problems 1 and 3, except for 3(c), in
BOOK
‘4.The Special Linear Group SL2 (R)’ on page 302.

NOTE Equation (4.1) in page 281.


Explanation: To see that the
 stabiliser
 H consists of matrices of the form given in this
a b
equation, consider any h = ∈ H. Then h ∈ SL2 (R) and h.e1 = re1 for some
c d
r ∈ R with r > 0. But,

a r a
     
h.e1 = and hence =
c 0 c

So a = r > 0 and c = 0. Since det(h) = ad − bc = 1, it follows that d = a−1 .


NOTE Line 11 from the bottom in page 281.‘Notice that H ∩ SO2 = {I}.’
a b
 
Explanation: We have seen already that the element h ∈ H are of the form .
0 a−1
If h is also in SO2 , then

a b a 0 1 0
    
t
hh = =
0 a−1 b a−1 0 1
that is,
 2
a + b2 ba−1 1 0
  
=
a−1 b a−2 0 1

36 Since a > 0, from a−1 b = 0, we get b = 0. From a−2 = 1, we get a = 1.


Continuity of f−1 defined in Proposition 4.2 on page 281. NOTE Special Groups
Explanation: The proof given in the book says that Q depends continuously on P
because the ray PR1 does. To understand this, we have to introduce a topology on the
set of rays in R2 . Since this proof is a little involved, we give an ad-hoc
 proof
 of the
a b
continuity of f−1 by describing Q explicitly in terms of P. Given P = consider
c d
√ a − √a2c+c2
!
2 2
the matrix Q = √a c+c √ a
. This matrix is in SO2 and
a2 +c2 a2 +c2

√ a
!
1 a 1
 
Qe1 = a2 +c2 =√ =√ Pe1 .
√ c a2 + c2 c a2 + c2
a2 +c2

Further,
√ √
√ a √ c a2 + c2 √ab+cd a2 + c2 √ab+cd
! ! !
a b

Q−1 P = a2 +c2 a2 +c2 = a2 +c2 = a2 +c2
− √a2c+c2 √ a
a2 +c2
c d 0 √ad−bc 0 √ 1
a2 +c2 a2 +c2

So, Q−1 P ∈ H.
Note that a2 + c2 6= 0 since
 ad − bc = 1. So, P Q and P Q−1 P are both continuous.
Thus, f−1 (P) = Q, Q−1 P is continuous.

3.5 SUMMARY

In this unit, we have discussed the following:

1. The definition of the Symplectic, Orthogonal and Unitary groups.

2. The identification of SU2 with the sphere S3 .

3. The description of the conjugacy classes in SU2 in terms of latitudes of S3 .

4. The description of longitudes of SU2 .

5. The identification of SL2 (R) with S1 × R.

3.6 SOLUTIONS/ANSWERS

Solutions to selected exercises in ‘1. The Classical Linear Groups.’ in page


300 and 301 of Artin’s book.

Q1) a) Check that


x y
 
x + iy, where x, y ∈ R, at least one of x or y is non-zero.
−y x

defines an isomorphism between GL2 (R) and C∗ .


b) Let zij = xij + iyij . Check that

x11 y11 ··· x1n y1n


 
z11 · · · z1n  −y11 x11 −y1n x1n
 

 
 .. .. ..   .. .. .. 
 . . .   . . . 
 
zn1 · · · znn  xn1 yn1 ··· xnn ynn 
−yn1 xn1 −ynn xnn
is an injective group homomorphism. 37
cos z − sin z
 
Study Guide-I
Q2) Note that matrices of the form are in SO2 (C) where z ∈ C.
sin z cos z
However, cos z → ∞ when z → ∞ along the imaginary axis. So, SO2 (C) is not
bounded.
Q3) Let
a b
 
A= ∈ GL2 (R).
c d
We have
a c 0 1 a b
   
t
A JA =
b d −1 0 c d
0 ad − bc
 
= . (1)
bc − ad 0
From Eqn. (1), it follows that A ∈ SL2 (R) if and only if A ∈ SP2 (R).(Why?)
2 5 0 0
 
1 3 0 0
Check that the matrix 0 0 1 0 is in SL4 (R), but not in SP4 (R).

0 0 0 1
Q11) Let
A B
 
P=
C D
be any 2n × 2n matrix where A, B, C and D are n × n matrices. We have
 t
A Ct

t
P = .
Bt Dt
 
0 −I
Let P be the matrix where I is the n × n identity matrix and 0 is the
I 0
0 I
 
n × n zero matrix. Then P = t . So,
−I 0
0 I 0 I
   
t 0 −I
P JP =
−I 0 −I 0 I 0
0 I
    
−I 0 0 −I
= =
0 −I I 0 −I 0
At A
! !
0 0
Let P = . Then, Pt = t . Therefore,
0 A−1 0 A−1
A
!  t
0 0 I A

t 0
P JP = t
0 A−1 −I 0 0 A−1
A
!
0 At 0 I
  
0
= t =
− A−1 0 0 A−1 −I 0
t t
since − A−1 At = − AA−1 = −I.
Let P be a 2n × 2n matrix of the form
I B
 
P=
0 I
I 0
 
where B is a n × n matrix with B = Bt . Then Pt = . We have
B I
I B 0 I I 0
   
t
P JP =
0 I −I 0 B I
38
Special Groups

−B I I 0 −B + Bt I
    
= = =J
−I 0 Bt I −I 0

since −B + Bt = 0.

Solutions to selected exercises in ‘2. The Special Unitary Group SU2 ’, in page 301
of Artin’s book.

a b c d
   
Q1) Let P = and Q = . Write
−b a −d c

a = x1 + ix2 , b = x3 + ix4 , c = y1 + iy2 and d = y3 + iy4

where x1 , x2 , x3 , x4 and y1 , y2 , y3 , y4 are real numbers satisfying

x21 + x22 + x23 + x24 = 1 and y21 + y22 + y23 + y24 = 1.

We have
ac − bd ad + bc e f
   
PQ = = (say)
−bc − ad −bd + ac −f e

We have

e = (ac − bd) = (x1 y1 − x2 y2 − x3 y3 − x4 y4 ) + i (x1 y2 + x2 y1 + x3 y4 − x4 y3 )


f = (ad + bc) = (x1 y3 + x3 y1 − x2 y4 + x4 y2 ) + i (x1 y4 + x4 y1 + x3 y2 − x3 y2 )

Now, PQ corresponds to the vector

(Re(e), Im(e), Re(f), Im(f))

You can check that

Re(e)2 + Im(e)2 + Re(f)2 + Im(f)2 = (x1 y1 − x2 y2 − x3 y3 − x4 y4 )2


+ (x1 y2 + x2 y1 + x3 y4 − x4 y3 )2 + (x1 y3 + x3 y1 − x2 y4 + x4 y2 )2
+ (x1 y4 + x4 y1 + x2 y3 − x3 y2 )2
= x21 + x22 + x23 + x24 y21 + y22 + y23 + y24 = 1
 

a b
 
Q2) We have to find an invertible matrix A = ∈ SU2 . Note that, since
−b a
a −b
 
2 2 −1
A ∈ SU2 , a, b has to satisfy |a| + |b| = 1. Also, A = .
b a

a b cos θ − sin θ a −b 0
     
λ1 (θ )
= ∀ θ ∈ R (2)
−b a sin θ cos θ b a 0 λ2 (θ )

where λ1 (θ ) and λ2 (θ ) depend on θ . Also, we should have ad − bc = 1. We can


rewrite Eqn. (2) as

a cos θ + b sin θ −a sin θ + b cos θ a −b 0


    
λ1 (θ )
=
−b cos θ + a sin θ b sin θ + a cos θ b a 0 λ2 (θ )

Since the off diagonal entries are 0,

(−b(a cos θ + b sin θ ) + a(−a sin θ + b cos θ )) = 0


a(−b cos θ + a sin θ ) + b(b sin θ + a cos θ ) = 0

39
Study Guide-I Simplifying, we get
−b2 sin θ − a2 sin θ = 0
2
a2 sin θ + b sin θ = 0
If we choose a and b so that a2 + b2 = 0 and |a|2 + |b|2 = 1, Eqn. (2) will be true.
From a2 + b2 = 0, we have a = ±ib. Let us take a = ib. Using this in
|a|2 + |b|2 = 1, we get 2|a|2 = 1, so we can try a = √i2 , b = √12 . Check that if
i 1
!
√ √
A= 2 2 then
− √12 − √i2
 −iθ
cos θ − sin θ e 0
  
−1
A A =
sin θ cos θ 0 eiθ

Solutions to selected exercises in ‘4. The Special Linear Group SL2 (R)’, in page
302 of Artin’s book.
a b
 
Q1) Let us first determine the stabiliser H. Let h = ∈ SL2 (C) be in H. By
c d
definition, we have
a b 1 r
    
= where r > 0
c d 0 0
But,
a b 1 a
    
=
c d 0 c
So, a = r > 0 and c = 0. Since det(h) = 1, we obtain
r b
 
h=
0 1r
a b
 
since det(h) = 1. Now, if P = ∈ SL2 (C), we set
c d
√ 2a 2 √ −c
 
|a| +|c| |a|2 +|c|2 
Q= . Check that Q ∈ SU2 (C) and Q−1 P ∈ H. Check
√ 2 2 √ 2a 2
c
|a| +|c| |a| +|c|
that f : SL2 (C) −→ SU2 (C) × H given by P Q, Q−1 P and f−1 , given by


(Q, h) Qh are continuous .


Q3) a) We have
det(P − xI) = det (P − xI)t = det Pt − xI
 

= det P−1 − xI ∵ Pt = P−1 for P ∈ SO3 (C)




So, P and P−1 have the same characteristic polynomials. In general, if λ is


an eigenvalue of P, λ −1 is an eigenvalue of P−1 . So, since P and P−1 have
the same characteristic polynomials, whenever λ is an eigenvalue of P, λ −1
is also an eigenvalue of P. Further, λ = λ −1 if and only if λ = ±1. Now, we
know that det(P) = 1, det(P) is the product of the roots of the characteristic
polynomial of P taken with correct multiplicity and the degree of the
characteristic polynomial of P is odd. Use these facts and show that 1 is an
eigenvalue of P.
b) Let X1 and X2 be an eigenvectors of P corresponding to eigenvalues λ1 and
λ2 , respectively. Then, Xt1 Pt X2 = (PX1 )t X2 = λ1 Xt1 X2 . But,
Xt1 Pt X2 = Xt1 P−1 X2 = λ2−1 Xt1 X2
or λ1 − λ2−1 Xt1 X2 = 0. The result follows.

40
UNIT 4 FREE GROUPS
Structure Page No.
4.1 Introduction 41
Objectives
4.2 Free Groups 41
4.3 Generators and Relations 41
4.4 Structure Theorem for Finitely Generated Abelian Groups 42
4.5 Summary 51
4.6 Solutions/Answers 51
Sections in Artin’s book related to this unit:

Chapter Section
6 7 and 8
12 4

4.1 INTRODUCTION

We have seen that the dihedral group Dn , the group of symmetries of a regular polygon
with n sides, isomorphic to the group generated by two elements x and y which satisfy
the relations yxy = x−1 , xn = 1 and y2 = 1. In this Unit, we will discuss the formal
background behind such representations. In Sec. 4.2, we introduce the concept of free
groups. In the next section, Sec. 4.3, we will see that any group can be realised as the
quotient of a free group. In Sec. 4.4, we will prove the structure theorem for finitely
generated abelian groups which says that we can, in a unique way (upto isomorphism),
write a finitely generated abelian group as a direct sum of cyclic groups.
Objectives
After studying this unit, you should be able to
• define a free group;
• explain the description of groups by generators and relations;
• explain the mapping property of the free groups;
• state and apply the structure theorem for finitely generated abelian groups.

4.2 FREE GROUPS

Read Section 6.7 in Artin’s book BOOK

Attempt Exercises 1 and 3 on Page 233 under ‘7. The Free Group.’ EXERCISES

4.3 GENERATORS AND RELATIONS

Read Section 6.8 in Artin’s book BOOK

Attempt Exercises 1, 7, 8 and 9 on page 234 under ‘8. Generators and Relations’. EXERCISES

41
Study Guide-I
4.4 STRUCTURE THEOREM FOR FINITELY GENERATED
ABELIAN GROUPS

You are already familiar with the concept of basis of a vector space. If V is a
n-dimensional vector space with a basis {e1 , e2 , . . . , en }, then we can write any element
x of V in the form x = ∑ni=1 xi ei for some scalars x1 , x2 , . . ., xn . In this section, we are
going to consider this notion in the setting of an abelian group.
Since we will be discussing only abelian groups in rest of this unit, we will use + for
the group operation.

Definition 4: An abelian group G is finitely generated if there is a finite subset


e1 , e2 , . . . , en in G such that we can write every element x ∈ G as x = ∑ni=1 xi ei .

Superficially, it would seem that the analogue of finite dimensionality for a vector space
is the finite generation of abelian groups. We know that every finite dimensional vector
space over any field has a basis consisting of finitely many elements. But, as we will
see later, not every finitely generated abelian group has a basis. Let us now formally
define the notion of a basis for an abelian group. It is along the expected lines.

Definition 5: Let G be a finitely generated agelian group. By a basis of G, we mean a


finite subset {e1 , e2 , . . . , en } of G with the property that every x ∈ G can be uniquely
written as x = ∑ni=1 xi ei where x1 , x2 , . . . , xn ∈ G. We say that G is a (finitely generated)
free abelian group if it has a (finite) basis.

Let us look at an example of a free abelian group.


Example 1: Consider Zn = Z {z· · · × Z}. If we let
| ×Z×
n times

ei = (0, 0, . . . , 1, 0, . . . , 0)

where the ith coordinate is 1, then Zn is a free abelian group with basis e1 , e2 , . . ., en .
∗∗∗

EXERCISES E1) Show that

Pn = a0 + a1 X + a2 X2 + · · · + an Xn ∈ Z[X] a0 , a1 , . . . , an ∈ Z


is a finitely generated free abelian group.

E2) Let G be a finitely generated, free abelian group. Hom(G, Z) be the set of all
group homomorphisms from G to Z. Then, we can give the structure of an abelian
group on Hom(G, Z) by defining (φ + ψ)(g) = φ (g) + ψ(g) for g ∈ G. Show that
Hom(G, Z) is a free abelian group of rank n under this operation.

E3) Let G be a finitely generated abelian group and H be a subgroup of G.


a) Is GH a finitely generated abelian group? Justify your answer.
G
b) If we assume that G is also free, is H free? Justify your answer.

Once we fix a basis S = {e1 , e2 , . . . , en } in a free abelian group G, we can talk of the
coordinate vectors of an element x with respect to the basis S. If x = ∑ni=1 xi ei , then we
call (x1 , x2 , . . . , xn ) the coordinate vector of G with respect to S. Also, the map

x (x1 , x2 , . . . , xn )

42 is a group isomorphism from G to Zn . We leave this to you as an exercise.


Just as in vector spaces, if G1 and G2 are two finitely generated free abelian groups, we Free Groups
can represent any homomorphism φ : G1 −→ G2 through a matrix by fixing ordered
bases S1 = {e1 , e2 , . . . , en } in G1 and S2 = {f1 , f2 , . . . , fm } in G2 as follows: Suppose
n
φ (ei ) = ∑ aij fj where aij ∈ Z.
j=1

If x = ∑ni=1 xi ei , then
! !
n n n m n m
φ (x) = ∑ xi φ (ei ) = ∑ xi ∑ aij fj =∑ ∑ aij xi fj = ∑ yj fj ,
i=1 i=1 j=1 j=1 i=1 j=1

where
y1 a11 a12 a1n x1
    
...
 y2   a21 a22 ... a2n  x2 
 
 ..  =  ..
   
.. ..   .. 
 .   . .
... .   .
ym am1 am2 . . . amn xn

Thus, the homomorphism φ can be represented by the matrix

a11 a12 . . . a1n


 
 a21 a22 . . . a2n 
..  ;
 
 .. ..
 . . ... . 
am1 am2 . . . amn

This is called the matrix of φ with respect to (ordered) bases S1 and S2 .


Also, suppose G1 , G2 and G3 are finitely generated free abelian groups,
 and
φ : G1 −→ G2 , ψ : G2 −→ G3 aregroup homomorphisms and if aij is the matrix of φ
with respect
 to S1 and S2 and bij is the matrix of ψ with respect to S2 and S3 , then
bij · aij is the matrix of ψ ◦ φ with respect to the bases S1 and S3 .
We know that, in finite dimensional vector spaces, any two bases have the same number
of elements and this number is called dimension of the space. Is this true for finitely
generated abelian groups? The answer is ‘yes’.

Proposition 1: Let G be a finitely generated free abelian group. Then, any two bases
have the same number of elements.

Proof: Suppose S1 = {e1 , e2 , . . . , en } and S2 = {f1 , f2 , . . . , fm } are two different ordered


bases of G. Write fi = ∑nj=1 aij ej and ei = ∑m j=1 bij fm for some aij , bij ∈ Z. We have
!
m m n n m
ei = ∑ bij fj = ∑ bij ∑ ajk ek = ∑ ∑ bij ajk ek
j=1 i=1 k=1 k=1 j=1

But,
n
ei = 0.e1 + 0.e2 + · · · + 0.ei−1 + 1.ei + 0.ei+1 + · · · + 0.en = ∑ δik ek ,
k=1

where δik is the Kronecker delta given by


(
1 if i = k,
δik =
0 otherwise.

By the uniqueness of representation with respect to a basis,


m
∑ bij ajk = δik 43
j=1
If we write A = aij and B = bij , the last equation means that BA = I, where I is the
 
Study Guide-I
n × n identity matrix. Note that A is a n × m matrix and B is a m × n matrix.
Suppose n 6= m, say n > m. Consider the matrices A0 = [A|0n×(n−m) ] and
A
 
0
B = . Then A0 and B0 are n × n matrices and B0 A0 = BA = I where I is the
0(n−m)×n
n × n identity matrix. We have det (A0 B0 ) = det(I) = 1, but det (A0 ) = 0 and
det (B0 ) = 0, which is a contradiction. This proves that n = m. 

We can now define the rank of a finitely generated free abelian group, which is the
analogue of dimension of a vector space.

Definition 6: Let G be a finitely generated free abelian group. Then, the rank of G is
the number of elements in a basis of G. By convention, we will say that the trivial
group has rank zero.

Remark 1: Let G be a free abelian group and suppose S = {e1 , e2 , . . . , en } is a subset of


G. To show that S forms a basis for G, we only have to show that S generates G over Z
and that, if ∑ni=1 ai ei = 0, with ai ∈ Z, then each ai = 0. This is enough to show that we
can represent any element uniquely as a linear combination of {e1 , e2 , e3 , . . . , en }. For, if
∑ni=1 ai ei and ∑ni=1 bi ei are two representations of the same element, ∑ni=1 ai ei = ∑ni=1 bi ei
or ∑ni=1 (ai − bi ) ei = 0. Therefore, ai − bi = 0 for all 1 ≤ i ≤ n, i.e. ai = bi for 1 ≤ i ≤ n.

We will prove the structure theorem for finitely generated abelian groups. This says that
any finitely generated abelian group can be written as a direct product of cyclic groups.
The precise statement of the theorem is as follows:

Theorem 2(Structure theorem for finitely generated abelian groups): Let G be a


finitely generated abelian group. Then

G ' Zd1 × Zd2 × · · · × Zdn × Zr (1)

where r, d1 , d2 , . . . , dk are integers with r ≥ 0, di ≥ 1 and di | di+1 for 1 ≤ i ≤ n − 1.


Further, the di and r are uniquely determined by G. In other words, if
0
G ' Zd01 × Zd02 × · · · × Zd0n × Zr (2)

with d0i ≥ 1, and d0i | d0i+1 for 1 ≤ i ≤ n − 1, then r = r0 and d0i = di .

Remark 2: Note that, in the direct product decompositions in Theorem 2, we have


taken the number of terms of the form Zs to be the same for both the decompositions.
We can do this because we allow s = 1. Since Z1 = {0}, we can add as many terms of
the form Z1 to whichever decomposition we want to make the number of terms of the
form Zs on both the decompositions to be equal n. Also, once we prove Theorem 2
with di ≥ 1, it follows that the theorem is also true with di > 1. Why? This is because,
from the conclusion that the number of di and d0i are the same and di = d0i for 1 ≤ i ≤ n,
it follows that the number of di s which are one and the number of d0i s which are one are
the same. So after dropping the Zdi , Zd0i with di = d0i = 1, we still have the same number
of terms left in both the decompositions and the conclusion holds with dk > 1. The
factors d1 , d2 , . . . , dk that we get, with di ≥ 1 for 1 ≤ i ≤ k, are called invariant factors.

The plan of the proof is as follows: We first prove that any finitely generated abelian
group is isomorphic to a quotient FF21 , where F1 and F2 are free abelian groups. We then
show that we can choose a basis {w1 , w2 , . . . , wm } for F1 and a basis {u1 , u2 , . . . , un } for
F2 with n ≤ m such that ui = di wi for 1 ≤ i ≤ n. From this, we deduce Eqn. (1). We
then prove the uniqueness of representation.
Before we can discuss the proof of the result, we have to prove some elementary results
44 related to free abelian groups.
Proposition 2: Let G be a free abelian group of finite rank n, and H be a subgroup of G. Free Groups
Then, H is a free abelian group of rank at most n.

Proof: If n = 0, both G and H are trivial, and so they have rank 0. Therefore, the result
is true in this case.
If the rank of G is 1, G is an infinite cyclic group. In fact, if G = hvi is any cyclic group
and H is a subgroup, H = hdvi for some d ∈ N. We leave this to you as an exercise.
Since H = hdvi, H is also a free abelian group of rank at most 1. It has rank 0 if d is
zero and rank 1 if d 6= 0. This proves the result for n = 1.
Now, assume that the result is true for all groups of rank ≤ n − 1. Suppose, n, the rank
of G, is greater than one and let H be a subgroup of G. Let {e1 , e2 , . . . , en } be a basis of
G over Z. Let K be the free abelian subgroup of G generated by {e1 , e2 , . . . , en−1 }. Let
us denote the image of x ∈ G under the natural map
G
φ : G −→ ,x x+H
H
by x. We claim that K has rank n − 1 and G K is a free abelian group of rank 1 generated
by en = en + K. Let us see why this is so.
Let x ∈ K G
, where x ∈ G. Then, x = ∑ni=1 ai ei and so x = an en + K since e1 , e2 ,
. . . , en−1 ∈ K. If an en = 0, an en ∈ K and we can write an en as a linear combination of e1 ,
e2 , . . . en−1 , i.e. ∑ni=1 bi ei = 0 with bn = an . Since we can represent zero uniquely as
∑ni=1 0ei , this implies that all the bi s are zero. In particular, an = 0.
Let H be the image of H under the natural homomorphism φ : G −→ G H . If H = {0},
then H ⊂ K and by induction, H is a free abelian group of rank at most n − 1.
If H 6= {0} then H is a free abelian group of rank 1, generated by den for some d ∈ N.
By induction hypothesis, H ∩ K is a free group of rank at most n − 1. Let {f1 , f2 , . . . , fm }
be a basis for H ∩ K over Z, where m ≤ n − 1. Then, we claim that {f1 , f2 , . . . , fm , fm+1 }
is a basis for H over Z, where we write fm+1 = den . Let us see why this is true.
Let us consider an element x ∈ H and let x be its image in G K . Since fm+1 generates H,
x = afm+1 , a ∈ Z. Since x − afm+1 ∈ K and both x and fm+1 are in H,
x − afm+1 ∈ H ∩ K. Since {f1 , f2 , . . . , fm } is a basis for H ∩ K, x − afm+1 = ∑m i=1 ai fi or
x = ∑mi=1 a i fi + afm+1 . This proves that the set {f 1 , f 2 , . . . , f m , f m+1 } generates H over Z.
To prove that the set {f1 , f2 , . . . , fm , fm+1 } forms a basis, we have to show that, if
∑m+1 m+1
i=1 ai fi = 0, ai ∈ Z, then ai = 0, i = 1, 2, . . . , m + 1. If ∑i=1 ai fi = 0, we have
m
am+1 fm+1 = − ∑i=1 ai fi ∈ K since fi ∈ K. So, am+1 fm+1 = 0. Since fm+1 is a basis for H,
am+1 = 0. So, ∑m i=1 ai fi = 0. Since {f1 , f2 , . . . , fm } is a basis for H ∩ K over Z, it follows
that ai = 0 for 1 ≤ i ≤ m. Hence H is a free abelian group with basis {f1 , f2 , . . . , fm+1 }
where m + 1 ≤ n − 1 + 1 = n since m ≤ n − 1. This completes the proof. 
Remark 3: The result in Proposition 2 is not true for non-abelian groups. A free
non-abelian group of rank n can have a subgroup of rank greater than n. For example, if
G is the free group on two generators, the subgroup H generated by u = x2 , v = y2 ,
w = xy is isomorphic to the free group on three generators.

Corollary 2: Let G be a finitely generated abelian group.


F1
a) There are finitely genertated free abelian groups F1 and F2 such that G ' F2 .
b) If H is a subgroup of G, then H is also finitely generated.

Proof:
a) Since G is finitely generated, there are elements x1 , x2 , . . ., xn which generate G.
Let F1 be a free group of rank n and let {e1 , e2 , . . . , en } be a basis of F1 . Let
φ : F1 −→ G be the group homomorphism given by ei xi . This map is onto since
xi generate G.(Why?) Let us call the kernel of this map F2 . Then F2 is a free
abelian group of rank ≤ n by Proposition 2 and FF12 ' G 45
Study Guide-I H
b) There is a subgroup H of F1 such that F2 ⊆ H ⊆ F1 and with ' H under the
F2
natural isomorphism φ : FF12 −→ G induced by φ . Since F1 is a free abelian group
of rank n, H is a free abelian group of rank at most n. The quotient of a finitely
generated abelian group is finitely generated.(See exercise 3 a).) So, FH2 is finitely
generated and therefore H is also finitely generated.

BOOK Read section 4 of chapter 12.
In the second paragraph of page 462 of the book, Artin mentions a final more serious
point in the proof of Theorem 4.11, namely, ensuring that S has a finite set of
generators. Proposition 2 on page 45 takes care of this since S is the subgroup of a
finitely generated abelian group W. So, no gap remains in the proof of Theorem 4.11.
Suppose G is a finitely generated abelian group. Then, as in Proposition 2 on the
previous page, we can get free abelian groups F1 and F2 such that F2 ⊂ F1 , a map
φ : F1 −→ G with kernel F2 . Let us now apply Theorem 4.11 with W = F1 and S = F2 .
We get a basis {w1 , w2 , . . . , wm } of F1 and a basis {u1 , u2 , . . . , un } of F2 such that
ui = di wi , di ≥ 1 for 1 ≤ i ≤ n and di | di+1 for 1 ≤ i ≤ n − 1.
Let us now prove that G is of the form given in Eqn. (1) on page 44. Let r = m − n. Let
d1 , d2 , . . .,dn be as in the statement of Theorem 4.11. We define a map
ψ : Zd1 × Zd2 × · · · × Zdn × Zr −→ FF12 ' G by
n r
(a1 , . . . , an , b1 , b2 , . . . , br ) ∑ ai wi + ∑ bi wn+i (3)
i=1 i=1

where wi = wi + F2 and ai denotes the residue class of ai mod di .


Let us check that ψ is well defined.  Suppose 
(a1 , . . . , an , b1 , b2 , . . . , br ) = a1 , . . . , a0n , b01 , b02 , . . . , b0r . Then, bi = b0i and ai + yi di = a0i .
0

So,
  n r
ψ a01 , . . . , a0n , b01 , b02 , . . . , b0r = ∑ a0i wi + ∑ b0i wn+i
i=1 i=1
n r
= ∑ a0i wi + ∑ bi wn+i (∵ bi = b0i )
i=1 i=1
n n r
= ∑ ai wi + ∑ yi di wi + ∑ bi wn+i
i=1 i=1 i=1
n r
= ∑ ai wi + ∑ bi wn+i (∵ di wi = 0).
i=1 i=1

Let us now check that ψ is injective. Suppose


n r
ψ (a1 , . . . , an , b1 , b2 , . . . , br ) = ∑ ai wi + ∑ bi wn+i = 0.
i=1 i=1

Then, we have ∑ni=1 ai wi + ∑ri=1 bi wn+i ∈ F2 , so


n r n
∑ ai wi + ∑ bi wn+i = ∑ di yi wi .
i=1 i=1 i=1

Since {w1 , w2 , . . . , wm } is a basis for F1 , it follows that ai = yi di for 1 ≤ i ≤ n and


bi = 0 for 1 ≤ i ≤ r. So, (a1 , . . . , an , b1 , b2 , . . . , br ) = (0, 0, . . . , 0) and ψ is injective.
It is easy to see that ψ is surjective. Given any element ∑ni=1 ai wi + ∑ri=1 bi wn+i in G,
we have
n r
ψ (a1 , . . . , an , b1 , b2 , . . . , br ) = ∑ ai wi + ∑ bi wn+i .
46 i=1 i=1
This completes the proof of the isomorphism. Free Groups

Let us now show that the decomposition is unique. Let us write


0
G0 = Zd1 × Zd2 × · · · × Zdn × Zr and G00 = Zd01 × Zd02 × · · · × Zd0n × Zr .

Suppose G ' G0 and G ' G00 too. Then, we have to show that r = r0 and di = d0i for
1 ≤ i ≤ n. To prove this, we need the notion of torsion group of an abelian group.

Definition 7: Let G be an abelian group. Then, the torsion group of G, written as Gtor
is the subgroup

{ g ∈ G| dg = 0 for some d ∈ N}

Our strategy for the proof of uniqueness is as follows: Since G0 ' G00 , we have
G0tors ' G00tors . (Check this.) We then prove that G0tors = Zd1 × Zd2 × · · · × Zdn . Similarly,
G00tors = Zd01 × Zd02 × · · · × Zd0n . From this, we deduce that d0i = di .
For any abelian group G and d ∈ N, let us write dG = {dg | g ∈ G}. We leave it to you
to check that, if f : G1 → G2 is a group isomorphism, f(dG1 ) = df(G1 ) = dG2 . We
prove that dn G0 is a free abelian group of rank r and dn G00 is a free abelian group of rank
r0 . Since dn G0 ' dn G ' dn G00 , r = r0 and we are done. Let us now carry out our strategy.
Let g ∈ G0 and suppose that g = (a1 , a2 , . . . , an , b1 , . . . , br ) where ai ∈ Zdi for 1 ≤ i ≤ n
and bi ∈ Z for 1 ≤ i ≤ r. If g ∈ G0tors , there is d ∈ N such that

dg = (da1 , da2 , . . . , dan , db1 , db2 , . . . , dbr ) = (0, 0, . . . , 0).

Then, dbi = 0 for 1 ≤ i ≤ r, so bi = 0 for 1 ≤ i ≤ r since d ∈ N. This shows that


 
  
0
Gtors = a1 , a2 , . . . , an , 0, . . . , 0 ai ∈ Zdi ' Zd1 × Zd2 × · · · × Zdn .


 | {z } 
r times

We can similarly prove that

G00tors ' Zd01 × Zd02 × · · · × Zd0n

In the next lemma we prove that di = d0i for 1 ≤ i ≤ n.

Lemma 2: Suppose

G = Zd1 × Zd2 × · · · × Zdn and H = Zd01 × Zd02 × · · · × Zd0n

where G ' H, di , d0i ≥ 1 for 1 ≤ i ≤ n. If di | di+1 and d0i | d0i+1 for 1 ≤ i ≤ n. Then
di = d0i for 1 ≤ i ≤ n.

Proof: We have | G |= ∏ni=1 di =| H |= ∏ni=1 d0i . We will prove the result by induction
on the order of G. If | G |= 1, all the di s, d0i s are 1 and we are done. Suppose the order
of G is greater than 1. It follows that there is a prime p such that p divides |G|. Then,
there is a t such that p divides dt+1 , . . . , dn but p does not divide d1 , d2 , . . . , dt and p
divides d0s+1 , . . . , d0n but p does not divide d01 , d02 , . . . , d0s . Here t ≤ n and s ≤ n. We have
pG ' pH. Also, check that
(
Zd if p - d
pZd =
Zd/p if p | d

So,

pG ' pZd1 × pZd2 × · · · × pZdn ' Zd1 × Zd2 × Zdt × Zdt+1 /p × · · · × Zdn /p . 47
Study Guide-I Similarly,

pH ' Zd01 × Zd02 × Zd0s × Zd0s+1/p × · · · × Zd0n /p .

Therefore, we have | pG |=| G | /pn−t+1 and | pH |=| H | /pn−s+1 . So, t = s. By


d0
induction, it follows that di = d0i for 1 ≤ i ≤ t and pi = dpi for t + 1 ≤ i ≤ n. The result
now follows. 

To complete the proof, we have to show that dn G0 is a free abelian group of rank r. For
this, consider any element g = (a1 , a2 , . . . , an , b1 , . . . , br ) where ai ∈ Zdi for 1 ≤ i ≤ n and
bi ∈ Z for 1 ≤ i ≤ r. Then,

dn g = (dn a1 , dn a2 , . . . , dn an , dn b1 , . . . , dn br ) = (0, 0, . . . , 0, dn b1 , . . . , dn br )

because di | dn and di ai = 0. So,


 
 
dn G0 = 0, 0, . . . , 0, dn b1 , . . . , dn br bi ∈ Z

 | {z } 
n times

This means that dG0 is the free abelian group generated by f1 , f2 , . . .,fr where fi is the
n + r tuple having dn as the (n + i)th component and all the other components are 0.
There is another way of decomposing a finitely generated abelian group into a product
of cyclic groups. We now state the result.

Theorem 3: Let G be a finitely generated abelian group. Then,

G ' Zpn1 × Zpn2 × Zpns s × Zr (4)


1 2

where the p1 , p2 , . . .,ps are primes, not necessarily distinct. The value of r in Eqn. (4) is
unique and the powers pn11 , pn22 , . . .,pns s are uniquely determined up to the order of the
factors.

To prove this theorem, we require the following lemma:

Lemma 3: Let C be a finite cyclic group. Then, we can write C as a product of cyclic
groups of prime power order.

Proof: We will prove the result by induction on the order of C. Suppose |C| = n. If
n = 1, there is nothing to prove. Let n > 1. If n is already a power there is nothing to
prove. Suppose n is not a prime power. Suppose n = mpk where (m, p) = 1. Then,
C ' Cpk × Cm where Cpk is a cyclic group of order pk and Cm is a cyclic group of order
m. We leave the proof of this fact as an exercise to you.
Since m < n, by induction hypothesis, we can write Cm as a product of cyclic groups of
prime power order. The result now follows. 

Proof of Theorem 3: We can easily prove Theorem 3 now. We apply Lemma 3 to each
of the groups Zd1 , Zd2 , . . .,Zdk in Eqn. (4) to complete the proof of Theorem 3. Since
we can write each of these groups as a product of cyclic groups of prime power order,
we can write the product of these groups as a product of groups of prime power order.
Let us prove that decomposition in Eqn. (4) is unique. Suppose G ' G0 and G ' G00
where
0
G0 ' Zpn1 × Zpn2 × Zpns s × Zr and G00 ' G ' Zpm1 × Zpm2 × Zpms s × Zr . (5)
1 2 1 2

In the equation, Eqn. (5), p1 , p2 , . . . , ps are primes, not necessarily distinct. By allowing
48 some of the mi s and ni s to be zero, we can assume that the number of cyclic groups of
prime power order are the same and the same set of primes occur on both sides because Free Groups
we can add as many factors with ni = 0 or mi = 0 as necessary.
The proof that r = r0 is similar to the proof we gave for Theorem 2. We take
d = ∏si=1 p`i i where `i = max{ni , mi }. Then, as before, dG0 ' dG ' dG00 and we can
show that dG0 and dG00 are free abelian groups of rank r and r0 , respectively. The result
now follows as before.
Let us now prove the uniqueness of the orders of the cyclic groups of prime power
order that occur in Eqn. (4).
Let
pe111 pe112 ... pe11n
pe221 pe222 ... pe22n
.. .. .. .. (6)
. . . .
pekk1 pekk2 . . . pekkn
be the prime powers that occur in the decomposition of G0 . We are abusing the notation
a little here and using the notation p1 , p2 , . . .,pk for primes that are distinct. Also, we
assume that the powers of the primes along the rows are in ascending order, i.e.
ei1 ≤ ei2 ≤ · · · ≤ ei n−1 ≤ ein for i = 1, 2, . . . , k. We take some of the eij s to be zero, if
necessary, so that all the rows have the same number of elements.
For example, if G0 ' Z2 × Z4 × Z3 × Z5 , we would arrange the prime powers as follows:
2 22
30 3
50 5

Similarly, let
pf111 pf112 ... pf11n
pf221 pf222 ... pf22n
.. .. .. .. (7)
. . . .
pfkk1 pfkk2 . . . pfkkn
be the prime powers that occur in the decomposition of G00 where
fi1 ≤ fi2 ≤ · · · ≤ fi n−1 ≤ fin for i = 1, 2, . . . , k. Some of the fij s could be zero.
e f
Now, define di = ∏kj=1 pj ji , d0i = ∏kj=1 pj ji . Note that di is the product of the prime
powers that occur in the ith column on Eqn. (6) and d0i is the product of prime powers
that occur in the ith column of Eqn. (7). Then, since ei1 ≤ ei2 ≤ · · · ≤ ei n−1 ≤ ein and
fi1 ≤ fi2 ≤ · · · ≤ fi n−1 ≤ fin , it follows that di | di+1 and d0i | d0i+1 . Also, regrouping the
terms, we have
G0 ' Zpe11 × Zpe21 × · · · × Zpek1 × Zpe12 × Zpe22 × · · · × Zpek2 × · · ·
1 2 k 1 2 k

× Zpe1n × Zpe2n × · · · × Zpekn × Zr (8)


1 2 k

' Zd1 × Zd2 × · · · × Zdn × Zr (9)


Note that, in Eqn. (8), we have used exercise E5) in the other direction. From E5), we
get Zpe1 × Zpe2 ' Zpe1 pe2 . We apply E5) repeatedly to get Eqn. (8) from Eqn. (9).
1 2 1 2
Similarly,
G0 ' Zpf11 × Zpf21 × · · · × Zpfk1 × Zpf12 × Zpf22 × · · · × Zpfk2 × · · ·
1 2 k 1 2 k
r
× Zpf1n × Zpf2n × · · · × Zpfkn × Z
1 2 k

' Zd1 × Zd2 × · · · × Zdn × Zr


So, di = d0i and so eij = fij for 1 ≤ i ≤ k and 1 ≤ j ≤ n. This completes the uniqueness
part of the proof.  49
Study Guide-I We now deduce the corollary for finite abelian groups.

Corollary 3: Let G be a finite abelian group. Then,

G ' Zd1 × Zd2 × · · · × Zdn (10)

where di > 1 and di | di+1 . Also,

G ' Zpn1 × Zpn2 × Zpnk (11)


1 2 k

where p1 , p2 , . . .,pk are primes.

Proof: We will prove Eqn. (10). The proof of Eqn. (11) is similar. Since every finite
abelian group is also finitely generated, we can apply Theorem 2 on page 44 to G. But,
Zr is an infinite group for r > 0. So, in Eqn. (1) on page 44, r = 0 if G is a finite abelian
group. The result now follows Theorem 2. 

The powers pn11 , . . .,pnkk in Theorem 3 are called the elementary divisors of the group
G. Let us now look at an example to understand Corollary 3.
Example 2: Consider the group

G ' Z6 × Z9 × Z12 × Z15

We can decompose G as a product of cyclic groups of prime power order as follows:

G ' (Z2 × Z3 ) × Z9 × (Z3 × Z4 ) × (Z3 × Z5 )


' (Z2 × Z4 ) × (Z3 × Z3 × Z3 × Z9 ) × Z5

Here, we have p1 = 2, p2 = 3 and p3 = 5. Let us arrange the powers as in Eqn. (6).


20 20 2 22
3 3 3 32
50 50 50 5
Note that, powers of three occurs maximum number of times and it occurs four times,
so we have arranged the powers in a grid with 4 columns. We have taken powers of
other primes to be zero while making sure that powers along a row are in ascending
order. The elementary divisors are 2, 22 , 3, 3, 3, 32 and 5.
We now multiply along columns to get d1 = 20 × 3 × 50 = 3, d2 = 20 × 3 × 50 = 3,
d3 = 2 × 3 × 50 = 6 and d4 = 22 × 32 × 5 = 180. These are the invariant factors of the
group G.
∗∗∗
Another application of Corollary 3 is in classifying abelian groups of a given order. Let
us look at an example to understand this.
Example 3: Let us use Corollary 3 to determine the number of non-isomorphic
abelian groups of order 180. The following are the possible elementary divisors of a
group of order 180 = 22 32 5:
1) 2, 2, 32 , 5 2) 22 , 32 , 5 3) 22 , 3, 3, 5 4) 2, 2, 3, 3, 5.
So, there are four non-isomorphic abelian groups of order 180.
∗∗∗

EXERCISES E4) Let Crs be a finite cyclic group of order rs where r and s are integers such that
(r, s) = 1. Then, Crs ' Cr × Cs .
E5) Find the elementary divisors and invariant factors of the group

50 Z6 × Z14 × Z15 × Z35


E6) Find the number of non-isomorphic abelian groups of order 2450 = 2 × 72 × 52 . Free Groups

4.5 SUMMARY

In this Unit, we have discussed the following:

1. Definition of a free group.

2. The description of groups by generators and relations.

3. The mapping property of the free groups.

4. The structure theorem for finitely generated abelian groups and its application in
classifying abelian groups of a given order.

4.6 SOLUTIONS/ANSWERS

E1) Check that 1, x, x2 , . . . , xn is a basis.




E2) Suppose that the G has rank n and let e1 , e2 , . . . en be a basis for G.
Define φ1 , φ2 , . . ., φn by
(
0 if i 6= j
φi (ej ) =
1 if i = j

Verify that φ1 , φ2 , . . ., φn forms a basis for Hom(G, Z). Note that Hom(G, Z) is
the analogue of the dual space of a vector space in the case of finitely generated
abelian groups.
G
E3) a) The group H is finitely generated. In fact, if e1 , e2 , . . .,en generate G, then
G
e1 , e2 , . . .,en generate H over Z.
b) No, the group G H need not be free. For example, Z is a free group over Z
generated by 1. But, the quotient Zn = Z/nZ is not a free group. If it is a free
abelian group, it will have a finite basis e1 , e2 , . . ., en .(This is because, if G is
a finitely generated group and S is another generating set, not necessarily
finite, then there is a finite subset of S that generates G. Here 1 generates
Z/nZ over Z.) But, in this basis there will be two representations for zero,
namely 0e1 + 0e2 + · · · + 0en and ne1 + ne2 + · · · + nen .

E4) Let us write

C = 1, x, x2 , . . . , xrs−1 , Cr = 1, y, y2 , . . . , yr−1 and Cs = 1, z, z2 , . . . , zs−1


  

If xi ∈ Crs , define a map ϕ xi = yi , zi . Check that this defines an isomorphism


 

of groups. The proof is similar to the proof that C6 ' C2 × C3 , given in page 62 of
Artin’s book.
E5) The elementary divisors are 2, 2, 3, 3, 5, 5, 7, 7 and the invariant factors are 210
and 210.
E6) There will be four non-isomorphic abelian groups of this order.
Solutions to exercises 1 and 3 on Page 233 under ‘7. The Free Group.’

Q1) No. The product of two cyclic groups of infinite order is abelian. The free group
on two generators is not abelian. 51
Study Guide-I Q3) Let us write a ∼ b if a and b are conjugates. Given a conjugacy class C there is a
reduced word of smallest length, say k, in the conjugacy class. Suppose
x = x1 x2 · · · xk . Then, x1 6= x−1
k since x ∼ x2 x3 · · · xk−2 and C will have a word of
smaller length, contradicting our choice of x. So, the closed loop formed from x
will be reduced also.
Suppose x0 is another reduced word in C of length k. Then, we claim that x0 is a
cyclic permutation of x so that x and x0 will give the same reduced, closed, loop.
Let us prove in general that if x and x0 are two reduced words of the same length
and x0 = uxu−1 for some reduced word u, then x0 is a cyclic permutation of x.
The proof is by induction on the length of u. Suppose u is a reduced word of
length one. Let us write x = x1 x2 · · · xk and x0 = x01 x02 · · · x0k . Since x and x0 have
the same length, either u and x1 cancel each other or u−1 and xk cancel out each
other. So, u = x1 −1 or u−1 = xk −1 . Suppose, u = x1 −1 . Then, u−1 = x1 and
uxu−1 = x2 x3 · · · xk x1 = x0 . So, x2 = x01 , x3 = x02 , . . .,xk = x0k−1 and x1 = x0k .
Thus, x0 is a cyclic permutation of x. If u−1 = x−1 k , check that
0
x = xk x1 x2 · · · xk−1 .
Suppose that, the result is true whenever the length of u is less than n and we
have uxu−1 = x0 where u is a reduced word of length n, n > 1. Since the length
of uxu−1 is the same as x, there has to be some cancellations and this can happen
only if u = vx−1
1 or u
−1 = x−1 v−1 for some word v of length n − 1. In the first
k
case, we have

uxu−1 =vx2 x3 · · · xk x1 v−1 = vyv−1 = x0 ,

say, with v a word of length n − 1 and y = x2 x3 · · · xk x1 . If u−1 = x−1


k v, we have

uxu−1 = vyv−1 = x0

where y = xk x1 x2 · · · xk−1 . So, by induction hypothesis, x0 is a cyclic permutation


of y. Since y itself is a cyclic permutation of x, it follows that x0 is a cyclic
permutation of x. So, we have proved that any conjugacy class corresponds to a
reduced loop.
Conversely, given a reduced closed word
x
xk 1 x2
· x3
· x
·· · 4

we can ‘cut it open’ at any point read off clockwise to get a reduced word. Two
reduced words, obtained by cutting open the reduced word at two different
positions will be conjugates of each other. For example, if we cut the above word
before x1 and after x1 , we will get x1 x2 · · · xk and x2 x3 · · · xk x1 and these words
are conjugates of each other. Therefore, if C1 and C2 are two different conjugacy
classes and the reduced words of smallest length x ∈ C1 and x0 ∈ C2 yield the
same reduced, closed, loop, then x and x0 are conjugates of each other, so
C1 = C2 .

Solutions to exercises 1, 7, 8 and 9 on page 234 under ‘8. Generators and


Relations’.

Q1) Let G be the group generated by a and b and let H be the subgroup generated by
bab3 and bab2 . It is enough to show that a and b are in H. We have
−1
bab2 bab3 = b−2 a−1 b−1 bab3 = b is in H. Since b and bab−2 in H, a is also in
H.

Q7) a) Any characteristic subgroup is carried to itself by inner automorphisms so


52 it is normal.
b) Let φ by any automorphism of G. Let us show that φ (Z(G)) = Z(G). Free Groups
Consider any φ (x) in φ (Z(G)) and y ∈ G. Then, since φ is onto, y = φ (z)
for some z ∈ G. Therefore, φ (x)y = φ (x)φ (z) = φ (xz) = φ (zx) since x is
Z(G). So, φ (x)y = φ (zx) = φ (z)φ (x) = yφ (x). Thus, φ (x) ∈ Z(G).
c) The group H is generated by elements of the form x1 x2 · · · xk where each xi
is of order n. So, K = φ (H) is generated by elements of the form
φ (x1 ) φ (x2 ) · · · φ (xk ). Since automorphisms preserves the order of an
element, φ (x1 ), φ (x2 ), · · · , φ (xk ) are elements of order n. So, K is a
subgroup of H. Also, let φ (yi ) = xi , 1 ≤ i ≤ k. Then, yi are elements of
order n. So, K = H.

Q8) All the subgroups of the quarternion group are normal. Only the centre is
characteristic. None of the subgroups of order four are characteristic. For
example, the subgroup {±i, ±1} is mapped to {±j, ±1} by the automorphism φ
defined by φ (±i) = ±j, φ (±j) = ±i and φ (±k) = ∓k, φ (±1) = ±1. (Check that
φ is an automorphism.)

Q9) a) Do it along the lines of E 7), c) above.


b) Note that C is generated by the commutators xyx−1 y−1 . So, xyx−1 y−1 = e
in G
C , i.e. xy = yx.

53
UNIT 5 APPLICATIONS OF SEMIGROUPS
Structure
5.1 Introduction
Objectives
5.2 Some Basic Concepts
Semigroups
Free Semigroups
5.3 Connections with (Semi)automata
5.4 Application to Formal Languages
5.5 Summary
5.6 Solutions/Answers To Exercises

5.1 INTRODUCTION
So far you have studied some aspects of groups from the textbook and the material in
this block. Now we look at simpler algebraic structures, i.e., semigroups and monoids.
In your undergraduate studies, you would have come across these algebraic objects in
passing. To start with, we recall their definition and some of their properties here.
Then, in the next two sections, we focus on the applications of semigroups in two
areas, namely, automata and formal languages, though there many other areas in
which the theory of semigroups is applied.

This unit is independent of the textbook by Artin. If you are interested in studying
more about the matter discussed here, you can refer to

1) ‘Applied Abstract Algebra’, by Lidl and Pilz, UTM, Springer-Verlag


2) ‘Algebra’, by PM Cohn, Wiley.

Objectives

After studying this unit, you should be able to

 define, and give examples of, a semigroup and a monoid;

 explain what a finitely generated semigroup/monoid is;

 define, and give examples of, a free semigroup/monoid;

 prove that different bases of a free semigroup/monoid must have the same
cardinality;

 explain what a semiautomaton is, and its relationship with a semigroup/monoid;

 explain the connections between formal languages and semigroups.

5.2 SOME BASIC CONCEPTS


Let us begin with a quick review of what a semigroup is. We will also present a few
of its properties that we would be using in the later sections.

Definition: A semigroup is an ordered pair (S, ) , where S is a non-empty set and 


is an associative binary operation on S .
Remarks: We will sometimes denote the semigroup (S, ) by S only, if the operation
is understood.

Commonly found examples of semigroups are (N, ), (N, ) (Z, ), (Z, ) and (R , ) .

Let us look in some detail at a few other examples.

Example 1: Let S   . Show that the set of all mappings from S to S , Map (S, S) ,
is a semigroup w.r.t. the composition of mappings.

Solution: Firstly, Map (S, S)   since S   . Next, if f , g  Map (S, S) , then


f  g  Map(S, S) . Finally, the composition of mappings is associative in general.
Therefore, (Map (S, S), ) is a semigroup.

Example 2: Show that every non-empty set can be turned into a semigroup.

Solution: Let S   . Define  : S  S  S : s1  s 2  s1 .


Then, you should check that (S, ) is a semigroup.

Try some exercises now.

E1) Check whether (M n(R ),  ), (R[x], ) and (Z, ) are semigroups.

E2) Let S be a non-empty set, and Re l (S) the set of all relations on S , i.e., subsets
of S  S . Define  : Re l (S)  Re l (S)  Re l (S) by
‘x (R1  R 2 ) y iff  z  S s.t. x R 1 z and z R 2 y’ ,
i.e., ‘ ( x , y)  R 1  R 2 iff  z  S s.t. ( x , z)  R 1 and (z, y)  R 2 ’.
Show that (Re l (S), ) is a semigroup.
[This is called the relation semigroup.]

E3) If (X, ) is a semigroup, then define  (X) to be the set of non-empty subsets of
X. Show that ( (X), ) is a semigroup, where
A  B  a  b,a  A, b  B A, B   (X).
[This is called the power semigroup of X.]

Now, as you know, a semigroup (S, ) would be a monoid if it has an identity


element, i.e., if  s  S such that s  x  x  s  x  x  S .

You also know that a semigroup (S, ) is a group if (S, ) is a monoid and every
element in S is invertible w.r.t.  .

For instance, (Z, ) is a monoid, while (N, ) is not. In fact, (N  {0}, ) is a


monoid with identity 0 , in which the only invertible element is 0 .

Now, within a monoid, we have a very natural group. Can you guess what it is?

Definition: Let (S, ) be a monoid. Define G S  {x  S | x is invertible} to be the


unit group of S , or the group kernel of S .

You can check that (G S , ) is a group, and hence the name ‘unit group’ is
appropriate. Why do you think it is called ‘group kernel’ also?

Let us consider some examples.


2
Example 3: Find the unit groups of (M n (R ), ), (Map(S, S), ) and (G, ) , where G Applications of
Semigroups
is a group and S is a non-empty set.

Solution: In the first case, the unit group is {A  M n ( R ) | A is invertible }  GL n ( R ) .


In the second case, it is the set of bijective functions from S to S .
In the third case, it is the whole of G , since every element of G is invertible.

Now, in the case of groups you studied subgroups and group homomorphisms. We
can define analogous objects for semigroups too.

Definition: Let (S, ) be a semigroup. Then T is called a subsemigroup of S if


T  , T  S and t 1  t 2  T  t 1 , t 2  T .
We denote this fact by T  S .

The next definition should not surprise you either.

Definition: Let (S1 , 1 ) and (S 2 ,  2 ) be two semigroups. A function f : S1  S 2 is a


semigroup homomorphism if f (a 1 b)  f (a )  2 f (b) a , b  S1 .

As in the case of groups, a semigroup homomorphism is a monomorphism,


(respectively epimorphism) if it is 1-1 (respectively onto). If a semigroup
homomorphism is both 1-1 and onto, we call it an isomorphism of semigroups.

As an example, consider f : (Z, )  ( N  {0}, ) : f (x)  | x | . Then, f is an


epimorphism, but not a monomorphism.

Here are some exercises now.

E4) Show that (N, )  (Z, ) .

E5) Find the group kernels of (N  {0},  ), (Z, ), { (S), } and Map(S, S) , where
S.

E6) Check whether f : (Z, )  (N  {0},  ) : f ( x )  0 is a homomorphism.

E7) Prove that ( ({1, 2, 3}), ) and ( ({a, b, c}), ) are isomorphic semigroups.

Subsemigroups have several properties analogous to those of subgroups.

Theorem 1: Any non-empty intersection of subsemigroups of a semigroup (S, ) is a


subsemigroup of (S, ) .

Proof: See E8.

This theorem allows us to give the following definition.

Definition: Let (S, ) be a semigroup, and   T  S . Then the subsemigroup of


S generated by T is the intersection of all the subsemigroups of S containing T . It
is denoted by  T  .

You can check that


i)  T  is the smallest subsemigroup of S containing T .
ii)  T   {t 1 t 2  t n | t i  T, n  N} .

3
Can you think of some examples of generating sets? For instance, given any
semigroup (S, ) , is S   S  ? In fact, it is. So, every semigroup has a generating
set, but the fewer the generators, the easier it is for us to ‘see’ the elements of the
semigroup. For instance, (N, )   N  . But (N,  )   1  also, since any element
of N is a finite sum 1  1    1 . In fact, (N, ) is an example of a finitely
generated semigroup, as you will just see.

Definition: (S, ) is called finitely generated if S   T  , where T is finite. If


| T |  1 , then (S, ) is called cyclic.

For example, (N, ) is cyclic. Is (N  {0},  ) cyclic?


Since 0  1 , (N  {0},  )   {0, 1}  , and hence it is finitely generated but not
cyclic.

An example of a semigroup which is not finitely generated is (N, ) . By the unique


factorisation theorem, every natural number, except 1 , is a product of prime numbers.
Also, the set of primes, P , is infinite. So (N, )   P  {1}  . Further, no finite
subset of P  {1} can generate (N, ) . Therefore, it is not finitely generated.

Here are some exercises now.

E8) Prove Theorem 1. (Note that an analogous statement is true for monoids.)
The union of
subsemigroups need not E9) Give an example of a semigroup S and two subsemigroups S1 , S 2 of S such
be a subsemigroup.
that S1  S 2 is not a subsemigroup of S .

E10) Show that every subsemigroup of a finite group G is a subgroup of G . Is the


same true for an infinite group? Give reasons for your answers.

E11) Under what conditions on a set S , will ( (S), ) be finitely generated?

Let us now look at a particular kind of semigroup which has extensive applications in
computer science.

5.2.2 Free Semigroups


In the last unit, you studied about free groups. A free semigroup can be defined along
the same lines, as you will see. It is, in fact, this semigroup which is the basic object
in applications to automata and formal languages.

Definition: Let B be a non-empty set. A semigroup (F, ) is called a free


semigroup on B if
F i) FB;
h ii) any mapping f from B into a semigroup (F, ) can be extended to a unique
semigroup homomorphism from F to F .
U f
B F
This is diagrammatically shown in Fig. 1. In this situation, B is called a basis of F ,
Fig. 1: F is free on B if and F is also called the word semigroup over B . (The reason for this name will be
the diagram is clear in Sec. 5.4.)
commutative

Let us consider a few examples.

4
Example 4: If B  {b} , then show that the free semigroup on B is Applications of
Semigroups
FB  {b, b  b  b 2 , b 3 , } , which is isomorphic to (N, ) .

Solution: Firstly, (FB , ) is a semigroup containing B .


For the second condition, for any map f : {b}  F , define h : FB  F by
h (b n )  [f (b)]n  n  1 .
Then h is a homomorphism.
Also, if h  : FB  F is any other homomorphism that extends f , then
h (b r )  [h (b)]r  [f (b)]r  h (b r ) r  1 , so that h   h . Thus, h is unique.
This shows that h satisfies (ii) of the definition.
Thirdly, define  : FB  N :  (b r )  r .
Then you can check that  is a semigroup isomorphism.

Remark: As you can see, the basis of a free semigroup F generates F.

Try some exercises now.

E12) Are (N, ), (N  {0},  ) and (R ,  ) free semigroups? Give reasons for your
answers.

E13) Define a submonoid and a free monoid on a set B , along the same lines as the
definitions related to semigroups.

You may well ask if given B   , there always exists FB . The following theorem
tells us about this.

Theorem 2 (Existence): For any set B   , there exists a semigroup F which is free
on B .

Proof: Let F  {b1b 2  b n | bi  B, n  N} , the set of all formal products (or strings)
of elements of B .
So, by definition, b1b 2  b n  c1c 2  c m iff n  m and bi  ci i  1 ,  , n.
Let x , y  F . So, x  b1 b 2  b r and y  c1c 2  c s , for some b i and c j in B .
Define x  y  b1 b 2  b r c1c 2  c s , i.e., the binary operation  on F is just the If x  F, then
concatenation (or juxtaposition). x  b1b 2  b n for some
Then (F, ) is a semigroup containing B .
b i  B. The length of
Now, let f : B  G be any map, where G is a semigroup.
x, denoted by l (x), is n.
Define h : F  G : h (b1  b r )  f (b1 ) f (b 2 )  f (b r ) .
Then h is a homomorphism, and h extends f .

Suppose h  is any other homomorphism from F to G that extends f . Then, for any
c1 c 2  c s  F ,
h (c1c 2  c s )  h (c1 ) h (c 2 )  h (c s )
 f (c1 ) f (c 2 )  f (c s )
 h (c1 c 2  c s ) .
Therefore, h   h .
This shows that h is unique, and F is free on B .

Theorem 1 tells us that given a set we can always find a free semigroup on it. The
next question that arises is: can a free semigroup have more than one basis? If so, are
these bases related?
5
In the case of groups you have seen that a free group can have any number of distinct
bases. But, the cardinality of all these bases must be the same. This is also true for
semigroups, as we now prove.

Theorem 3: Let F be free on B and on B . Then | B |  | B | .

To prove this theorem, let us first prove a lemma.

Lemma: Let F and F be free semigroups on B and B, respectively. Then


F~ F   | B |  | B | .

Proof of lemma: Let us prove this diagrammatically, using Fig. 1. The situation here
is as in Fig. 2, where  is an isomorphism from F to F .


F ~ F

|
 ̄  ̄
B

B B
~ B
Fig. 2:  | : B 
B

For b  B , let  ( b)  b1  b n , b i  B .


Then b   1 (b1  b n )   1 (b1 )  1 (b 2 )   1 (b n ) .
Since the string lengths have to be the same, n  1 .
So  is a bijection from B onto B .
Thus, B  B .
Conversely, let B  B , and f : B  B be a bijection. Then, from Fig. 3, and using
~ F .
an argument similar to the one above, we see that F 

h
F F

i
2
i f
1 U U
B B
Fig. 3: h extends i 2  f : B  F

Proof of Theorem 3: Take F  F in the lemma, to get the result.

From the lemma we also get another useful result.

Theorem 4: Let F and F be two free semigroups on B . Then they must be


isomorphic.

Proof: Take B  B in the lemma, to get the result.

Try an exercise now.


6
Applications of
Semigroups
E14) Can a finite semigroup be free? Why, or why not?

E15) Find a free semigroup F , and an epimorphism from F onto (Z n , ) .

E16) Check whether or not  : ( Z,  )  ( Z n ,  ) :  (i)  i (mod n) is a


homomorphism. Is it a monomorphism?

The reason for discussing semigroups in this course is that they have several
applications. For instance, in biology they are being used for classifying organisms
vis-à-vis the hereditary laws. They are also useful for studying the DNA protein-
coding problem. Semigroups are also being used in some of the social sciences to
study various aspects of social and financial networks.

However, in this unit we shall consider the close relationship that semigroups and
monoids have with applications pertaining to automata and to formal languages.

5.3 CONNECTIONS WITH (SEMI)AUTOMATA


An automaton is a machine into which somethings are input, which transform into ‘Automata’ is the plural
some output. Examples are a telephone switch board, a computer, etc. In this section of ‘automaton’.
we will define automata in the abstract, and see that given an automaton we can obtain
a monoid, and vice versa.

Definition: A semiautomaton is a triple, s  (S, A, ) , where S and A are non-


empty sets and  : S  A  S . S is called the set of states, A is the input alphabet
and  is the “next-state function” of s .

So, as you can see, a semiautomaton doesn’t have an output function. For this, we
extend this object to an automaton.

Definition: An automaton is a 5-tuple A  (S, A1 , A 2 , ,  ) , where (S, A 1 , ) is a


semiautomaton, A 2 is a non-empty set (called the output alphabet) and
 : S  A1  A 2 is called the output function.

Relating this definition to the use of an input-output device, for s  S, a  A1 ,


(s, a )  S is the next state into which s is transformed by the input a . Further,
 (s, a )  A 2 is the output of s resulting from the input a .

Now, an automaton is called finite (or Mealy) if all the sets involved are finite. In
practical examples, there are usually a collection of only two switching states– on and
off. In such a situation, S will be Z 2  Z 2    Z 2 . And A1 and A 2 will look
similar too.

Let us consider some examples.

Example 5 (Marriage Automaton): Define the semiautomaton and automaton in the


following situation:
Hari and Wasima are a married couple. Wasima is always either angry, or bored, or
pleased. Hari either argues calmly, or shouts, or cooks their favourite dishes. When
he argues calmly it doesn’t change her mood. When he shouts, she gets angry. When
he cooks, she is pleased. Also, Wasima only shouts when she is angry and Hari
shouts. Otherwise she is quiet.

7
Solution: Let us take
s1 : Wasima is angry a 1 : Hari argues calmly
s 2 : Wasima is bored a 2 : Hari shouts
s 3 : Wasima is pleased a 3 : Hari cooks their favourite dishes
b1 : Wasima shouts
b 2 : Wasima is quiet.
Then take S  {s1 , s 2 , s3 } , A1  {a1 , a 2 , a 3 } and A 2  {b1 , b 2 } . Define the functions 
and  by the following tables.

 a1 a2 a3  a1 a2 a3
s1 s1 s1 s3 s1 b2 b1 b2
s2 s2 s1 s3 s2 b2 b2 b2
s3 s3 s1 s3 s3 b2 b2 b2

Then (S, A1 , A 2 , ,  ) is an automaton.

Example 6: Let S  {s 0 , s1 }, A1  A 2  {0, 1} , and let  and  be given by

 0 1  0 1
s0 s0 s1 s0 0 1
s1 s1 s0 s1 0 1
Give a situation that could be described by (S, A1 , A 2 , ,  ) .

Solution: One situation could be the following:


Take s 0 : Machine stores 0 , s1 : Machine stores 1 .
The input i operates on S by taking the state s i to the state s i 1 , addition being
modulo 2. The output is the same as the input.

Try some exercises now.

E17) i) A stamp automaton s has a capacity of ten stamps. Define this as a


semiautomaton such that the state s i means there are i stamps in s ; the
inputs are ‘no coin is inserted’, ‘a correct coin is inserted’ and ‘a wrong coin
is inserted’. Describe this semiautomaton by a table.
ii) Extend (i) to an automaton, taking the outputs as ‘no output’, ‘a stamp’, ‘a
coin’. Describe the automaton by a table.

E18) i) Given a semiautomaton (S, A, ) , with A  S , is (S, ) a semigroup, where


a  b  ( a , b )  a , b  S ?

ii) Further, given a semigroup (S, ) , can it be considered as a semiautomaton


(S, S, ) ? Give reasons for your answers.

What E18 tells us is that any semigroup gives rise to a semiautomaton. In fact, the
converse is also true, but we will not be proving it over here. These facts may give
you some sense of how the study of semigroups can help us understand
(semi)automata.

Let us now move to another application of semigroups.

8
Applications of
5.4 APPLICATION TO FORMAL LANGUAGES Semigroups

You may be familiar with two or three languages that are spoken around you, like
English, Hindi, etc. These are examples of ‘natural’ languages. Apart from such
languages, mathematicians and computer scientists have defined ‘formal’ languages.
These languages also have alphabets and words, but the words may not mean anything
to a listener or reader. Such languages focus on syntax, and are useful for studying
linguistic patterns as well as the syntax of programming languages.

Let us define such a language formally now.

Definition: Let A be a non-empty set. Let A* be the free monoid on A , i.e., it is


FA  {} , where  is the empty word. So A* is the set of all finite strings of
elements of A , i.e., A*  a1a 2  a n | a i  A, n  N  0 , together with the
operation of concatenation (or juxtaposition).
A formal language L over the set A is a subset of A* . Here A is called the
alphabet over which L is defined, and elements of A* are called words.

For example, L can be all of A* . This is called the universal language.


Or, if A  {a , } , then L can be A , or L can be {aaa  a,  a } , or any other finite
or infinite subset of A * . (Here, note that '' is just a formal symbol, not carrying the
usual meaning you associate with it, i.e., equality.)

Another possibility is L   , which is called the empty language.

So, as you can see from the above, a formal language is about the form (i.e., the
syntax) and not about meaning. Further, the mathematical theory of formal languages
doesn’t study individual languages, but the classes of language, and the mechanisms
that describe these classes. Noam Chomsky, American linguist and philosopher, has
presented an hierarchy of these classes, viz., L1  L 2  L 3  L 4 , where
L1 : Regular languages, characterised by finite state automata; Fig. 4: Noam Chomsky
(born: 1928)
L 2 : Context-free languages, characterised by pushdown automata;
L 3 : Context-sensitive languages, characterised by linear bounded automata;
L 4 : Computable languages, characterised by Turing machines.

Let us see what the semigroups/monoids are that characterise these languages. For
this, let us consider Chomsky’s approach to formal languages. This is based on the
use of grammar, which we now define.

Definition: A phase-structure grammar is a 4-tuple G  (A, G, , g 0 ), where A


and G are non-empty, disjoint finite sets, g o  G and  is a finite relation from
FV into V*, i.e., a finite subset of FV  V*, where V  A  G. This 4-tuple is often
called ‘grammar’, in short.

Here A is called the alphabet of G; G is the set of grammar symbols; V is the


complete vocabulary; the elements of  are called rewriting rules; and g 0 is the
initial symbol.

The relation  is the heart of the grammar. It tells us how one string transforms into
another. For x  FV , y  V* , we will write (x, y)  also as x  y. For instance, if
w1  uxv and w 2  uyv , then x  y applied to w1 gives us w 2 . This is also

9
indicated
⇒w by saying that w1 derives w 2 , or that w 2 is derived from w1 , and denoted
by w1 2.

So, a succession of strings can be obtained by applying the rewriting rules.


If w1  w 2    ⇒w w n , then we say that w1 , derives w n , and denote this string of
*
derivations by w1 n.
*
So, ⇒ could be zero, one or more applications of the rewriting rules. For example,
*
w⇒ w is always true.

Now, we are ready to define a language based on a grammar.

Definition: Let G  (A, G,  , g o ) be a phase-structure grammar. Then


 * 
L(G )   w  A* g o ⇒ w  is called the language generated by G.
 
Note: L (G )  A* , while {w g 0 
*
w}  V* .
Let us consider some examples.

Example 7: If G  a , g 0  , g 0  a , g 0  , find L (G ).

Solution: Firstly, g 0  a, by definition of a grammar. Now, any element of L (G ) is


of the form w  {a}* such that g o * w. Since w {a}*, w  a n , where n  N, or
w  .
If w1 is derived from g 0 by applying g 0  a, then w1  a. Also, w1  w 2 by
applying g 0  a gives us w2 =a, since there is no presence of g 0 in the string w1.
Thus, L (G )  a.

Example 8: Let G  a , g 0  , g 0  a, g 0  ag 0  , g 0  . Find L (G ).

Solution: Here g o  z means z  a or z  ag 0 . Applying any rewriting rule to


z gives us z1  a or z1  aa or z1  aag 0 .
Thus, the strings of A* that can be derived from g 0 by applying  are a, aa, aaa, 
 
So L (G )  a n n  N  FA , where A  a .

Why don’t you try some exercises now?

E19) Let A  a, b , G  g 0  , where g 0  A. Determine L (G ) if the set of


rewriting rules is given by
i) g 0  a, g 0  b
ii) g 0  , g 0  a, g 0  aba
iii) g 0  g 0 
E20) Find a grammar that generates b, aba, aabaa, aaabaaa, 

E21) Find a grammar with the alphabet set A that generates A* , i.e., the universal
language.

10
Applications of
Semigroups
Now, let us look at what kind of grammars make up the classes L1 to L 4 in
Chomsky’s hierarchy, mentioned above.

Definitions: 1) A language L  A* is regular if it can be generated by a grammar


with rewriting rules of the form x  ay, x   for x  G, a, y  A. (For those of you
familiar with deterministic finite accepters (dfa), a language L is regular iff
L  L (M) for some dfa M.)

2) A grammar G is called context-sensitive if for every rewriting rule x  y in


G , we have l (x)  l (y), where l (x) is the length of the string x  A* . A
language L is called context-sensitive if L  L (G ) for some context sensitive
grammar G .

3) A grammar G is called context-free if for every rewriting rule x  y in


G , we have l (x)  1. A language L is called context-free if L  L (G ) for some
context-free grammar G .

4) A language L is called a computable language if L  L (G ) for some grammar


G.

Let us look at some examples.

Example 9: Show that every finite language is regular.

Solution: Let L  x1 , x 2 ,  , x n   A* for some A. Each of these elements can be


obtained from a finite set of elements S of A by applying the rules of the form
g 0  g 0 a, and g 0   , where g  G, a  S. Therefore, L is regular.

 
Example 10: Show that L  xy n n  0 is a regular language.

Solution: Here A  x, y , G  g 0  , g 0  A, and the rules are g 0  x, g 0  g 0 y.


Then L  L (G ), and hence L is regular.

Remark: You can check that these languages are also context-free.

Now, there is a lot of work going on in the area of characterising the various languages
in terms of semigroups. We will state, without proof, a result here to help us get an
example of a language which is not regular.

Theorem 5: Let A  a and L  A* . L is regular if and only if A set p n n  N is called

 
L  a n  P , where P is a periodic subset of N  0 .
n
periodic if  k, n 0  N
such that p n  k  p n is

 
Using result, this we can immediately say that a n n  N  0 is not regular since
2 constant  n  n o .

n 2

n  N  0 is not a periodic set.

Try some exercises now.

11
E22) Show that L (G1 )  L (G2 ), where
Different grammars can G1  a, b , g 0  , g 0  g 0 g 0 , g 0  aa , g 0  and
G2  a, b , g 0  , g 0  ag 0 a, g 0  aa , g 0  .
generate the same
language.


E23) Check whether a n n  3(mod 4) is regular. 
You have seen that a language is a subset of a free monoid. There are some operations
that can be applied on the set of languages over an alphabet A . These are

i) Concatenation: If L1 and L 2 are languages over A , then define


L1 L 2  {vw | v  L1 , w  L 2 } .
ii) Intersection: If L1 and L 2 are languages over A , then define
L1  L 2  {v | v  L1 and v  L 2 } .
iii) Complement: If L is a language over A , then define
Lc  A* \ L .
iv) Reversal: If L is a language over A , define its reversal to be
LR  {w R | w  L} , where R  , (a 1a 2  a n ) R  a n a n 1  a 2 a 1 .
v) Union: If L1 and L 2 are languages over A , then define
L1  L 2  {v | v  L1 or v  L 2 } .

With this we come to the end of our discussion on semigroups, monoids and their
applications.

Let us now take a look at the points covered by us in this unit.

5.5 SUMMARY
In this unit we have discussed the following points.

1) The definition, and some examples, of a semigroup/monoid.

2) What the group kernel (or the unit group) of a monoid is.

3) The definition of a subsemigroup and of a semigroup


homomorphism/isomorphism.

4) Any non-empty intersection of subsemigroups of a semigroup S is a subsemigroup


of S. This is not true if ‘intersection’ is replaced by ‘union’.

5) The subsemigroup of S generated by T  S is the smallest subsemigroup of S


containing T, which is  Si Si  S, T  Si . This is denoted by  T  .
i

6) The definition, and examples, of a free semigroup.

7) A free semigroup is infinite, even if its basis is finite.

8) For any set A   , there exists a semigroup FA which is free on A. In fact,


FA  a1a 2  a n a i  A}, the set of all formal products of elements of A.

9) Let a semigroup F be free on a set A and on a set A. Then A  A .

10) If F and F are both free semigroups on A , then F  F.


12
Applications of
11) The definition, and examples, of (semi)automata. Semigroups

12) Any semigroup gives rise to a semiautomaton, and vice versa.

13) For A   , A*  FA   is a monoid with respect to concatenation, where


 is the empty word. A formal language L over the set A is a subset of A* .

14) The definition of the operations of intersection, union, concatenation,


complement and reversal on the set of languages.

5.6 SOLUTIONS/ANSWERS
E1) All three sets are non-empty, and closed w.r.t. the operations given. However,
the first two are semigroups, and (Z, ) is not, since ‘–’ is not an associative
operation.

E2) Since S  , S  S   . Therefore, Rel (S)   .


For R 1 , R 2  S  S, R 1  R 2  S  S .
Therefore,  is a binary operation.
Now, let R 1 , R 2 , R 3  Rel (S) .
Then (, )  (R 1  R 2 )  R 3
  s  S s.t. (, s)  R1  R 2 and (s, )  R 3
  s  S, t  S s.t. (, t)  R 1 and (t, s)  R 2 and (s, )  R 3
  t  S s.t. (, t)  R1 and (t, )  R 2  R 3
 (, )  R1  (R2  R 3 )
Therefore,  is associative.
Thus, (Rel (S), ) is a semigroup.

E3)  (X)  , since X  . Next,  is a binary operation on  (X) . Finally,


(A  B)  C  A  (B  C)  A, B, C   (X), since  is associative on S.

E4) Since n1  n 2  N  n1 , n 2  N, it follows that (N, )  (Z, ) .

E5) i) As noted earlier the unit group for this is {0} .


ii) In this case the identity is 1.
 the unit group is { 1} .
iii) Here the identity is S . Therefore, the only subset of S which has an
inverse is S .
iv) Id : S  S is the identity map.
Thus, Bij (S, S) , the set of all bijective mappings from S onto S , is the
group kernel of Map (S, S) .

E6) f (x.y)  0  0  0  f (x)  f (y) x, y  Z .


 f is a homomorphism.

E7) Define  :  ({1, 2, 3})   ({a, b, c}) : ()  , (1)  a, (2)  b, (3)  c , and
extend  elementwise.
The subsets of {1, 2, 3} are , {1}, {2}, {3}, {1, 2}, {1, 3}, {2, 3}, {1, 2, 3} .
So, under  , the images of these are
, {a},{b}, {c}, {a, b}, {a, c}, {b, c}, {a, b, c} .

13
You can check that  (S1  S2 )   (S1 )   (S2 )  subsets S1 , S 2 of {1, 2, 3} .
Also,  is clearly a monomorphism and an epimorphism. Hence,  is an
isomorphism.

E8) Consider T   Si , where Si  S  i  I , the indexing set


iI
For x, y  T, x, y  Si  i  I .
 x  y  Si  i  I
 x  yT
 T  S.

E9) Consider S  (Z ,  ), and take S1  (N,  ), S2  ( Z  ,  ), where Z  is the set of


negative integers.
Then S1  S2  Z \ 0 , which is not a semigroup w.r.t. addition.

E10) Let S  {s1 ,  , s n } be the subsemigroup of G .


Consider s  S , and take S  {ss1 , ss 2 ,  , ss n } . Then S  S and | S |  | S | .
 S  S .
  i s.t. ss i  s in S , and in G .
 si  e .
Thus e  S .
Using a similar argument, we can show that s 1  S  s  S .
Thus, S is a subgroup of G .
However, if G is infinite, this need not hold. E.g., (N, )  (R* , ) , but (N, )
is not a group.

E11) In general,  (S) is generated by itself. Thus, if  (S) is finite, i.e., if S is


finite, then  (S) will be finitely generated.
Now, take the case when S is infinite. Then  (S) is infinite. Suppose it is
finitely generated, say, by S1 ,  , Sr . Then
 
S1 ,  , Sr  Si1    Sin 1  i1  i 2    i n  r is also a finite collection of
subsets of S, and hence this is not infinite. Therefore, it cannot be  (S) . Thus,
 (S) is f.g. iff S is finite.

E12) (N, ) is not free. [Had it been free, it would be a free commutative semigroup.
So it would be isomorphic to (N, ) . But (N, ) has no identity element,
while (N, ) does. So, we reach a contradiction.]
On the same lines you can show that (N  {0}, ) and (R, ) are not free
semigroups.

E13) i) A non-empty subset A , of a monoid (M, ) , is called a submonoid if A


contains the identity of M and is closed w.r.t.  .

ii) A monoid (M, ) is called free (on a non-empty set B ) if


 M  B ; and
 any mapping f from B into a monoid ( M , ) can be extended to a
unique homomorphism from M to M  .

E14) Let s  S . If S is free, then {s n | n  N}  S , where s n  s m for n  m.


 S can’t be finite.

14
Applications of
Semigroups
E15) Let {a 1 , a 2 ,  , a r } generate ( Z n , ) , where a i  N . Take F to be the free
semigroup on {a 1 , a 2 ,  , a r } . Define
 : F  Z n :  (x1x 2  x k )  (x1  x 2    x k ) mod n , where x 1 x 2  x k is a
string in F .
Then  is surjective, and  ( xy)   ( x )   ( y)  x , y  F .

E16) Since  ( x  y)   ( x )   ( y),  is a homomorphism. It is not 1  1 , since,


e.g.,  (0)   (n ) and 0  n in Z .

E17) i) S  {s 0 , s1 , s 2 ,  , s10 }, A  {a 1 , a 2 , a 3 } , where


a 1 : No coin is inserted
a 2 : A correct coin is inserted
a 3 : A wrong coin is inserted

 : S  A  S is defined by the following table.

 a1 a2 a3
s0 s0 s0 s0
s1 s1 s0 s1
s2 s2 s1 s2
   
s10 s10 s9 s10

ii) Let A 2  {b1 , b 2 , b 3 } , where


b1 : No output
b 2 : A stamp
b 3 : A coin
Then  : S  A1  A 2 is defined by the following table:

 a1 a2 a3
s0 b1 b3 b3
s1 b1 b2 b3
s2 b1 b2 b3
   
s10 b1 b2 b3

E18) i) Firstly, S  .
Next, a  b  S  a , b  S .
Thirdly, (a  b)  c  ((a , b), c) and a  (b  c)  (a, (b, c) for
a , b, c  S .
Therefore,  need not be associative.
So, (S, ) need not be a semigroup.

ii) It can, since  : S  S  S.

E19) i) Starting with g 0 , any derivation will lead to a or b and to no other string.
So, L(G )  a, b.

15
ii) Here, any string can be , a, aba.
So, L(G )  , a, aba .

iii) Here, there is no string in A* derived from g 0 . So, L(G )  .

E20) Take G = a, b , g 0  , g 0  b, g 0  ag 0 a , g 0  , where g 0  a, b.


Then the strings would be b, aba, aabaa, .

 
E21) G = A, g 0  , g 0  , g 0  g 0 a a  A , g 0 , where g 0  A.
Then L (G )  A* .


E22) In both cases L   aa  n  N .
n

 
E23) Since n n  3(mod 4) is a periodic set, a n n  3(mod 4) is regular.

16
UNIT 6 CONGRUENCES AND APPLICATIONS
Structure Page No.
6.1 Introduction 71
Objectives
6.2 Basic Results on Congruences 71
6.3 The Chinese Remainder Theorem 75
6.4 The Quadratic Reciprocity Law 78
6.5 Applications of Congruences 87
Primality Testing
Error Checking Using Congruences
6.6 Summary 91
6.7 Solutions/Answers 92

6.1 INTRODUCTION
In this unit, we will discuss congruences and their applications. Gauss, in his book
Disquisitiones Arithmeticae formulated the notion of congruences and introduced the
notation that we use for congruences at present. With the help of the notion of
congruences he revolutionised number theory and changed it from a collection of
isolated results, due to other mathematicians like Euler, Fermat, Lagrange and
Legendre, into a coherent subject. He not only reformulated many results known earlier
in terms of congruences, he also proved many new results. In the recent times, P. Fermat
congruences have led to many interesting applications in computing. In Sec. 9.2, we (1601–1665)
prove basic results regarding congruences using basic concepts from algebra that you
have studied in your degree course. In Sec. 9.3, we will prove the Chinese remainder
theorem, which has many applications, and derive some of its consequences. One of the
results in the study of congruences, which is important from both theoretical and
applications point of view, is the quadratic reciprocity law. In Sec. 9.4, we will prove
quadratic reciprocity which was proved rigorously by Gauss although the result was
known earlier to Euler and Legendre. In Sec. 9.5, we will discuss some applications of
congruences to primality testing and in checking whether an ISBN number is valid or
not.
Objectives C. F. Gauss
(1777–1855)
After studying this unit, you should be able to
• solve linear congruences ( mod n);
• use the Chinese Remainder Theorem to solve simultaneous linear congruences;
• calculate the legendre symbol;
• solve the equation x2 − a = 0 (mod p), when p is a prime and a and p are odd
numbers coprime to each other, using quadratic reciprocity;
• explain how congruences are used for checking primality and for checking whether
an ISBN number is valid number or not.

6.2 BASIC RESULTS ON CONGRUENCES


In this section, we quickly recall some of the basic concepts from algebra. As we
expect you to be familiar with them, we just state the main results that we require 71
Study Guide-I without proofs. While doing so, we also set up the notations and conventions necessary
in the rest of the unit.
Recall that Z is a Euclidean Domain and hence a Principal Ideal Domain(PID) and a
Unique Factorisation Domain(UFD).
Z
If n ∈ Z, is also a ring, denoted by Zn , and we have a canonical ring
(n)
homomorphism
ψ : Z −→ Zn (1)
We write a for the image ψ(a) of a ∈ Z. Recall that a = ψ(a) is actually a set and not a
single element. In fact
ψ(a) = a + (n) = {a + kn|k ∈ Z}
We call a, the residue class of a. We have a = b if and only if a − b ∈ (n) or
equivalently, n | a − b. If a = b, we write a ≡ b (mod n) which is read as ‘a is
congruent to b modulo n.’. (Note that ≡ is an equivalence relation.) The map ψ gives
us a method of translating the results about the ring Zn into an assertion regarding
congruences. We will frequently use this to move back and forth between results
regarding congruences and results regarding the ring Zn .

Definition 8: We say that {a1 , a2 , . . . , an }, where ai ∈ Z, is a complete set of residues


modulo n if ai 6≡ aj (mod n) for i 6= j.

One natural set of complete residues modulo n is {0, 1, 2, . . . , n − 1}.


As an immediate consequence of the ring homomorphism ψ in Eqn. (1), we get the
following result:

Proposition 1: If a ≡ b (mod n) and c ≡ d (mod n), then a + c ≡ b + d (mod n) and


ac ≡ bd (mod n).

We leave the proof to you as an exercise.


In many applications, we have to find a solution the congruence Eqn. (2).
ax ≡ b (mod n) (2)
How can we do this?
This is equivalent to finding a solution to the equation
ax = b (3)
in Zn .
For example, finding a solution to 3x ≡ 5 (mod 7) is equivalent to finding a solution to
the equation 3x = 5 in Z7 .
If a is a unit in Zn , then x = a−1 b is a solution to Eqn. (3). The next proposition tells us
when is a a unit in Zn .

Proposition 2: a ∈ Z
(n) is a unit if and only if (a, n) = 1.

Proof: Suppose (a, n) = 1. In general, if d is the gcd of a, n, then there are u, v ∈ Z


(a, n) stands for the greatest such that ua + vn = d. Since (a, n) = 1, we can find u and v such that ua + vn = 1. We
common divisor of a and n. have
ψ(1) = ψ(ua + vn) = ψ(u)ψ(a) + ψ(v)ψ(n)
= ψ(u)ψ(a), since ψ(n) = 0
= ua
72 = 1 since ψ(1) = 1 from the RHS
So, a u = 1. Thus, u = a−1 . We leave it to you to prove that, if a is a unit in Zn , then Congruences and
Applications
(a, n) = 1. 

Corollary 4: If (a, n) = 1, any x such that x = a−1 .b is a solution to the congruence


ax ≡ b (mod n).

In the proof of Proposition 2, we showed that, if (a, n) = 1 and u and v are such that
ua + vn = 1, then u is the inverse of a. Translated in terms of congruences, this means
that u is a solution to the equation ax ≡ 1 (mod n). So, to find a−1 , we have to find u
and v such that au + vn = 1.
Recall that, in general, if (p, q) = d, then we can find d using Euclidean algorithm. The
same algorithm can be modified to find x and y such that xp + yq = d. The general
procedure is as follows: There is nothing to do if p = q because d = p = q in this case.
So, suppose p < q. Then, we can find q1 and d1 , 0 ≤ d1 < p such that q = p.q1 + d1 . We
stop if d1 = 0 because p | q and so (p, q) = p. Again, we can find q2 and d2 , 0 ≤ d2 < d1
such that p = q2 d1 + d2 . If d2 = 0, we stop. The gcd is d1 . Otherwise, we find q3 and d3
such that d1 = q3 d2 + d3 and so on. Since d1 > d2 > d3 > · · · , and di are non-negative,
for some n, we must have dn = 0(Why?). Then, dn−1 is the gcd of p and q.
We can modify the same algorithm to find x and y such that xp + yq = d. All we need is
to do some additional ‘book keeping’. We have q = pq1 + d1 , so,
d1 = −q1 p + q = x1 p + y1 q, say. Again, we have p = d1 q2 + d2 , so

d2 = p − d1 q2 = p − q2 (−q1 p + q) = (1 + q1 q2 ) p − q2 q = x2 p + y2 q

say. Let us write di = xi p + yi q. Then, we have from the above discussion,

x1 = −q1 , y1 = 1, x2 = 1 + q1 q2 , y2 = −q2 (4)

di−1 = qi+1 di + di+1


∴ di+1 = di−1 − qi+1 di
= (xi−1 p + yi−1 q) − qi+1 (xi p + yi q)
= (xi−1 − qi+1 xi ) p + (yi−1 − qi+1 yi )

But,

di+1 = xi+1 p + yi+1 q


∴ xi+1 = (xi−1 − qi+1 xi ) yi+1 = (yi−1 − qi+1 yi ) (5)

Using Eqn. (4) and Eqn. (5), we can easily calculate xi , yi recursively for all i ≥ 3.
Example 1: We have (93, 141) = 3. Find u and v such that 93u + 141v = 3 using
Euclidean algorithm.
Solution: The steps of the Euclidean algorithm are as follows: We have
141 = 1 · 93 + 48 , so, q1 = 1, d1 = 48. Therefore, from Eqn. (4),

x1 = −1, y1 = 1

Again, 93 = 1 · 48 + 45. Therefore, q2 = 1, d2 = 45, x2 = 1 + q1 q2 = 2,


y2 = −q2 = −1. Thus,

x2 = 2, y2 = −1

48 = 1 · 45 + 3, So, q3 = 1, d3 = 3, x3 = x1 − q3 x2 = −1 − 2 = −3,
y3 = y1 − q3 y2 = 1 − 1 · 1 = 2. We note that d3 | d2 and so d4 = 0. So, we can stop here.
We have x3 = −3 and y3 = 2, so 3 = −3 · 93 + 2 · 143, i.e. u = −3, v = 2.
∗∗∗ 73
Study Guide-I Let us now look at an example to see how to solve congruences of the type in Eqn. (2).
Example 2: Find a solution to the equation 3x ≡ 5 (mod 7).
Solution: Here (3, 7) = 1. So, we can find u, v ∈ Z such that 3u + 7v = 1. We have
−1
7 = 2 · 3 + 1. So, 1 = 7 − 2 · 3. Therefore, we can take u = −2, v = 1. Hence, 3 = −2.
We have,
−1
x≡3 · 5 ≡ −2 · 5 ≡ −10 ≡ 4 (mod 7)
Thus, x = 4 is a solution to the congruence 3x ≡ 5 (mod 7).
∗∗∗
We next prove a result regarding cancellation of a constant occurring in both the sides
of a congruence.

Proposition 3: If (a, n) = 1 and a` ≡ am (mod n), then ` ≡ m (mod n).

Proof: In Zn , we can translate a` ≡ am (mod n) as a` = a m in Zn . Since (a, n) = 1, a


is a unit. So, we can multiply both sides of the equation a` = a m by a−1 to get ` = m.
Translating this back into congruences, we get what we want. 

What can we say about the solution to Eqn. (2) in general? Here is the result.

Proposition 4: The congruence ax ≡ b (mod n) has a solution if and only if (a, n) | b.

Proof: Let d = (a, n). If x ∈ Z is a solution to Eqn. (2), then n | (ax − b). Since d | n,
d | (ax − b). Since d | a, d also divides b.
Conversely, if d | b, n | (ax − b) if and only if dn da x − db . So, Eqn. (2) has a solution if


and only if da x ≡ bd (mod dn ) has a solution. Since da , nd = 1 we can use Corollary 4 to



Note that, if a ≡ b (mod n),
ad ≡ bd (mod dn). get a solution to the congruence da x ≡ bd (mod nd ). 

Here are some exercises for you to try.

EXERCISES E1) If a ≡ b (mod n) and c ≡ d (mod n), show that:


i) a + c ≡ b + d ≡ (mod n)
ii) ac ≡ bd (mod n).
E2) Solve the following congruences:
i) 3x ≡ 2 (mod 17) 2) 4x ≡ 6 (mod 18)

Note that the units in Zn form a group, usually denoted by Z∗n . From Proposition 2, it
follows that
Z∗n = a ∈ Zn \ {0} (a, n) = 1


If S is a complete set of residues for Zn , then


Z∗n = {a | a ∈ S, (a, n) = 1}
In particular, we can take S = {1, 2, . . . , n − 1}. Then,
Z∗n = {a | 1 ≤ a ≤ n − 1, (a, n) = 1} (6)
We write
φ (n) = |Z∗n |
From Eqn. (6), we have
φ (n) = |{a | 1 ≤ a ≤ n − 1, (a, n) = 1}| (7)
74 φ (n) is called the Euler phi-function.
Proposition 5(Euler’s Theorem): If (a, n) = 1 where a ∈ Z, then Congruences and
Applications
aφ (n) ≡ 1 (mod n) (8)

Proof: For any finite group G and any a ∈ G, we have a|G| = 1. In the case of Z∗n , we
have aφ (n) = 1 ∀a ∈ Z∗n . If a ∈ Z and (a, n) = 1, then a ∈ Z∗n and aφ (n) = 1. Translating
this in the language of congruences, aφ (n) ≡ 1 (mod n). 

As it stands, Eqn. (8) doesn’t tell us much regarding the computation of φ (n). Later, we
will see an expression for φ (n) in Eqn. (17). However, when p is a prime, we get the
following interesting result immediately.

Corollary 5(Fermat’s Little Theorem): If p is a prime, a ∈ Z and (p, a) = 1

ap−1 ≡ 1 (mod p) (9)

Proof: For every a ∈ Z, 1 ≤ a ≤ p − 1, we have (a, p) = 1. So,

|{a|1 ≤ a ≤ p − 1, (a, p) = 1}| = |{a|1 ≤ a ≤ p − 1}| = p − 1

The result now follows from Proposition 5. 

We close this section here. In the next section, we will see how to solve simultaneous
congruences, for example, pairs of congruences of the type x ≡ 3 (mod 11), x ≡ 2
(mod 7).

6.3 THE CHINESE REMAINDER THEOREM


In some computers, modular arithmetic is used to add and multiply large integers. The
idea is as follows: Suppose we have to add two large numbers N1 and N2 . The numbers
may be too large that they may fit within a single word in computer. For example, the
word size in 32 bit computers is 232 and N1 and N2 may be large compared to this. We
can break up the task of adding N1 and N2 into adding numbers which are smaller as
follows: We pick some natural numbers n1 , n2 , . . ., nk such that all of them are pairwise
coprime and smaller than the word size. Suppose N1 ≡ ai (mod ni ) and N2 ≡ bi
(mod ni ). We find (a1 + b1 , a2 + b2 , . . . , ak + bk ). Using Chinese Remainder Theorem
that we will discuss in this section, we can then find N1 + N2 from
(a1 + b1 , a2 + b2 , . . . , ak + bk ).
Let us first look at an example involving the Chinese Remainder Theorem.
Example 3: A class has to be divided into groups for carrying out an activity. When the
teacher divided the class into three groups, one student was left and cannot be assigned
to any group. When she divided the class into five groups, three students were left.
When she divided the class into seven groups, 6 students were left. What is the
minimum number of students in the class?
Solution: Suppose the minimum number of students in the class is x. Since one
student was left if the class was divided into three groups, x ≡ 1 (mod 3). Similarly,
from the other information we have, we get the congruences x ≡ 3 (mod 5) and x ≡ 6
(mod 7). So, we have to find the smallest solution to the simultaneous congruences

x ≡ 1 (mod 3)
x ≡ 3 (mod 5)
x ≡ 6 (mod 7)

We will see how to solve this using the Chinese Remainder Theorem in Example 4.
∗∗∗ 75
Study Guide-I Theorem 4: If n1 , n2 , . . ., nk are pairwise relative prime integers (i.e. (ni , nj ) = 1 if
i 6= j) and a1 , a2 , . . .,ak are any integers, there is a solution x0 to the following
simultaneous congruences:

x ≡ a1 (mod n1 ) 

x ≡ a2 (mod n2 ) 


.. (10)
. 


x ≡ an (mod nk )

If x0 and x00 are two solutions, then x0 ≡ x00 (mod N), where N = n1 n2 · · · nk .

Proof: Let us first solve a special case of Eqn. (10). Let us fix an i and suppose that
ai = 1 and aj = 0, for j 6= i. Let

Ni = ∏ nj
j6=i

Then, (Ni , ni ) = 1 and we can find integers a and b such that aNi + bni = 1. This gives
the congruences

aNi ≡ 1 (mod ni ) (11)


aNi ≡ 0 (mod nj ) for j 6= i (12)

since Ni is divisible by nj if j 6= i. So, xi = aNi satisfies the

xi ≡ 0 (mod nj ) for j 6= i (13)


and xi ≡ 1 (mod ni ) (14)

For each i, 1 ≤ i ≤ k we find an xi satisfying Eqn. (13) and Eqn. (14). We can use the
xi s to get an x satisfying Eqn. (10) by taking x = a1 x1 + a2 x2 + · · · + ak xk . Then,
x ≡ ai xi ≡ ai (mod ni ) for 1 ≤ i ≤ k since aj xj ≡ 0 (mod ni ) if j 6= i.
If x0 , x00 two solutions to the simultaneous congruences in Eqn. (10), x0 ≡ ai (mod ni )
and x00 ≡ ai (mod ni ), so x0 ≡ x00 (mod ni ) or ni |(x0 − x00 ) for each i. Since ni are
pairwise coprime, N = ∏ ni also divides x0 − x00 , i.e. x0 ≡ x00 (mod N). 

Remark 4: In the proof of Theorem 4, we saw that we have to construct xi such that
xi ≡ 0 (mod nj ) for j 6= i and xi ≡ 1 (mod ni ). We constructed such an xi by taking the
solution a to the congruences in Eqn. (11) and Eqn. (12) and multiplying it by Ni . The
−1
congruence in Eqn. (11) implies that a ≡ Ni in Zni . So, if we choose N0i such that
−1
N0i = Ni in Zni , the congruence in Eqn. (13) is satisfied for a = N0i . For all j 6= i, since
Ni ≡ 0 (mod nj ), N0i Ni ≡ 0 (mod nj ). So, we choose xi such that xi = Ni N0i in ZNi ,
multiply the xi by ai and sum them up to get a solution to the congruence in Eqn. (10).
So, if Eqn. (10) is solvable, x = ∑ki=1 ai Ni N0i is a solution to it, where
−1
N = ∏ nj N i = ∏ nj N0i = Ni in Zni
j j6=i

To find the smallest non-negative solution, we take the smallest non-negative residue of
x (mod N).

Let us look at an example that illustrates the above remark.


Example 4: Solve the following congruences that we obtained in Example 3:

x ≡ 1 (mod 3)
x ≡ 3 (mod 5)
76 x ≡ 6 (mod 7)
Solution: Let us take n1 = 3, n2 = 5 and n3 = 7. Then N = 105. Also, Congruences and
Applications
−1
N1 = 35 ≡ 2 (mod 3) 2 = 2 in Z3 N01 = 2
−1
N2 = 21 ≡ 1 (mod 5) 1 = 1 in Z5 N02 = 1
−1
N3 = 15 ≡ 1 (mod 7) 1 = 1 in Z7 N03 = 1

So,

x = a1 N1 N01 + a2 N2 N02 + a3 N3 N03


= 1 · 35 · 2 + 3 · 21 · 1 + 6 · 15 · 1 = 223

So, the minimum number of students in the class is the smallest non-negative residue of
223 (mod 105) which is 13.
∗∗∗
Let us now use Theorem 4 to find more about the structure of Zn . Let n be a natural
number n = pα1 1 pα2 2 · · · pαk k . Then, since (n) ⊂ (pαi ) for 1 ≤ i ≤ k, we have ring
homomorphisms φi : Zn −→ Zpαi . Putting together the φi s, we have a ring
i
homomorphism

g : Zn −→ Zpα1 × Zpα2 · · · Zpαk , m (φ1 (m), φ2 (m), . . . , φk (m)) (15)


1 2 k

Proposition 6: The map given by Eqn. (15) is an isomorphism of rings.

Proof: Since each φi is a ring homomorphism, g is a ring homomorphism. Let


(a1 , a2 , . . . , ak ) ∈ Zpα1 × Zpα2 · · · Zpαk . Then,
1 2 k

g(m) = (a1 , a2 , . . . , ak )

if and only if m is the solution to the congruences

m ≡ a1 (mod pα1 1 )
m ≡ a2 (mod pα2 2 )
..
.
m ≡ ak (mod pαk k )

By Chinese Remainder Theorem, given any (a1 , a2 , a3 , . . . , ak ) ∈ Z∗pα1 × Z∗pα2 · · · Z∗αk ,


1 2 pk
there is always an m ∈ Z such that m ≡ ai mod pαi i . So, the map is surjective. The


map is also injective because the Chinese RemainderTheorem also says that if m, m0
are two solutions to the congruences x ≡ ai mod pαi i , then m ≡ m0 (mod n). 

As an immediate consequence of Proposition 6, we get the following result:

Corollary 6: Let n be a natural number n = pα1 1 pα2 2 · · · pαk k . Then, the map g in
Proposition 6 induces an isomorphism

g : Z∗n −→ Z∗pα1 × Z∗pα2 × · · · × Z∗pαk (16)


1 2 k

Further,
k
1
 
φ (n) = n ∏ 1 − (17)
i=1 pi

Also,

φ (mn) = φ (m)φ (n) if (m, n) = 1 (18) 77


Study Guide-I Proof: Since g is a ring isomorphism, it induces a isomorphism of the unit groups.
Further, we have
 ∗
Zpα1 × Zpα2 × · · · × Zpαk = Z∗pα1 × Z∗pα2 × · · · × Z∗pαk
1 2 k 1 2 k

This proves that the map in Eqn. (16) is an isomorphism.


We know that φ (n) = |Z∗n |. From Eqn. (16), it follows that
k
|Z∗n | = ∏ Z∗pαi

i=1

To prove Eqn. (17) it is enough to show that

|Zpα |∗ = pα−1 (p − 1) = pα − pα−1 (19)


k  k
1

and n ∏ 1 − = ∏ pαi i −1 (pi − 1) (20)
i=1 pi i=1

Note that the LHS and RHS of Eqn. (20) are trivial rearrangements of each other.
Let us now check Eqn. (19). Now,

{a | 0 ≤ a ≤ pα − 1, p | a} = kp | 0 ≤ k < pα−1


and
kp | 0 ≤ k < pα−1 = pα−1


Note that (a, pα ) = 1 ⇔ p - a. So,

|Z∗pα | = |{a | 0 ≤ a < pα − 1}| − |{a |0 ≤ a < pα − 1, p | a }| = pα − pα−1

The result in Eqn. (18) is an immediate consequence of Eqn. (17). 

EXERCISES E3) Solve the following set of simultaneous congruences:

x≡2 (mod 5)
x≡4 (mod 7)
x≡3 (mod 11)

We close this section here. In the next section, we will discuss solution of quadratic
congruences, i.e. congruences of the type x2 ≡ a (mod n).

6.4 THE QUADRATIC RECIPROCITY LAW

In this section, we will prove the quadratic reciprocity law which was proved by Gauss
in his path breaking work Disquisitiones Arithmeticae. When he did this work, he was
not even 18 years old. The result was known to Euler, Legendre and other
mathematicians, but none of them were able to prove it. Gauss called the result
L. Euler ‘Theorem Aureum’ meaning ‘Golden theorem’. He gave several proofs of the theorem.
(1707–1783) Many proofs were given by others also. The proof we will give is due to Eisenstein, one
of the gifted students of Gauss.
Let us consider the congruence x2 ≡ m (mod n) where m and n are odd. Suppose

78 n = pα1 1 pα2 2 · · · pαk k


Then, x0 is a solution to the congruence x2 ≡ m (mod n) if and only if x0 is a solution Congruences and
Applications
to the congruences

x2 ≡ m (mod pα1 1 )
x2 ≡ m (mod pα2 2 )
..
.
x2 ≡ m (mod pαk k )

Again, we have the following result:

Proposition 7: Let p be an odd prime. If x2 ≡ a (mod p) has a solution, then x2 ≡ a


(mod pk ) also has solution for k ∈ N, k ≥ 2.

If a ≡ 0 (mod p), this is easy to prove. So, let us assume that a 6≡ 0 (mod p). We can
prove Proposition 7 by starting with a root of x2 ≡ a (mod p) and repeatedly applying
the following lemma.

Lemma 4: Let k ∈ N, k ≥ 1 and p be an odd prime. If α ∈ Z is a solution to the


congruence x2 ≡ a (mod pk ), where (a, p) = 1, there is an α 0 ∈ Z such that α ≡ α 0
(mod pk ) and α 02 ≡ a (mod pk+1 )

Proof: If α 2 ≡ a (mod pk+1


),2we can
 take
0
α = α and
 we are done. So, let us assume
2 k+1 k
α 6≡ a (mod p ), i.e p α − a , p
k+1 6 α 2 − a . Therefore

α 2 − a = upk with (u, p) = 1 (21)

Consider the ‘Taylor series expansion’ of x2 − a about α:

x2 − a = α 2 − a + 2α(x − α) + (x − α)2 (22)

We need to find an α 0 such that

α 2 − a + 2α(α 0 − α) + (α 0 − α)2 ≡ 0 (mod pk+1 ) (23)


0 k
α ≡α (mod p ) (24)

If α 0 = α + vpk then Eqn. (24) is satisfied. So, if we can find a v such that α 0 = α + vpk
satisfies Eqn. (23), we are done. Let us put α 0 = α + vpk in Eqn. (23) and see if we can
solve for v. Note that p2k | (α 0 − α)2 , so pk+1 | (α 0 − α)2 . So, Eqn. (23) reduces to

upk + 2αvpk ≡ 0 (mod pk+1 ). (25)

where u is defined as in Eqn. (21). From the congruence in Eqn. (25) it follows that

u + 2αv ≡ 0 (mod p)

or
2αv ≡ −u (mod p) (26)

We can solve the last equation for v since (2α, p) = 1. This is because, if p | α, from
the congruence α 2 ≡ a (mod pk ), it will follow that p | a, a contradiction to our choice
of a. 

Because of Proposition 7, we can restrict ourselves to finding the solutions of x2 ≡ a


(mod p) where p is a prime and (a, p) = 1.

Definition 9(Quadratic Residue): We say that a ∈ Z, (a, p) = 1, is a quadratic residue


modulo p if the congruence x2 ≡ a (mod p) has a solution. 79
Study Guide-I Note that a ∈ Z, (a, p) = 1 is a quadratic residue if a is a square in Z∗p

  Let p be an odd prime and a ∈ Z be coprime to p.


Definition 10(Legendre Symbol):
Then, the Legendre Symbol pa is defined by

  (
a 1 if a is a quadratic residue
= (27)
p −1 if a is not a quadratic residue
   
a a
Remark 5: Note that pis 1 or −1 according as a is a square in Z∗p or not. So, p is
   0
determined by the residue class of a modulo p. Therefore, pa = ap if a ≡ a0
(mod p).

Example 5: Find the quadratic residues modulo 7.


Solution: From the definition, it is clear that whether a ∈ Z is a quadratic residue
modulo 7 or not depends only on whether its residue class modulo 7 is a square in Z∗7 or
not. So, let us first find all the squares in Z∗7 .

a 1 2 3 4 5 6

a2 1 4 2 2 4 1

So, a ∈ Z, (a, 7) = 1 is a quadratic residue modulo 7 if and only if a = 1, 2, 4.


∗∗∗
In the above example, we computed all the squares modulo 7 to find the quadratic
residues modulo 11. This is a very tedious procedure if the prime p is large. There is a
simple criterion due to Euler that helps us to check whether a is a quadratic residue
modulo p or not.

Theorem 5(Euler’s Criterion): If p > 2 is any prime and a ∈ Z, (a, p) = 1, then

a
 
(p−1)
a 2 ≡ (mod p) (28)
p
 
a ∗
In particular, a p induces a group homomorphism Zp −→ {1, −1}.

To prove this we need the following fact about cyclic groups.

Lemma 5: Let G be a cyclic group of order n and suppose d | n. Then, G has a unique
subgroup of order d given by

x ∈ G xd = 1 (29)


Further,
n n o
x ∈ G xd = 1 = x d x ∈ G (30)


We leave the proof of the lemma to you as an exercise.

Proof of Euler’s Criterion: Note that x xn is a group homomorphism in any


p−1
abelian group. Consider the homomorphism f : Z∗p −→ Z∗p given by x x 2 .
The group Z∗p is a cyclic group of order p − 1. We will see a proof of this in the unit on
 p−1 2
80 finite fields in Block 3, so you can take this for granted for now. We have x 2 = 1,
p−1
so x is in the unique subgroup of order 2, namely 1, −1 . In other words,

2 Congruences and
p−1 Applications
x = ±1. The result in Eqn. (28) will follow if we show that f(a) = 1 if a is a
2

quadratic residue and it is −1 if it is a quadratic non-residue. Since we already have


p−1
f(a) = a 2 ∈ 1, −1 it is enough to show that the kernel of f is precisely


x x ∈ Z∗p . If we apply Lemma 5 with n = p − 1 and d = p−1


 2
2 we have

n p−1 o 
x ∈ Z∗p x 2 = 1 = x2 x ∈ Z∗p

So, the kernel of f is x2 x ∈ Z∗p .




p−1 p−1 p−1 p−1 p−1 p−1  


a
We have (ab) 2 ≡ a 2 b 2 (mod p) since (ab) 2 =a 2 b 2
. The fact that a p
is a homomorphism follows from Eqn. (28) 

Let us now look at an example that explains how to use Eqn. (28) for finding the
legendre symbol.

 6: Find the
Example
3 19
following legendre
6
symbols:
a) b) c)

7 41 11
Solution:
7−1 3
a) We have 3 2 ≡ 33 ≡ 6 ≡ −1 (mod 7). So,

7 = −1.
41−1
b) We have to find 19 2 ≡ 1920 (mod 41). We have

192 = 361 ≡ 33 (mod 41)


∴ 194 ≡ 332 = 1089 ≡ 23 (mod 41)
∴ 198 ≡ 232 = 529 ≡ 37 (mod 41)
∴ 1916 ≡ 372 = 1369 ≡ 16 (mod 41)
∴ 1920 = 1916 194 ≡ 16.23 ≡ 40 ≡ −1 (mod 41)
19
So, 41

= −1
6 2
 3 5 2
c) We have 11 11 . We have 2 = 32 ≡ −1 (mod 11). So, 11 = −1.
 
= 11
Also, 33 = 27 ≡ 5 (mod 11), 32 ≡ 9 (mod 11). So, 35 ≡ 5.9 = 45 ≡ 1 (mod 11).
6
So, 11 = −1.1 = −1. Of course, we can evaluate 65 (mod 11) directly also.
∗∗∗
The following exercises gives you some practice in finding the legendre symbol. Also,
you will find an outline of the proof of Lemma 5 in the exercises. Try these exercises
now.

E4) Prove Lemma 5as follows: EXERCISES


a) Prove that x xd = 1 has order d. Deduce that this is the unique subgroup
of order d.
 n
b) Show that x d x ∈ G is a group of order d. Deduce Eqn. (30).

E5) Find thefollowing legendre symbols:


5 15
a) b)

11 19

Note that, the ring homomorphism ψ : Z −→ Zn induces a ring homomorphism


e : Z[x] −→ Zn [x] given by
ψ

e ∑ ai xi = ∑ ψ (ai ) xi

ψ

If p(x) ∈ Z, we will call ψ


e (p(x)) the reduction of p(x) modulo n.
Consider the polynomial x2 − 7. If we reduce it modulo 3, this polynomial becomes
x2 − 1. Since 1 is a quadratic residue modulo 3, this polynomial splits into linear factors 81
Study Guide-I in the field Zp . On the other hand, if we reduce the polynomial modulo 5, the
polynomial becomes x2 − 2 and this does not split into linear factors because
2 5−1 2
2 2 ≡ 4 ≡ −1 (mod 5), and so

5 ≡ 5 = −1.
We have the following question: Is it possible to describe all the primes p such that
x2 − 7 splits into linear factors modulo p? More generally, given a prime q, is it possible
to describe all the primes p such that x2 − q splits into linear factors modulo p? The
quadratic reciprocity law helps us to answer the question when p is an odd prime.

Theorem 6(Quadratic Reciprocity): If p and q are odd primes,

p q
   
p−1 q−1
= (−1) 2 2 (31)
q p

Remark 6: The quadratic reciprocity is stated often in the following form also.

p q
  
p−1 q−1
= (−1) 2 2 (32)
q p
 
This follows form Eqn. (31) because pq = ±1.

When p = 2, we have the following result.

Proposition 8: We have
(
2 p2 −1 1 if p ≡ ±1 (mod 8)
 
= (−1) 8 = (33)
p −1 if p ≡ ±3 (mod 8)

In other words, 2 is a quadratic residue modulo p if p ≡ ±1 (mod 8) and it is a


quadratic non residue if p ≡ ±3 (mod 8).

Regarding the polynomial x2 + 1 = 0, we have the following result:

Proposition 9: We have
  (
−1 1 if p ≡ 1 (mod 4)
= (34)
p −1 if p ≡ 3 (mod 4)

Before we prove these results, let us look at some examples.


Example 7: Describe the primes p for which x2 − 7 splits into linear factors.
Solution: For p = 2, we get the polynomial x2 − 1 when we reduce x2 − 7 modulo 2
2
and x − 1 = x2 − 1.
Let p be any odd prime. Using Eqn. (32), we have

7 p   p
p−1 3
p
 
p−1 p−1
= (−1)3( 2 ) = (−1) 2 = (−1) 2 (35)
p 7 7 7
p−1
We want to know the primes for which the RHS of Eqn. (35) is 1. It is 1 if both (−1) 2
p
and 7 are −1 or both are 1.
p−1
Let us first consider the case where (−1) 2 = 1 and p7 = 1. From Proposition 9 we


must have p ≡ 1 (mod 4). Also, from the table of squares in Example 5, we have
a ≡ 1, 2 or 4 (mod 7). So, p should satisfy one of the following set of congruences:

p ≡ 1 (mod 4) p ≡ 1 (mod 4) p ≡ 1 (mod 4)


82 p ≡ 1 (mod 7) p ≡ 2 (mod 7) p≡4 (mod 7)
As we saw in the previous section, we will first solve the congruences Congruences and
Applications
x1 ≡ 1 (mod 4) x2 ≡ 0 (mod 4)
x1 ≡ 0 (mod 7) x2 ≡ 1 (mod 7)
The solutions are x1 = 21 and x2 = 8. So, the solution of the congruences
p ≡ 3 (mod 4)
p ≡ 1 (mod 7)
is x1 + x2 = 29 ≡ 1 (mod 28). Here is the complete table:
Congruences Solution
p ≡ 1 (mod 4) x1 + x2 = 29 ≡ 1 (mod 28)
p ≡ 1 (mod 7)
p ≡ 1 (mod 4) x1 + 2x2 = 21 + 16 = 37 ≡ 9 (mod 28)
p ≡ 2 (mod 7)
p ≡ 1 (mod 4) x1 + 4x2 = 21 + 32 = 53 ≡ 25 (mod 28)
p ≡ 4 (mod 7)
p−1
The other possibility is (−1) 2 = −1 and 7p = −1. In this case p ≡ 3 (mod 4) and


p ≡ 3, 5 or 6 (mod 7). As before, this leads to the following set of congruences:


p ≡ 3 (mod 4) p ≡ 3 (mod 4) p ≡ 3 (mod 4)
p ≡ 3 (mod 7) p ≡ 5 (mod 7) p ≡ 6 (mod 7)
Here is the complete table:
Congruences Solution
p ≡ 3 (mod 4) 3x1 + 3x2 = 63 + 24 = 87 ≡ 3 (mod 28)
p ≡ 3 (mod 7)
p ≡ 3 (mod 4) 3x1 + 5x2 = 63 + 40 = 103 ≡ 19 (mod 28)
p ≡ 5 (mod 7)
p ≡ 3 (mod 4) 3x1 + 6x2 = 63 + 48 ≡ 27 (mod 28)
p ≡ 6 (mod 7)
So, x2 − 7 splits completely modulo p for an odd prime p if p ≡ 1, 3, 9,19, 25, 27
(mod 28).
∗∗∗

Remark 7: Note that, if we don’t have equation Eqn. (35), to check  whether
 x2 − 7
splits into linear factors modulo a prime p, we will be forced to find p7 for each p.
However, with the help of Eqn. (35), we are able to reduce this to checking whether p is
in one of the finitely many residue classes modulo p, which is much easier to do!
For example, if we want to check whether x2 − 7 splits in 263081503 or not, we need
263081503−1
not compute 7 2 = 7131540751 (mod 263081503). We find that 263081503 ≡ 27
(mod 28) and 27 figures in the list of residue classes we have obtained in Example 7.
So, x2 − 7 splits into linear factors modulo 263081503!

Let us now prove quadratic reciprocity. The proof is along the lines of the proof in the
book Course in Arithmetic by J. P. S ERRE, pages 9—10. For proving quadratic
reciprocity, we need some preliminary results.
Let p be a prime. Let S be any set such that Z∗p is the disjoint union of S and −S where
−S = {−s|s ∈ S}. The set {1, 2, . . . , p−1
2 } has this property. So, we will choose
S = {1, 2, . . . , p−1 ∗
2 }. For s ∈ S and a ∈ Zp , either sa or −sa is in S. So, we can write
sa = es (a)sa where es (a) = ±1 and sa ∈ S. Note that es (a) = 1 if as ∈ S and es (a) = −1,
if as ∈ −S. For example, let us take p = 7, S = {1, 2, 3}. If a = 6, s = 3,
sa = 18 ≡ 4 ≡ −3 ≡ (−1)3 (mod 7). So, e3 (6) = −1 and 63 = 3 in this case. 83
Study Guide-I Proposition 10(Gauss Lemma): For any prime p and a ∈ Z, p - a

a
 
= ∏ es (a) (36)
p s∈S

Proof: If s and s0 are two distinct elements of S, then sa 6= sa0 . If sa = sa0 , then
es (a)as = es0 (a)as0 or es (a)s = es0 (a)s0 . Therefore, s = ±s0 , which contradicts the choice
of S. So, s sa is a bijection of S to itself. Multiplying the equalities as = es (a)sa , we
get
! !
p−1
a 2
∏ s = ∏ es (a) ∏ sa = ∏ es (a) ∏ s
s∈S s∈S s∈S s∈S s∈S

Hence
!
p−1
a 2 = ∏ es (a)
s∈S

The result now follows from Euler’s criterion. 

We need a few more auxiliary lemmas to prove the quadratic reciprocity.

Lemma 6: For all n ≥ 1, we have

1 1 2n+1 n−1 1 2i+1


   
2n+1
x − 2n+1 = x − + ∑ ai,n x − (37)
x x i=0 x

where ai,n ∈ Z.

The proof is not difficult. First, verify it for n = 1, 2, 3. You will be able to prove the
lemma with the insight gained from this. We leave it to you as an exercise.
We also need the following trigonometric lemma.

Lemma 7: We have
sin(2` + 1)x 2 2πj
 
2
= (−4) ∏ sin x − sin
`
(38)
sin x 1≤j≤` 2` + 1

Proof: Let us divide Eqn. (37) by x − 1x to get


1
x2`+1 − x2`+1 1 2` `−1 1 2i
   
= x− + ∑ ai,` x − . (39)
x − 1x x i=0 x

Let us substitute eix for x in Eqn. (39). Then, LHS of Eqn. (39) becomes
1
x2`+1 − x2`+1 e(2`+1)ix − e−(2`+1)ix sin(2` + 1)x
= = (40)
x − 1x eix − e−ix sin x

The RHS of Eqn. (39) becomes


`−1 `−1
(2i)2` sin2` x + ∑ aj,` (2i)2j sin2j x = (−4)` sin2` x + ∑ (−4)j aj,` sin2j x
j=1 j=1

Let us write
`−1
P(T) = (−4)` T` + ∑ (−4)j aj,` Tj (41)
84 j=1
Then, Congruences and
Applications
sin(2` + 1)x
= P(sin2 x) (42)
sin x
So, we have
2πj
2 2πj
sin(2` + 1) 2`+1
 
P sin = 2πj
= 0 for 1 ≤ j ≤ ` (43)
2` + 1 sin 2`+1

In other words,
2πj
sin2 , 1≤j≤`
2` + 1
are the roots of the polynomial P(T). So,

2 2πj
 ` 
P(T) = (−4) ∏ T − sin
`
j=1 2` + 1

Setting T = sin2 x in the last equation we get the required result. 

We can now prove quadratic reciprocity.

Proof of Quadratic reciprocity: Let p and q be distinct, odd primes. As before, let

p−1
S = {1, 2, . . . , }
2
From Proposition 10, Gauss lemma, we get

q
 
= ∏ es (q)
p s∈S

From qs = es (q)sq , we have

2π 2π
sin qs = es (q) sin sq
p p
0
(Note that, if a ≡ a0 (mod p), then sin 2πa
p = sin
2πa
. This is because we can write
 p 
2πa0 0
a = a0 + pr for some r ∈ Z and so sin 2πa
p = sin 2rπ + p = sin 2πa
p . So, it makes
sense to write sin 2πs ∗
p for s ∈ Zp .)
Multiplying these equations and taking into account that s sq is a bijection, we get

q sin 2πqs
 
p
= ∏ es (q) = ∏ 2πs
p s∈S s∈S sin p

By applying Lemma 7 with q = 2` + 1 we can write this as

q 2 2πs 2 2πt
   
q−1
= ∏(−4) ∏ sin2 − sin
p s∈S t∈T p q
2 2πs 2 2πt
 
(q−1)(p−1)
= (−1) 4
∏ sin p − sin q
s∈S,t∈T

where T is the set


q−1
 
1, 2, . . . ,
2 85
Study Guide-I Interchanging the roles of p and q, we obtain similarly

p 2πt 2πs
   
(q−1)(p−1)
2 2
q
= (−1) 4 ∏ sin q − sin p
s∈S,t∈T
   
The factors giving qp and qp are identical up to sign. Since there are (p−1)(q−1)
4 of
these, we have

q p
   
(p−1)(q−1)
= (−1) 4
p q

Let us now prove Proposition 8 and Proposition 9.

Proof of Proposition 8: We use Gauss lemma, Proposition 10, to prove this. Let us
p−1 p−1
take a = 2 and S = {1, 2, . . . , 2 }. We have es (2) = 1 if 2s ≤ 2 and es (2) = −1
2
otherwise. From this, we get p = (−1)n(p) where n(p) is the number of integers s
p−1 p−1
such that 4 <s≤ 2 .

Case 1: p ≡ 1 (mod 4). Let p = 1 + 4k. Then, p−1 p−1


4 = k, 2 = 2k and n(p) is the
number of s with k < s ≤ 2k. So, n(p) = k in this case. Therefore,

n(p) is even ⇔ k is even ⇔ p = 4(2m) + 1 = 8m + 1


n(p) is odd ⇔ k is odd ⇔ p = 4(2m + 1) + 1 = 8m + 5

Therefore
  (
2 1 if p ≡ 1 (mod 8)
=
p −1 if p ≡ 5 (mod 8)

Case 2: p ≡ 3 (mod 4). Let p = 4k + 3. Then, p−1 1 p−1


4 = k + 2 , 2 = 2k + 1 and n(p) is
the number of s with k + 1 ≤ s ≤ 2k + 1. So, n(p) = k + 1. Therefore,

n(p) is even ⇔ k is odd ⇔ p = 4(2m + 1) + 3 = 8m + 7


n(p) is odd ⇔ k is even ⇔ p = 4(2m) + 3 = 8m + 3
  (
2 1 if p ≡ 7 (mod 8)
=
p −1 if p ≡ 3 (mod 8)
To complete the proof, note that 7 ≡ −1 (mod 8) and 5 ≡ −3 (mod 8). 
Let us now prove Proposition 9.

Proof of Proposition 9: If −1 is a square in Z∗p , say y2 = −1, then y4 = 1 and y2 6= 1.


So, y is an element of order 4 and hence 4 | p − 1, i.e. p ≡ 1 (mod 4). Conversely, if
p ≡ 1 (mod 4), 4 | p − 1. Since Z∗p is cyclic there is an element y ∈ Z∗p of order 4. Then
2
y2 6= 1 since y has order 4. Also y2 = y4 = 1, so y2 = −1 and so −1 is a square in
Z∗p . 

EXERCISES E6) Prove Lemma 6.

We close this section here. In the next section we will discuss some applications of
86 congruences to primality testing and in checking bar codes.
Congruences and
Applications
6.5 APPLICATIONS OF CONGRUENCES

In this section, we will discuss some applications of congruences. First, we will discuss
some applications of congruences to primality testing. Then, we will discuss the use of
check digits to check errors in International Standard Book Number(ISBN), which is a
unique number assigned to books.

6.5.1 Primality Testing

The next result gives a necessary and sufficient condition for a number to be a prime.

Proposition 11: An integer p > 1 is a prime if and only if (p − 1)! ≡ −1 (mod p).

Proof: Suppose p is a prime. Then, Zp is a finite field and so Z∗p is cyclic. If p = 2, the
result is trivially true. So, let us assume that p > 2 is an odd prime. Note that, if
a = a−1 , a2 = 1 and so a = −1 or 1 since these are the only elements in Z∗p that satisfy
x2 = 1. This is because Z∗p is a cyclic group, there is a unique cyclic subgroup of order
2 and all the elements in Z∗p that satisfy x2 = 1 must lie in this subgroup. Since the
cyclic subgroup generated by −1 has order 2, this must be the unique subgroup of order
2. So, for all the other elements of Z∗p , a 6= a−1 . Therefore the terms in the product
∏ a can be grouped into pairs of the form a a−1 . So, this product is 1. Therefore,
a6=1,−1

∏ a = 1.−1. ∏ a = −1 (44)
a∈Z∗p a6=1,−1

On the other hand,


p−1
∏ a = ∏ i = (p − 1)! (45)
a i=1

From Eqn. (44) and Eqn. (45), we get (p − 1)! = −1 in Z∗p . Translating this in the
language of congruences, we get the result.
Conversely, suppose that (p − 1)! ≡ −1 (mod p), i.e. p | (p − 1)! + 1. If p is not a
prime, it has a prime factor q, 1 < q < p. Since q occurs in the product (p − 1)!, it
doesn’t divide (p − 1)! + 1. But, since q | p and p | (p − 1)! + 1, q has to divide
(p − 1)! + 1, a contradiction. 

The necessary part of the Proposition 11 is known as Wilson’s theorem.


Note that the result is not very useful for primality testing because computing (p − 1)!
is very time consuming. In practice, we use probabilistic algorithms for primality
testing. In these algorithms, there is a small probability that the answer may be wrong.
It may wrongly say that a composite number is a prime with a small probability.
However, it says a number is composite, it will be correct. Here is one such simple
algorithm based on Fermat’s little theorem.

Proposition 12: If (a, n) = 1 and an−1 6≡ 1 (mod n) for some a, then n is not a prime.

Note that Proposition 12 is simply the contrapositive of Fermat’s little


theorem. Proposition 12 provides a method for testing whether a number n is prime. If,
for a number a, 1 < a < n − 1 n does not satisfy Eqn. (8) for some a, it is a composite
number. However, the converse is not true. In other words, there are composite
numbers that satisfy Eqn. (8) for all a with (a, n) = 1. Such numbers are called
Carmichael numbers. For example, 561 is a Carmichael number. 87
Study Guide-I A composite number n that satisfies Eqn. (8) is called base-a pseudoprime.
A practical and widely used test for primality is the Rabin-Miller test. It is based on the
following proposition.

Proposition 13: Let p be an odd prime and let (a, p) = 1. Suppose p − 1 = r2t with r
odd. Then, a satisfies at least one of the following conditions:
i) ar ≡ 1 (mod p)
i
ii) a2 r ≡ −1 (mod p) for some i, 0 ≤ i < t.

Proof: If ar ≡ 1 (mod p), we are done. Suppose ar 6≡ 1 (mod p). Consider the set
m
n o
S = m | a2 r ≡ 1 (mod p)
t
We have a2 r = ap−1 ≡ 1 (mod p). So, S 6= 0/ because t ∈ S. Also, 0 6∈ S because we
have assumed ar 6≡ 1 (mod p). Let t0 = min S. Then, t0 ≥ 1. Also, by choice of t0 ,
t −1
 t −1 2 t t −1
ar2 0 6≡ 1 (mod p). However, ar2 0 = ar2 0 ≡ 1 (mod p), so ar2 0 ≡ −1
(mod p). 

To apply Rabin-Miller test, we proceed as follows: To check whether a number n is


composite, we choose a a less than n and check if (a, n) = 1. If (a, n) = d 6= 1, d is a
factor of n and so n is composite. If (a, n) = 1 we check ar ≡ ±1 (mod p). If ar ≡ ±1
(mod p), we conclude that p is a ‘probable prime’. We move on to next a. If ar 6≡ ±1
(mod p), we repeatedly square ar modulo p. If we get −1 at some point, then n is
probably a prime and we move on to next a. If we do not get −1 at any stage even after
squaring ar t times, then n is a composite number. Suppose, after checking for k
different values of a, we get the answer that n is a prime number, the probability of the
answer being wrong is less than 21k .(We will not prove this in our course.) So, for a
large number n, after testing for 100 values of a, if the test says the number is prime, the
1
probability that the answer is wrong is less than 2100 which is very negligible.
The above tests are for general primes. For primes of the special form, there are special
tests. One such test is the Lucas-Lehmer test for Mersenne primes which are primes of
the form 2p − 1 where p is a prime.
Let us write Mp = 2p − 1. We define the sequence Sn inductively by S1 = 4,
Sn = S2n−1 − 2.

Theorem 7(Lucas-Lehmer): Mp is a prime if and only if Mp divides Sp−1 .

We will prove the implication only in one direction in this course. We will only show
that Mp is a prime if Mp divides sp−1 .
√ √
Let ω = 2 + 3 and ω = 2 − 3. You can check that ωω = 1.

Lemma 8:
m−1 m−1
Sm = ω 2 + ω2

You can easily prove this by induction. We leave this to you as an exercise.
For an odd prime q, let X denotes the set {(a, b) | a, b ∈ Zq }. We can define binary
operations, addition and multiplication, as follows:
(a, b) + (c, d) = (a + c, b + d)
(a, b)(c, d) = (ac + 3bd, ad + bc)
Then, X is a commutative ring with an identity element and it has q2 elements. So, X∗ ,
88 the group of units in X, has at most q2 − 1 elements.
Proposition 14: Mp is a prime if Mp divides Sp−1 . Congruences and
Applications
p−2 p−2
Proof: If Mp | Sp−1 , from Lemma 8, it follows ω 2 + ω 2 ≡ 0 (mod Mp ), so
p−2 p−2 p−2
ω 2 + ω 2 = NMp for some integer N. Multiplying this by ω 2 we find that
p−1 p−2
ω2 = NMp ω 2 − 1. (46)

Squaring

p
 p−2
2
ω 2 = NMp ω 2 − 1 (47)

Suppose Mp is composite. 2
√ Then, there is a prime divisor q of√Mp , q odd, with q ≤ Mp .
Consider
√ the ring {a + b 3 | a, b ∈ Z}. Then, the map f : Z[ 3] −→ X,
a+ b 3 (a, b) gives aring homomorphism. Consider f(ω). Since q | Mp ,
p−1

= 0. So, from Eqn. (46) and Eqn. (47), f (ω)2
p−1 p−1
f NMp ω 2 = NMp f ω 2 = −1
2p
and f (ω) = 1. (Here we need that fact that q is an odd prime.) So, f(ω) ∈ X∗ has
order 2p . The order of f(ω) divides |X∗ | so 2p ≤ q2 − 1. But, q2 − 1 ≤ Mp − 1 = 2p − 2
and we have a contradiction. 

Here are some exercises that asks you to fill in the details in the proof of Proposition 14.

E7) Prove Lemma 8. EXERCISES


E8) Verify that X is a commutative ring with (1, 0) as the identity element.

6.5.2 Error Checking Using Congruences

You know that data is transmitted as strings of 0s and 1s. Suppose we transmit a string
x1 x2 . . . xn where each xi is 0 or 1. We add one more digit xn+1 so that

x1 + x2 + · · · + xn ≡ xn+1 (mod 2)

so that

x1 + x2 + · · · + xn + xn+1 ≡ 0 (mod 2)

Suppose we transmit the string x1 x2 . . . xn+1 and the recipient receives the string
y1 y2 . . . yn+1 . The recipient checks if y1 + y2 + · · · + yn+1 ≡ 0 (mod 2). If it is not, at
least one of the bits xi has been changed and the recipient can ask us to transmit the
string again.
Of course, this method can detect only one error, i.e. if one of the zeros has been
changed to 1 or a 1 has been changed to 0. If two bits are changed during transmission,
the test can’t detect it.
Every book published recently has a unique number associated to it called the ISBN
number. This is a sequence of 9 digits x1 , x2 , . . . , x9 together with a check digit
x10 ∈ {0, 1, 2, . . . , 9, X}. We use the single digit X to represent 10. We can find the
check digit of the number from the first 9 digits and by comparing it with the check
digit, we can check whether the ISBN number is correct or not.
Suppose we order a book over the telephone and give the dealer the ISBN number of
the book. How can the dealer make sure that he has correctly noted down the ISBN
number? Let us see how the dealer can check whether the ISBN number is correct
using the arithmetic modulo 11. 89
Study Guide-I Let us now look at an example to know how to use the check digit.
Example 8: The ISBN number of the prescribed text book for the course, Artin’s
Algebra book is 81-203-0871-9. The last digit is the check digit. Check whether the
number is correct using the check digit.
Solution: Let us first find the check digit from the first 9 digits and see if it matches
with the check digit for the book which is the last digit, namely 9. We have

8 1 2 0 3 0 8 7 1
l l l l l l l l l
x1 x2 x3 x4 x5 x6 x7 x8 x9

We can calculate the check digit as follows:


i) Multiply x1 by 1, x2 by 2,. . ., x9 by 9.
i) Add all the products and reduce it modulo 11. Then, n, 0 ≤ n ≤ 10, such that
n ≡ ∑9i=1 ixi is the check digit.
If the check digit matches the 10th digit, the ISBN number is correct.
Thus,
9
x10 = ∑ ixi
i=1
= [1 · 8 + 2 · 1 + 3 · 2 + 4 · 0 + 5 · 3 + 6 · 0 + 7 · 8 + 8 · 7 + 9 · 1] (mod 11)
≡ 9 (mod 9)

So, the ISBN number is valid.


∗∗∗
Here is an exercise for you to try.

EXERCISES E9) Check whether the ISBN number 0-387-97329-X is a valid ISBN number.

Another way to to look at computation is as follows: We can look at the ISBN Number
as an element of Z911 by putting it in the form (8, 1, 2, 0, 3, 0, 8, 7, 1). If (a1 , a2 , . . . , a9 )
and (b1 , b2 , . . . , b9 ) are in Z911 , consider ‘dot product’:

(a1 , a2 , . . . , a9 ) · (b1 , b2 , . . . , b9 ) = a1 b1 + a2 b2 + · · · + a9 b9

Let us write α = 1, 2, · · · , 9 . Then, the check digit is the number i, 0 ≤ i ≤ 10, such


that i ≡ α · x (mod 11) where x is the 9 digit ISBN number regarded as an element of
Z911 .
From 2007, all the new books will have a 13 digit ISBN number instead of 10 digits.
The check digit is calculated as follows:
Suppose x1 x2 . . . x12 are the first 12 digits of the ISBN number:
i) Starting from the left, multiply the first digit by 1, the second digit by 3, the third
digit by 1 again, fourth digit by 3 again, etc. and add them up. In other words, we
multiply the odd digits by 1 and even digits by 3 and add them up.
ii) Reduce the number modulo 10 and subtract it from 10, i.e. take the additive
inverse modulo 10 of the number. This will give the check digit.
Let us look at an example now.
Example 9: Calculate the check digit of the ISBN number 978-0-11-000222. We have
(9 + 8 + 1 + 0 + 0 + 2) + 3(7 + 0 + 1 + 0 + 2 + 2) = 20 + 36 = 56 ≡ 6 (mod 10) and
−6 ≡ 4 (mod 10). So, the check digit is 4.
90 ∗∗∗
We regard the 12 digit ISBN number as an element of Z12 9
10 instead of Z11 . We let
Congruences and
12
β = (1, 3, 1, 3, 1, 3, 1, 3, 1, 3, 1, 3). If x ∈ Z10 is any ISBN number, the check digit is the Applications

number i, 0 ≤ i ≤ 9 such that i + β .x ≡ 0 (mod 10).


Let us now discuss the mathematics behind the check digits. We fix a modulus ` and
consider a sequence of mappings σ1 , σ2 , . . . , σ` from Z` to Z` . For any string of
elements a1 , a2 , . . . , a`−1 , we add an extra digit a` such that

σ1 (a1 ) + σ2 (a2 ) + · · · + σ` (a` ) ≡ 0 (mod `) (48)

We call the sequence σ1 , σ2 , . . . , σ` a check digit scheme. Typically, σ` is chosen as


identity map or the negative of the identity map.
A single error, where one of the digits ai is replaced by a0i can be detected only if
σi (ai ) 6= σ (a0i ) (mod `), i.e. σi is a permutation. A transposition error where
. . . ai ai+1 . . . aj aj+1 . . . is replaced by . . . aj ai+1 . . . ai aj+1 . . . will be detected only if
σi (ai ) + σj aj 6= σi aj + σj (ai ) (mod `).
Typically, we use left multiplication by an element in Z∗` so that each of the σi is a
permutation. Suppose σi (a) = αi · a (mod `). Then, we can easily determine which
errors are not possible to detect.

Proposition 15: Suppose σi is the map a αi a and an identification number


(a1 , a2 , . . . , an ) satisfies
n
∑ αi ai ≡ 0 (mod `)
i=1

Then, a single position error ai → a0i is undetectable if and only if αi (ai − a0i ) ≡ 0
(mod `) and a transposition error that
 interchanges ith position and jth position is
undetectable if and only if ai − aj αi − αj ≡ 0 (mod `)


Proof: Suppose there is a single error in the ith position, ai has been replaced by a0i . Let
a be the correct number and let b be the number with ith digit changed. Then the
difference α · (a − b) is (ai − a0i )αi . We will not be able to detect this error if and only if
(ai − a0i )αi ≡ 0 (mod `).
Suppose there is a transposition error where . . . ai ai+1 . . . aj aj+1 . . . is replaced by
. . . aj ai+1 . . . ai aj+1 . . .. Once again, let a be the correct number and
 b be the wrong
number. The difference α(a − b) = αi ai + αj aj − αj ai + αi aj = αi − αj ai − aj .
  

So, the error is undetectable if and only if αi − αj ai − aj ≡ 0 (mod `). 

6.6 SUMMARY
In this Unit, we have discussed the following:

1. Method for solving linear congruences ( mod n);


2. How to use Chinese Remainder Theorem to solve simultaneous linear
congruences;
3. How to calculate the legendre symbol;
4. How to solve the equation x2 − a = 0 (mod p), when p is a prime and a and p are
odd numbers coprime to each other, using quadratic reciprocity;
5. How we can use congruences for checking primality and for checking whether an
ISBN number is valid number or not.
91
Study Guide-I
6.7 SOLUTIONS/ANSWERS

E1) i) We have a ≡ b (mod n) is equivalent to ψ(a) = ψ(b) and c ≡ d (mod n) is


equivalent to ψ(c) = ψ(d).To show that a + c ≡ b + d (mod n), we have to
show that a + c = b + d. We have

a + c = ψ(a + c) = ψ(a) + ψ(c)


= ψ(b) + ψ(d)
= ψ(b + d) = b + d

We leave the proof of the remaining part to you as an exercise.


−1
E2) i) We have to find 3 ≡ mod 17. So, we have to find u and v such that
3u + 17v = 1. We have17 = 5 · 3 + 2, so x1 = −q1 = −5, y1 = 1. Again,
3 = 2 · 1 + 1. So, q2 = 1, x2 = 1 + q1 q2 = 6, y2 = −q2 = −1. Now, d1 = 2,
d2 = 1 and d2 | d1 . So, u = x2 = 6, v = y2 = −1. So, 6 · 3 − 17 = 1 for
−1
3 = 6. So, x ≡ 6 · 2 (mod 17) or x ≡ 12 (mod 17).
ii) Here, (4, 18) = 2, and 2 | 6, so this has a solution. We first divide both sides
of the congruence by 2 to get 2x ≡ 3 (mod 9). We proceed as before and
−1
find u = −4, v = 1 satisfy 2u + 9v = 1. So, 2 = −4 = 5. From 2x ≡ 3
(mod 9), we get x ≡ 15 ≡ 6 (mod 9).

E3) We take n1 = 5, n2 = 7 and n3 = 11. Then


−1
N1 = 77 ≡ 2 (mod 5) 2 = 3 inZ5 N01 = 3
−1
N2 = 55 ≡ 6 (mod 7) 6 = 6 inZ7 N02 = 6
−1
N3 = 35 ≡ 2 (mod 11) 6 = 6 inZ7 N03 = 6

So,

x = a1 N1 N01 + a2 N2 N02 + a3 N3 N03


= 2 · 77 · 3 + 4 · 55 · 6 + 3 · 35 · 6 = 2412

The smallest non-negative solution is the smallest non-negative residue of 2412


(mod 385) which is 102.

E4) Let K = x ∈ G xd = 1 . Since G is cyclic, let G = hgi.




a) Suppose xd = 1. Let x = gm , 0 ≤ m ≤ n − 1. Then xd = gmd = 1, so n | md or


n n n
d | m. Let m = k d . Since 0 ≤ m < n, 0 ≤ k d < n or 0 ≤ k < d. So, there are
at most d values for k. Therefore, there are at most d elements in G satisfying
n
xd = 1. On the other hand g d satisfies xd = 1 and it generates a subgroup of
order d and every x element of this group will satisfy xd = 1. Further, if H
any subgroup of G of order d, every element x ∈ H will also satisfy xd = 1
and so H ⊂ K. Since |H| = |K|, H = K.
b) It is a subgroup of K defined in the solution to part a). You have to prove that
n
g d has order d. The result will then follow.
E5) a) We have to find 55 (mod 11). We have

52 = 25 ≡ 3 (mod 11)
4 2
5 ≡ 3 ≡ 9 (mod 11)
55 ≡ 9 × 5 = 45 ≡ 1 (mod 11)
5
92 So, 11 = 1. We leave part b) to you.

1
2n+1
E6) Proof:Note that, the lemma says that x2n+1 − x2n+1 is the sum of x − x1 with Congruences and
Applications
a polynomial in

1 1 3 1 2n−1
     
x− , x− ,..., x−
x x x
with integer coefficients.
We apply induction on n. For n = 1, we have

1 1 3 1
   
3
x − = x− +3 x−
x x x
So, the result is true for n = 1.
Suppose for all k ≤ n − 1, we have

1 1 2k+1 k−1 1 2i+1


   
x2k+1 − 2k+1 = x − + ∑ ai,k x − (49)
x x i=0 x

where ai,k ∈ Z.

1 2n+1 n
1
 
x− = x2n+1 + ∑ (−1)i C(2n + 1, i)x2n+1−i −i
x i=1 x
2n
1 1
+ ∑ (−1)i C(2n + 1, i)x2n+1−i − 2n+1
i=n+1 x −i x
1 1 2n+1 n
 
∴ x2n+1 − = x− − ∑ (−1)i C(2n + 1, i)x2n+1−2i
x2n+1 x i=1
2n
− ∑ (−1)i C(2n + 1, i)x2n+1−2i (50)
i=n+1

To complete the proof, we have to show that


n 2n
∑ (−1)i C(2n + 1, i)x2n+1−2i + ∑ (−1)i C(2n + 1, i)x2n+1−2i (51)
i=1 i=n+1

is a polynomial in

1 1 3 1 2n−1
     
x− , x− ,..., x−
x x x
with integer coefficients.
We now group the term in the first sum corresponding to i = 1, which is
−C(2n + 1, 1)x2n−1 , with the term corresponding to i = 2n in the second sum
which is

(−1)2n C(2n + 1, 2n)x−(2n−1) = C(2n + 1, 2n)x−(2n−1)


= C(2n + 1, 1)x−(2n−1) (∵ C(n, r) = C(n, n − r))

We get the term


1
 
−C(2n + 1, 1) x2n−1 −
x2n−1
Similarly, we group together the term corresponding to i = 2 in the first sum with
the term corresponding to 2n − 1 in the second sum to get
1
 
C(2n + 1, 2) x2n−3 − 2n−3
x 93
Study Guide-I In general, we group the term corresponding to i = m in the first sum and the term
corresponding to 2n − (m − 1) in the second sum. The term corresponding to
i = m in the first sum is

(−1)m C(2n + 1, m)x2(n−m)+1 (52)

The sum corresponding to i = 2n − (m − 1) is the second term is

1
(−1)2n−m+1 C(2n + 1, 2n − m + 1)x2n+1−(2n−m+1)
x2n−m+1
= −(−1)m C(2n + 1, 2n − m + 1)x2n+1−(2n−m+1)−(2n−m+1)
= −(−1)m C(2n + 1, m)x−(2(n−m)+1) (53)

Grouping the terms in Eqn. (52) and Eqn. (53) together, we get the term
 
(−1)m C(2n + 1, m) x2(n−m)+1 − x−(2(n−m)+1)

Thus, the sum in Eqn. (51) equals


n  
i 2(n−i)+1 −(2(n−i)+1)
∑ (−1) C(2n + 1, i) x − x (54)
i=1

Since 2(n − i) + 1 ≤ 2(n − 1) + 1, by induction hypothesis, for i ≤ n − 1, we

x2(n−i)+1 − x−(2(n−i)+1)

is a polynomial in

1 1 3 1 2(n−i)+1
     
x− , x− ,..., x−
x x x

with integer coefficients. So, it now follows that


n  n−1  1 2i+1
 
− ∑ (−1)i C(2n + 1, i) x2(n−i)+1 − x−(2(n−i)+1) = ∑ an,i x −
i=1 i=1 x

for some an,i ∈ Z. 

E7) Since ω + ω = 4, it is true for m = 1. Apply induction on m using the fact that
m−1 2
 m−1  m m
ω2 + ω2 − 2 = ω2 + ω2

since ωω = 1.
E8) We have a, b c, d = ac + 3bd, ad + bc = ca + 3db, da + cb = c, d a, b
     

E9) 1 · 0 + 2 · 3 + 3 · 8 + 4 · 7 + 5 · 9 + 6 · 7 + 7 · 3 + 8 · 2 + 9 · 9 = 10 (mod 11). So, the


number is correct.

94
Errata Errata
Page Replace By
i i
88 a2 r ≡ −1 (mod p) for some i, 0 ≤ i ≤ a2 r ≡ −1 (mod p) for some i, 0 ≤ i <
t. t.

95

You might also like